Adv Med Surg Final

Lakukan tugas rumah & ujian kamu dengan baik sekarang menggunakan Quizwiz!

35. A patient with cirrhosis has experienced a progressive decline in his health; and liver transplantation is being considered by the interdisciplinary team. How will the patients prioritization for receiving a donor liver be determined? A) By considering the patients age and prognosis B) By objectively determining the patients medical need C) By objectively assessing the patients willingness to adhere to post-transplantation care D) By systematically ruling out alternative treatment options

Ans: B Feedback: The patient would undergo a classification of the degree of medical need through an objective determination known as the Model of End-Stage Liver Disease (MELD) classification, which stratifies the level of illness of those awaiting a liver transplant. This algorithm considers multiple variables, not solely age, prognosis, potential for adherence, and the rejection of alternative options.

11. A patient undergoing a hip replacement has autologous blood on standby if a transfusion is needed. What is the primary advantage of autologous transfusions? A) Safe transfusion for patients with a history of transfusion reactions B) Prevention of viral infections from another person's blood C) Avoidance of complications in patients with alloantibodies D) Prevention of alloimmunization

Ans: B Feedback: The primary advantage of autologous transfusions is the prevention of viral infections from another person's blood. Other secondary advantages include safe transfusion for patients with a history of transfusion reactions, prevention of alloimmunization, and avoidance of complications in patients with alloantibodies.

18. A patient with a diagnosis of syndrome of inappropriate antidiuretic hormone secretion (SIADH) is being cared for on the critical care unit. The priority nursing diagnosis for a patient with this condition is what? A) Risk for peripheral neurovascular dysfunction B) Excess fluid volume C) Hypothermia D) Ineffective airway clearance

Ans: B Feedback: The priority nursing diagnosis for a patient with SIADH is excess fluid volume, as the patient retains fluids and develops a sodium deficiency. Restricting fluid intake is a typical intervention for managing this syndrome. Temperature imbalances are not associated with SIADH. The patient is not at risk for neurovascular dysfunction or a compromised airway.

31. A nurse who is a member of the local disaster response team is learning about blast injuries. The nurse should plan for what event that occurs in the tertiary phase of the blast injury? A) Victims' pre-existing medical conditions are exacerbated. B) Victims are thrown by the pressure wave. C) Victims experience burns from the blast. D) Victims suffer injuries caused by debris or shrapnel from the blast.

Ans: B Feedback: The tertiary phase of the blast injury results from the pressure wave that causes the victims to be thrown, resulting in traumatic injury. None of the other listed events occurs in this specific phase of a blast.

30. A patient diagnosed with myasthenia gravis has been hospitalized to receive plasmapheresis for a myasthenic exacerbation. The nurse knows that the course of treatment for plasmapheresis in a patient with myasthenia gravis is what? A) Every day for 1 week B) Determined by the patients response C) Alternate days for 10 days D) Determined by the patients weight

Ans: B Feedback: The typical course of plasmapheresis consists of daily or alternate-day treatment, and the number of treatments is determined by the patients response.

28. A patient with a history of injection drug use has been diagnosed with hepatitis C. When collaborating with the care team to plan this patients treatment, the nurse should anticipate what intervention? A) Administration of immune globulins B) A regimen of antiviral medications C) Rest and watchful waiting D) Administration of fresh-frozen plasma (FFP)

Ans: B Feedback: There is no benefit from rest, diet, or vitamin supplements in HCV treatment. Studies have demonstrated that a combination of two antiviral agents, Peg-interferon and ribavirin (Rebetol), is effective in producing improvement in patients with hepatitis C and in treating relapses. Immune globulins and FFP are not indicated.

3. A nurse is caring for a patient with impaired renal function. A creatinine clearance measurement has been ordered. The nurse should facilitate collection of what samples? A) A fasting serum potassium level and a random urine sample B) A 24-hour urine specimen and a serum creatinine level midway through the urine collection process C) A BUN and serum creatinine level on three consecutive mornings D) A sterile urine specimen and an electrolyte panel, including sodium, potassium, calcium, and phosphorus values

Ans: B Feedback: To calculate creatinine clearance, a 24-hour urine specimen is collected. Midway through the collection, the serum creatinine level is measured.

21. An industrial site has experienced a radiation leak and workers who have been potentially affected are en route to the hospital. To minimize the risks of contaminating the hospital, managers should perform what action? A) Place all potential victims on reverse isolation. B) Establish a triage outside the hospital. C) Have hospital staff put on personal protective equipment. D) Place hospital staff on abbreviated shifts of no more than 4 hours.

Ans: B Feedback: Triage outside the hospital is the most effective means of preventing contamination of the facility itself. None of the other listed actions has the potential to prevent the contamination of the hospital itself.

38. A nurse is caring for a patient with gallstones who has been prescribed ursodeoxycholic acid (UDCA). The patient askshow this medicine is going to help his symptoms. The nurse should be aware of what aspect of this drugs pharmacodynamics? A) It inhibits the synthesis of bile. B) It inhibits the synthesis and secretion of cholesterol. C) It inhibits the secretion of bile. D) It inhibits the synthesis and secretion of amylase.

Ans: B Feedback: UDCA acts by inhibiting the synthesis and secretion of cholesterol, thereby desaturating bile. UDCA does not directly inhibit either the synthesis or secretion of bile or amylase.

35. A patient is admitted to the unit with acute cholecystitis. The physician has noted that surgery will be scheduled in 4 days. The patient asks why the surgery is being put off for a week when he has a sick gallbladder. What rationale would underlie the nurses response? A) Surgery is delayed until the patient can eat a regular diet without vomiting. B) Surgery is delayed until the acute symptoms subside. C) The patient requires aggressive nutritional support prior to surgery. D) Time is needed to determine whether a laparoscopic procedure can be used.

Ans: B Feedback: Unless the patients condition deteriorates, surgical intervention is delayed just until the acute symptoms subside (usually within a few days). There is no need to delay surgery pending an improvement in nutritional status, and deciding on a laparoscopic approach is not a lengthy process.

34. The critical care nurse is monitoring the patient's urine output and drains following renal surgery. What should the nurse promptly report to the physician? A) Increased pain on movement B) Absence of drain output C) Increased urine output D) Blood-tinged serosanguineous drain output

Ans: B Feedback: Urine output and drainage from tubes inserted during surgery are monitored for amount, color, and type or characteristics. Decreased or absent drainage is promptly reported to the physician because it may indicate obstruction that could cause pain, infection, and disruption of the suture lines. Reporting increased pain on movement has nothing to do with the scenario described. Increased urine output and serosanguineous drainage are expected.

18. A patient with gallstones has been prescribed ursodeoxycholic acid (UDCA). The nurse understands that additional teaching is needed regarding this medication when the patient states: A) It is important that I see my physician for scheduled follow-up appointments while taking this medication. B) I will take this medication for 2 weeks and then gradually stop taking it. C) If I lose weight, the dose of the medication may need to be changed. D) This medication will help dissolve small gallstones made of cholesterol.

Ans: B Feedback: Ursodeoxycholic acid (UDCA) has been used to dissolve small, radiolucent gallstones composed primarily of cholesterol. This drug can reduce the size of existing stones, dissolve small stones, and prevent new stones from forming. Six to 12 months of therapy is required in many patients to dissolve stones, and monitoring of the patient is required during this time. The effective dose of medication depends on body weight.

6. The nurse caring for a patient receiving a transfusion notes that 15 minutes after the infusion of packed red blood cells (PRBCs) has begun, the patient is having difficulty breathing and complains of severe chest tightness. What is the most appropriate initial action for the nurse to take? A) Notify the patient's physician. B) Stop the transfusion immediately. C) Remove the patient's IV access. D) Assess the patient's chest sounds and vital signs.

Ans: B Feedback: Vascular collapse, bronchospasm, laryngeal edema, shock, fever, chills, and jugular vein distension are severe reactions. The nurse should discontinue the transfusion immediately, monitor the patient's vital signs, and notify the physician. The blood container and tubing should be sent to the blood bank. A blood and urine specimen may be needed if a transfusion reaction or a bacterial infection is suspected. The patient's IV access should not be removed.

26. The nurses assessment of a patient with thyroidectomy suggests tetany and a review of the most recent blood work corroborate this finding. The nurse should prepare to administer what intervention? A) Oral calcium chloride and vitamin D B) IV calcium gluconate C) STAT levothyroxine D) Administration of parathyroid hormone (PTH)

Ans: B Feedback: When hypocalcemia and tetany occur after a thyroidectomy, the immediate treatment is administration of IV calcium gluconate. This has a much faster therapeutic effect than PO calcium or vitamin D supplements. PTH and levothyroxine are not used to treat this complication.

12. A nurse is preparing a plan of care for a patient with pancreatic cysts that have necessitated drainage through the abdominal wall. What nursing diagnosis should the nurse prioritize? A) Disturbed Body Image B) Impaired Skin Integrity C) Nausea D) Risk for Deficient Fluid Volume

Ans: B Feedback: While each of the diagnoses may be applicable to a patient with pancreatic drainage, the priority nursing diagnosis is Impaired Skin Integrity. The drainage is often perfuse and destructive to tissue because of the enzyme contents. Nursing measures must focus on steps to protect the skin near the drainage site from excoriation. The application of ointments or the use of a suction apparatus protects the skin from excoriation.

39. An ED nurse is triaging patients according to the Emergency Severity Index (ESI). When assigning patients to a triage level, the nurse will consider the patients' acuity as well as what other variable? A) The likelihood of a repeat visit to the ED in the next 7 days B) The resources that the patient is likely to require C) The patient's or insurer's ability to pay for care D) Whether the patient is known to ED staff from previous visits

Ans: B Feedback: With the ESI, patients are assigned to triage levels based on both their acuity and their anticipated resource needs. Ability to pay, the likelihood of repeat visits, and the history of prior visits are not explicitly considered.

6. A 37-year-old male patient presents at the emergency department (ED) complaining of nausea and vomiting and severe abdominal pain. The patients abdomen is rigid, and there is bruising to the patients flank. The patients wife states that he was on a drinking binge for the past 2 days. The ED nurse should assist in assessing the patient for what health problem? A) Severe pancreatitis with possible peritonitis B) Acute cholecystitis C) Chronic pancreatitis D) Acute appendicitis with possible perforation

Ans: A Feedback: Severe abdominal pain is the major symptom of pancreatitis that causes the patient to seek medical care. Pain in pancreatitis is accompanied by nausea and vomiting that does not relieve the pain or nausea. Abdominal guarding is present and a rigid or board-like abdomen may be a sign of peritonitis. Ecchymosis (bruising) to the flank or around the umbilicus may indicate severe peritonitis. Pain generally occurs 24 to 48 hours after a heavy meal or alcohol ingestion. The link with alcohol intake makes pancreatitis a more likely possibility than appendicitis or cholecystitis.

7. A triage nurse in the emergency department is assessing a patient who presented with complaints of general malaise. Assessment reveals the presence of jaundice and increased abdominal girth. What assessment question best addresses the possible etiology of this patients presentation? A) How many alcoholic drinks do you typically consume in a week? B) To the best of your knowledge, are your immunizations up to date? C) Have you ever worked in an occupation where you might have been exposed to toxins? D) Has anyone in your family ever experienced symptoms similar to yours?

Ans: A Feedback: Signs or symptoms of hepatic dysfunction indicate a need to assess for alcohol use. Immunization status, occupational risks, and family history are also relevant considerations, but alcohol use is a more common etiologic factor in liver disease.

1. The nurse is caring for a patient diagnosed with hypothyroidism secondary to Hashimotos thyroiditis. When assessing this patient, what sign or symptom would the nurse expect? A) Fatigue B) Bulging eyes C) Palpitations D) Flushed skin

Ans: A Feedback: Symptoms of hypothyroidism include extreme fatigue, hair loss, brittle nails, dry skin, voice huskiness or hoarseness, menstrual disturbance, and numbness and tingling of the fingers. Bulging eyes, palpitations, and flushed skin would be signs and symptoms of hyperthyroidism.

7. A major earthquake has occurred within the vicinity of the local hospital. The nursing supervisor working the night shift at the hospital receives information that the hospital disaster plan will be activated. The supervisor will need to work with what organization responsible for coordinating interagency relief assistance? A) Office of Emergency Management B) Incident Command System C) Centers for Disease Control and Prevention (CDC) D) American Red Cross

Ans: A Feedback: The Office of Emergency Management coordinates the disaster relief efforts at state and local levels. The Incident Command System is a management tool to organize personnel, facilities, equipment, and communication in an emergency situation. The CDC is the agency for disease prevention and control and it supports state and local health departments. The American Red Cross provides additional support.

9. The announcement is made that the facility may return to normal functioning after a local disaster. In the emergency operations plan, what is this referred to as? A) Demobilization response B) Post-incident response C) Crisis diffusion D) Reversion

Ans: A Feedback: The demobilization response occurs when it is deemed that the facility may return to normal daily functioning. This is not known as the post-incident response, crisis diffusion or reversion.

19. A nurse is assisting with serving dinner trays on the unit. Upon receiving the dinner tray for a patient admitted with acute gallbladder inflammation, the nurse will question which of the following foods on the tray? A) Fried chicken B) Mashed potatoes C) Dinner roll D) Tapioca pudding

Ans: A Feedback: The diet immediately after an episode of acute cholecystitis is initially limited to low-fat liquids. Cooked fruits, rice or tapioca, lean meats, mashed potatoes, bread, and coffee or tea may be added as tolerated. The patient should avoid fried foods such as fried chicken, as fatty foods may bring on an episode of cholecystitis.

12. Emergency department (ED) staff members have been trained to follow steps that will decrease the risk of secondary exposure to a chemical. When conducting decontamination, staff members should remove the patient's clothing and then perform what action? A) Rinse the patient with water. B) Wash the patient with a dilute bleach solution. C) Wash the patient chlorhexidine. D) Rinse the patient with hydrogen peroxide.

Ans: A Feedback: The first step in decontamination is removal of the patient's clothing and jewelry and then rinsing the patient with water. This is usually followed by a wash with soap and water, not chlorhexidine, bleach, or hydrogen peroxide.

15. The nurse is planning patient teaching for a patient with ESKD who is scheduled for the creation of a fistula. The nurse would include which of the following in teaching the patient about the fistula? A) A vein and an artery in your arm will be attached surgically. B) The arm should be immobilized for 4 to 6 days. C) One needle will be inserted into the fistula for each dialysis treatment. D) The fistula can be used 2 days after the surgery for dialysis treatment.

Ans: A Feedback: The fistula joins an artery and a vein, either side-to-side or end-to-end. This access will need time, usually 2 to 3 months, to "mature" before it can be used. The patient is encouraged to perform exercises to increase the size of the affected vessels (e.g., squeezing a rubber ball for forearm fistulas). Two needles will be inserted into the fistula for each dialysis treatment.

12. An emergency department nurse learns from the paramedics that they are transporting a patient who has suffered injury from a scald from a hot kettle. What variables will the nurse consider when determining the depth of burn? A) The causative agent B) The patients preinjury health status C) The patients prognosis for recovery D) The circumstances of the accident

Ans: A Feedback: The following factors are considered in determining the depth of a burn: how the injury occurred, causative agent (such as flame or scalding liquid), temperature of the burning agent, duration of contact with the agent, and thickness of the skin. The patients preinjury status, circumstances of the accident, and prognosis for recovery are important, but are not considered when determining the depth of the burn.

36. A patient who is in the acute phase of recovery from a burn injury has yet to experience adequate pain control. What pain management strategy is most likely to meet this patients needs? A) A patient-controlled analgesia (PCA) system B) Oral opioids supplemented by NSAIDs C) Distraction and relaxation techniques supplemented by NSAIDs D) A combination of benzodiazepines and topical anesthetics

Ans: A Feedback: The goal of treatment is to provide a long-acting analgesic that will provide even coverage for this long-term discomfort. It is helpful to use escalating doses when initiating the medication to reach the level of pain control that is acceptable to the patient. The use of patient-controlled analgesia (PCA) gives control to the patient and achieves this goal. Patients cannot normally achieve adequate pain control without the use of opioids, and parenteral administration is usually required.

26. You are the clinic nurse caring for a patient with a recent diagnosis of myasthenia gravis. The patient has begun treatment with pyridostigmine bromide (Mestinon). What change in status would most clearly suggest a therapeutic benefit of this medication? A) Increased muscle strength B) Decreased pain C) Improved GI function D) Improved cognition

Ans: A Feedback: The goal of treatment using pyridostigmine bromide is improvement of muscle strength and control of fatigue. The drug is not intended to treat pain, or cognitive or GI functions.

34. The ED nurse is planning the care of a patient who has been admitted following a sexual assault. The nurse knows that all of the nursing interventions are aimed at what goal? A) Encouraging the patient to gain a sense of control over his or her life B) Collecting sufficient evidence to secure a criminal conviction C) Helping the patient understand that this will not happen again D) Encouraging the patient to verbalize what happened during the assault

Ans: A Feedback: The goals of management are to provide support, to reduce the patient's emotional trauma, and to gather available evidence for possible legal proceedings. All of the interventions are aimed at encouraging the patient to gain a sense of control over his or her life. The patient's well-being should be considered a priority over criminal proceedings. No health professional can guarantee the patient's future safety and having the patient verbalize the event is not a priority.

23. A patient is brought to the emergency department with a burn injury. The nurse knows that the first systemic event after a major burn injury is what? A) Hemodynamic instability B) Gastrointestinal hypermotility C) Respiratory arrest D) Hypokalemia

Ans: A Feedback: The initial systemic event after a major burn injury is hemodynamic instability, which results from loss of capillary integrity and a subsequent shift of fluid, sodium, and protein from the intravascular space into the interstitial spaces. This precedes GI changes. Respiratory arrest may or may not occur, largely depending on the presence or absence of smoke inhalation. Hypokalemia does not take place in the initial phase of recovery.

4. The nurse is working on the renal transplant unit. To reduce the risk of infection in a patient with a transplanted kidney, it is imperative for the nurse to do what? A) Wash hands carefully and frequently. B) Ensure immediate function of the donated kidney. C) Instruct the patient to wear a face mask. D) Bar visitors from the patient's room.

Ans: A Feedback: The nurse ensures that the patient is protected from exposure to infection by hospital staff, visitors, and other patients with active infections. Careful handwashing is imperative; face masks may be worn by hospital staff and visitors to reduce the risk for transmitting infectious agents while the patient is receiving high doses of immunosuppressants. Visitors may be limited, but are not normally barred outright. Ensuring kidney function is vital, but does not prevent infection.

38. The nurse is preparing to administer a unit of platelets to an adult patient. When administering this blood product, which of the following actions should the nurse perform? A) Administer the platelets as rapidly as the patient can tolerate. B) Establish IV access as soon as the platelets arrive from the blood bank. C) Ensure that the patient has a patent central venous catheter. D) Aspirate 10 to 15 mL of blood from the patient's IV immediately following the transfusion.

Ans: A Feedback: The nurse should infuse each unit of platelets as fast as patient can tolerate to diminish platelet clumping during administration. IV access should be established prior to obtaining the platelets from the blood bank. A central line is appropriate for administration, but peripheral IV access (22-gauge or larger) is sufficient. There is no need to aspirate after the transfusion.

32. The nurse providing care for a patient with Cushing syndrome has identified the nursing diagnosis of risk for injury related to weakness. How should the nurse best reduce this risk? A) Establish falls prevention measures. B) Encourage bed rest whenever possible. C) Encourage the use of assistive devices. D) Provide constant supervision.

Ans: A Feedback: The nurse should take action to prevent the patients risk for falls. Bed rest carries too many harmful effects, however, and assistive devices may or may not be necessary. Constant supervision is not normally required or practicable.

9. A nurse is caring for a patient with hepatic encephalopathy. While making the initial shift assessment, the nurse notes that the patient has a flapping tremor of the hands. The nurse should document the presence of what sign of liver disease? A) Asterixis B) Constructional apraxia C) Fetor hepaticus D) Palmar erythema

Ans: A Feedback: The nurse will document that a patient exhibiting a flapping tremor of the hands is demonstrating asterixis. While constructional apraxia is a motor disturbance, it is the inability to reproduce a simple figure. Fetor hepaticus is a sweet, slightly fecal odor to the breath and not associated with a motor disturbance. Skin changes associated with liver dysfunction may include palmar erythema, which is a reddening of the palms, but is not a flapping tremor.

10. You are developing a care plan for a patient with Cushing syndrome. What nursing diagnosis would have the highest priority in this care plan? A) Risk for injury related to weakness B) Ineffective breathing pattern related to muscle weakness C) Risk for loneliness related to disturbed body image D) Autonomic dysreflexia related to neurologic changes

Ans: A Feedback: The nursing priority is to decrease the risk of injury by establishing a protective environment. The patient who is weak may require assistance from the nurse in ambulating to prevent falls or bumping corners or furniture. The patients breathing will not be affected and autonomic dysreflexia is not a plausible risk. Loneliness may or may not be an issue for the patient, but safety is a priority.

13. A nurse takes a shift report and finds he is caring for a patient who has been exposed to anthrax by inhalation. What precautions does the nurse know must be put in place when providing care for this patient? A) Standard precautions B) Airborne precautions C) Droplet precautions D) Contact precautions

Ans: A Feedback: The patient is not contagious, and anthrax cannot be spread from person to person, so standard precautions are initiated. Airborne, contact, and droplet precautions are not necessary.

34. A patient is receiving a blood transfusion and complains of a new onset of slight dyspnea. The nurse's rapid assessment reveals bilateral lung crackles and elevated BP. What is the nurse's most appropriate action? A) Slow the infusion rate and monitor the patient closely. B) Discontinue the transfusion and begin resuscitation. C) Pause the transfusion and administer a 250 mL bolus of normal saline. D) Discontinue the transfusion and administer a beta-blocker, as ordered.

Ans: A Feedback: The patient is showing early signs of hypervolemia; the nurse should slow the infusion rate and assess the patient closely for any signs of exacerbation. At this stage, discontinuing the transfusion is not necessary. A bolus would worsen the patient's fluid overload.

27. The triage nurse is working in the ED. A homeless person is admitted during a blizzard with complaints of being unable to feel his feet and lower legs. Core temperature is noted at 33.2°C (91.8ºF). The patient is intoxicated with alcohol at the time of admission and is visibly malnourished. What is the triage nurse's priority in the care of this patient? A) Addressing the patient's hypothermia B) Addressing the patient's frostbite in his lower extremities C) Addressing the patient's alcohol intoxication D) Addressing the patient's malnutrition

Ans: A Feedback: The patient may also have frostbite, but hypothermia takes precedence in treatment because it is systemic rather than localized. The alcohol abuse and the alteration in nutrition do not take precedence over the treatment of hypothermia because both problems are a less acute threat to the patient's survival.

13. A patient presents at the walk-in clinic complaining of diarrhea and vomiting. The patient has a documented history of adrenal insufficiency. Considering the patients history and current symptoms, the nurse should anticipate that the patient will be instructed to do which of the following? A) Increase his intake of sodium until the GI symptoms improve. B) Increase his intake of potassium until the GI symptoms improve. C) Increase his intake of glucose until the GI symptoms improve. D) Increase his intake of calcium until the GI symptoms improve.

Ans: A Feedback: The patient will need to supplement dietary intake with added salt during episodes of GI losses of fluid through vomiting and diarrhea to prevent the onset of addisonian crisis. While the patient may experience the loss of other electrolytes, the major concern is the replacement of lost sodium.

1. Which patient should the nurse prioritize as needing emergent treatment, assuming no other injuries are present except the ones outlined below? A) A patient with a blunt chest trauma with some difficulty breathing B) A patient with a sore neck who was immobilized in the field on a backboard with a cervical collar C) A patient with a possible fractured tibia with adequate pedal pulses D) A patient with an acute onset of confusion

Ans: A Feedback: The patient with blunt chest trauma possibly has a compromised airway. Establishment and maintenance of a patent airway and adequate ventilation is prioritized over other health problems, including skeletal injuries and changes in cognition.

8. The paramedics bring a patient who has suffered a sexual assault to the ED. What is important for the sexual assault nurse examiner to do when assessing a sexual assault victim? A) Respect the patient's privacy during assessment. B) Shave all pubic hair for laboratory analysis. C) Place items for evidence in plastic bags. D) Bathe the patient before the examination.

Ans: A Feedback: The patient's privacy and sensitivity must be respected, because the patient will be experiencing a stress response to the assault. Pubic hair is combed or trimmed for sampling. Paper bags are used for evidence collection because plastic bags retain moisture, which promotes mold and mildew that can destroy evidence. Bathing the patient before the examination would destroy or remove key evidence.

1. The nurse is assessing a patient suspected of having developed acute glomerulonephritis. The nurse should expect to address what clinical manifestation that is characteristic of this health problem? A) Hematuria B) Precipitous decrease in serum creatinine levels C) Hypotension unresolved by fluid administration D) Glucosuria

Ans: A Feedback: The primary presenting feature of acute glomerulonephritis is hematuria (blood in the urine), which may be microscopic (identifiable through microscopic examination) or macroscopic or gross (visible to the eye). Proteinuria, primarily albumin, which is present, is due to increased permeability of the glomerular membrane. Blood urea nitrogen (BUN) and serum creatinine levels may rise as urine output drops. Some degree of edema and hypertension is noted in most patients.

33. A patient asks the nurse why kidney problems can cause gastrointestinal disturbances. What relationship should the nurse describe? A) The right kidney's proximity to the pancreas, liver, and gallbladder B) The indirect impact of digestive enzymes on renal function C) That the peritoneum encapsulates the GI system and the kidneys D) The left kidney's connection to the common bile duct

Ans: A Feedback: The proximity of the right kidney to the colon, duodenum, head of the pancreas, common bile duct, liver, and gallbladder may cause GI disturbances. The proximity of the left kidney to the colon (splenic flexure), stomach, pancreas, and spleen may also result in intestinal symptoms. Digestive enzymes do not affect renal function and the left kidney is not connected to the common bile duct.

34. The nurse is providing health education to a patient who has a C6 spinal cord injury. The patient asks why autonomic dysreflexia is considered an emergency. What would be the nurses best answer? A) The sudden increase in BP can raise the ICP or rupture a cerebral blood vessel. B) The suddenness of the onset of the syndrome tells us the body is struggling to maintain its normal state. C) Autonomic dysreflexia causes permanent damage to delicate nerve fibers that are healing. D) The sudden, severe headache increases muscle tone and can cause further nerve damage.

Ans: A Feedback: The sudden increase in BP may cause a rupture of one or more cerebral blood vessels or lead to increased ICP. Autonomic dysreflexia does not directly cause nerve damage.

2. A nurse knows that specific areas in the ureters have a propensity for obstruction. Prompt management of renal calculi is most important when the stone is located where? A) In the ureteropelvic junction B) In the ureteral segment near the sacroiliac junction C) In the ureterovesical junction D) In the urethra

Ans: A Feedback: The three narrowed areas of each ureter are the ureteropelvic junction, the ureteral segment near the sacroiliac junction, and the ureterovescial junction. These three areas of the ureters have a propensity for obstruction by renal calculi or stricture. Obstruction of the ureteropelvic junction is most serious because of its close proximity to the kidney and the risk of associated kidney dysfunction. The urethra is not part of the ureter.

31. A patient is undergoing testing for suspected adrenocortical insufficiency. The care team should ensure that the patient has been assessed for the most common cause of adrenocortical insufficiency. What is the most common cause of this health problem? A) Therapeutic use of corticosteroids B) Pheochromocytoma C) Inadequate secretion of ACTH D) Adrenal tumor

Ans: A Feedback: Therapeutic use of corticosteroids is the most common cause of adrenocortical insufficiency. The other options also cause adrenocortical insufficiency, but they are not the most common causes.

30. The nurse is caring for a patient admitted with a drug overdose. What is the nurse's priority responsibility in caring for this patient? A) Support the patient's respiratory and cardiovascular function. B) Provide for the safety of the patient. C) Enhance clearance of the offending agent. D) Ensure the safety of the staff.

Ans: A Feedback: Treatment goals for a patient with a drug overdose are to support the respiratory and cardiovascular functions, to enhance clearance of the agent, and to provide for safety of the patient and staff. Of these responsibilities, however, support of vital physiologic function is a priority.

24. An 82-year-old man is admitted for observation after a fall. Due to his age, the nurse knows that the patient is at increased risk for what complication of his injury? A) Hematoma B) Skull fracture C) Embolus D) Stroke

Ans: A Feedback: Two major factors place older adults at increased risk for hematomas. First, the dura becomes more adherent to the skull with increasing age. Second, many older adults take aspirin and anticoagulants as part of routine management of chronic conditions. Because of these factors, the patients risk of a hematoma is likely greater than that of stroke, embolism, or skull fracture.

18. A patient with elevated BUN and creatinine values has been referred by her primary physician for further evaluation. The nurse should anticipate the use of what initial diagnostic test? A) Ultrasound B) X-ray C) Computed tomography (CT) D) Nuclear scan

Ans: A Feedback: Ultrasonography is a noninvasive procedure that passes sound waves into the body through a transducer to detect abnormalities of internal tissues and organs. Structures of the urinary system create characteristic ultrasonographic images. Because of its sensitivity, ultrasonography has replaced many other diagnostic tests as the initial diagnostic procedure.

5. The nurse is providing pre-procedure teaching about an ultrasound. The nurse informs the patient that in preparation for an ultrasound of the lower urinary tract the patient will require what? A) Increased fluid intake to produce a full bladder B) IV administration of radiopaque contrast agent C) Sedation and intubation D) Injection of a radioisotope

Ans: A Feedback: Ultrasonography requires a full bladder; therefore, fluid intake should be encouraged before the procedures. The administration of a radiopaque contrast agent is required to perform IV urography studies, such as an IV pyelogram. Ultrasonography is a quick and painless diagnostic test and does not require sedation or intubation. The injection of a radioisotope is required for nuclear scan and ultrasonography is not in this category of diagnostic studies.

39. A 55-year-old female patient with hepatocellular carcinoma (HCC) is undergoing radiofrequency ablation. The nurse should recognize what goal of this treatment? A) Destruction of the patients liver tumor B) Restoration of portal vein patency C) Destruction of a liver abscess D) Reversal of metastasis

Ans: A Feedback: Using radiofrequency ablation, a tumor up to 5 cm in size can be destroyed in one treatment session. This technique does not address circulatory function or abscess formation. It does not allow for the reversal of metastasis.

22. A nurse has reported for a shift at a busy burns and plastics unit in a large university hospital. Which patient is most likely to have life-threatening complications? A) A 4-year-old scald victim burned over 24% of the body B) A 27-year-old male burned over 36% of his body in a car accident C) A 39-year-old female patient burned over 18% of her body D) A 60-year-old male burned over 16% of his body in a brush fire

Ans: A Feedback: Young children and the elderly continue to have increased morbidity and mortality when compared to other age groups with similar injuries and present a challenge for burn care. This is an important factor when determining the severity of injury and possible outcome for the patient.

4. A nurse educator is teaching a group of recent nursing graduates about their occupational risks for contracting hepatitis B. What preventative measures should the educator promote? Select all that apply. A) Immunization B) Use of standard precautions C) Consumption of a vitamin-rich diet D) Annual vitamin K injections E) Annual vitamin B12 injections

Ans: A, B Feedback: People who are at high risk, including nurses and other health care personnel exposed to blood or blood products, should receive active immunization. The consistent use of standard precautions is also highly beneficial. Vitamin supplementation is unrelated to an individuals risk of HBV.

38. A patient with spinal cord injury is ready to be discharged home. A family member asks the nurse to review potential complications one more time. What are the potential complications that should be monitored for in this patient? Select all that apply. A) Orthostatic hypotension B) Autonomic dysreflexia C) DVT D) Salt-wasting syndrome E) Increased ICP

Ans: A, B, C Feedback: For a spinal cord-injured patient, based on the assessment data, potential complications that may develop include DVT, orthostatic hypotension, and autonomic dysreflexia. Salt-wasting syndrome or increased ICP are not typical complications following the immediate recovery period.

37. The nurse is caring for a patient who has just returned to the post-surgical unit following renal surgery. When assessing the patient's output from surgical drains, the nurse should assess what parameters? Select all that apply. A) Quantity of output B) Color of the output C) Visible characteristics of the output D) Odor of the output E) pH of the output

Ans: A, B, C Feedback: Urine output and drainage from tubes inserted during surgery are monitored for amount, color, and type or characteristics. Odor and pH are not normally assessed.

What types of food should an ESKD patient receiving dialysis restrict from their diet? (Select all that apply) A. Protein B. Sodium rich foods C. Phosphorus enriched foods D. Food containing potassium

Ans: A, B, C, D

23. A patient with pancreatic cancer has been scheduled for a pancreaticoduodenectomy (Whipple procedure). During health education, the patient should be informed that this procedure will involve the removal of which of the following? Select all that apply. A) Gallbladder B) Part of the stomach C) Duodenum D) Part of the common bile duct E) Part of the rectum

Ans: A, B, C, D Feedback: A pancreaticoduodenectomy (Whipple procedure or resection) is used for potentially resectable cancer of the head of the pancreas (Fig. 50-7). This procedure involves removal of the gallbladder, a portion of the stomach, duodenum, proximal jejunum, head of the pancreas, and distal common bile duct. The rectum is not affected.

39. A nurse on the renal unit is caring for a patient who will soon begin peritoneal dialysis. The family of the patient asks for education about the peritoneal dialysis catheter that has been placed in the patient's peritoneum. The nurse explains the three sections of the catheter and talks about the two cuffs on the dialysis catheter. What would the nurse explain about the cuffs? Select all that apply. A) The cuffs are made of Dacron polyester. B) The cuffs stabilize the catheter. C) The cuffs prevent the dialysate from leaking. D) The cuffs provide a barrier against microorganisms. E) The cuffs absorb dialysate

Ans: A, B, C, D Feedback: Most of these catheters have two cuffs, which are made of Dacron polyester. The cuffs stabilize the catheter, limit movement, prevent leaks, and provide a barrier against microorganisms. They do not absorb dialysate.

30. The nurse is caring for a patient status after a motor vehicle accident. The patient has developed AKI. What is the nurse's role in caring for this patient? Select all that apply. A) Providing emotional support for the family B) Monitoring for complications C) Participating in emergency treatment of fluid and electrolyte imbalances D) Providing nursing care for primary disorder (trauma) E) Directing nutritional interventions

Ans: A, B, C, D Feedback: The nurse has an important role in caring for the patient with AKI. The nurse monitors for complications, participates in emergency treatment of fluid and electrolyte imbalances, assesses the patient's progress and response to treatment, and provides physical and emotional support. Additionally, the nurse keeps family members informed about the patient's condition, helps them understand the treatments, and provides psychological support. Although the development of AKI may be the most serious problem, the nurse continues to provide nursing care indicated for the primary disorder (e.g., burns, shock, trauma, obstruction of the urinary tract). The nurse does not direct the patient's nutritional status; the dietician and the physician normally collaborate on directing the patient's nutritional status.

38. The nurse is preparing to collect an ordered urine sample for urinalysis. The nurse should be aware that this test will include what assessment parameters? Select all that apply. A) Specific gravity of the patient's urine B) Testing for the presence of glucose in the patient's urine C) Microscopic examination of urine sediment for RBCs D) Microscopic examination of urine sediment for casts E) Testing for BUN and creatinine in the patient's urine

Ans: A, B, C, D Feedback: Urine testing includes testing for specific gravity, glucose, RBCs, and casts. BUN and creatinine are components of serum, not urine.

The nurse has a new admission of a patient diagnosed with hypoparathyroidism. What interventions will not be included in the care plan? (Select all that apply) A. Calcium level to 15-20 mg/dL B.Monitor potassium levels C. Bronchodilators D. Diet high in milk products

Ans: A, B, D

What are the clinical manifestations of Leukemia? (Select all that apply) A. Fever B. Visual disturbances C. Weight gain D. Bone or joint pain

Ans: A, B, D

What is included in the fluid volume management for a patient with CKD? (Select all that apply) A. Drug therapy B. Nutrition therapy C. Hydration D. Dialysis

Ans: A, B, D

16. A patient with ESKD is scheduled to begin hemodialysis. The nurse is working with the patient to adapt the patient's diet to maximize the therapeutic effect and minimize the risks of complications. The patient's diet should include which of the following modifications? Select all that apply. A) Decreased protein intake B) Decreased sodium intake C) Increased potassium intake D) Fluid restriction E) Vitamin D supplementation

Ans: A, B, D Feedback: Restricting dietary protein decreases the accumulation of nitrogenous wastes, reduces uremic symptoms, and may even postpone the initiation of dialysis for a few months. Restriction of fluid is also part of the dietary prescription because fluid accumulation may occur. As well, sodium is usually restricted to 2 to 3 g/day. Potassium intake is usually limited, not increased, and there is no particular need for vitamin D supplementation.

3. A patients assessment and diagnostic testing are suggestive of acute pancreatitis. When the nurse is performing the health interview, what assessment questions address likely etiologic factors? Select all that apply. A) How many alcoholic drinks do you typically consume in a week? B) Have you ever been tested for diabetes? C) Have you ever been diagnosed with gallstones? D) Would you say that you eat a particularly high-fat diet? E) Does anyone in your family have cystic fibrosis?

Ans: A, C Feedback: Eighty percent of patients with acute pancreatitis have biliary tract disease such as gallstones or a history of long-term alcohol abuse. Diabetes, high-fat consumption, and cystic fibrosis are not noted etiologic factors.

25. The nurse is caring for a 77-year-old woman with MS. She states that she is very concerned about the progress of her disease and what the future holds. The nurse should know that elderly patients with MS are known to be particularly concerned about what variables? Select all that apply. A) Possible nursing home placement B) Pain associated with physical therapy C) Increasing disability D) Becoming a burden on the family E) Loss of appetite

Ans: A, C, D Feedback: Elderly patients with MS are particularly concerned about increasing disability, family burden, marital concern, and the possible future need for nursing home care. Older adults with MS are not noted to have particular concerns regarding the pain of therapy or loss of appetite.

33. A patient with Cushing syndrome has been hospitalized after a fall. The dietician consulted works with the patient to improve the patients nutritional intake. What foods should a patient with Cushing syndrome eat to optimize health? Select all that apply. A) Foods high in vitamin D B) Foods high in calories C) Foods high in protein D) Foods high in calcium E) Foods high in sodium

Ans: A, C, D Feedback: Foods high in vitamin D, protein, and calcium are recommended to minimize muscle wasting and osteoporosis. Referral to a dietitian may assist the patient in selecting appropriate foods that are also low in sodium and calories.

21. A patients physician has ordered a liver panel in response to the patients development of jaundice. When reviewing the results of this laboratory testing, the nurse should expect to review what blood tests? Select all that apply. A) Alanine aminotransferase (ALT) B) C-reactive protein (CRP) C) Gamma-glutamyl transferase (GGT) D) Aspartate aminotransferase (AST) E) B-type natriuretic peptide (BNP)

Ans: A, C, D Feedback: Liver function testing includes GGT, ALT, and AST. CRP addresses the presence of generalized inflammation and BNP is relevant to heart failure; neither is included in a liver panel.

33. A patient with end-stage liver disease has developed hypervolemia. What nursing interventions would be most appropriate when addressing the patients fluid volume excess? Select all that apply. A) Administering diuretics B) Administering calcium channel blockers C) Implementing fluid restrictions D) Implementing a 1500 kcal/day restriction E) Enhancing patient positioning

Ans: A, C, E Feedback: Administering diuretics, implementing fluid restrictions, and enhancing patient positioning can optimize the management of fluid volume excess. Calcium channel blockers and calorie restriction do not address this problem.

33. A patient is in the acute phase of a burn injury. One of the nursing diagnoses in the plan of care is Ineffective Coping Related to Trauma of Burn Injury. What interventions appropriately address this diagnosis? Select all that apply. A) Promote truthful communication. B) Avoid asking the patient to make decisions. C) Teach the patient coping strategies. D) Administer benzodiazepines as ordered. E) Provide positive reinforcement.

Ans: A, C, E Feedback: The nurse can assist the patient to develop effective coping strategies by setting specific expectations for behavior, promoting truthful communication to build trust, helping the patient practice appropriate strategies, and giving positive reinforcement when appropriate. The patient may benefit from being able to make decisions regarding his or her care. Benzodiazepines may be needed for short-term management of anxiety, but they are not used to enhance coping.

29. A patient admitted to the ED with severe diarrhea and vomiting is subsequently diagnosed with food poisoning. The nurse caring for this patient assesses for signs and symptoms of fluid and electrolyte imbalances. For what signs and symptoms would this nurse assess? Select all that apply. A) Dysrhythmias B) Hypothermia C) Hypotension D) Hyperglycemia E) Delirium

Ans: A, C, E Feedback: The patient is assessed for signs and symptoms of fluid and electrolyte imbalances, including lethargy, rapid pulse rate, fever, oliguria, anuria, hypotension, and delirium. Hyperglycemia and hypothermia are not typically associated with fluid and electrolyte imbalances.

11. The nurse is performing a shift assessment of a patient with aldosteronism. What assessments should the nurse include? Select all that apply. A) Urine output B) Signs or symptoms of venous thromboembolism C) Peripheral pulses D) Blood pressure E) Skin integrity

Ans: A, D Feedback: The principal action of aldosterone is to conserve body sodium. Alterations in aldosterone levels consequently affect urine output and BP. The patients peripheral pulses, risk of VTE, and skin integrity are not typically affected by aldosteronism.

9. Through the process of hematopoiesis, stem cells differentiate into either myeloid or lymphoid stem cells. Into what do myeloid stem cells further differentiate? Select all that apply. A) Leukocytes B) Natural killer cells C) Cytokines D) Platelets E) Erythrocytes

Ans: A, D, E Feedback: Myeloid stem cells differentiate into three broad cell types: erythrocytes, leukocytes, and platelets. Natural killer cells and cytokines do not originate as myeloid stem cells.

28. The school nurse is giving a presentation on preventing spinal cord injuries (SCI). What should the nurse identify as prominent risk factors for SCI? Select all that apply. A) Young age B) Frequent travel C) African American race D) Male gender E) Alcohol or drug use

Ans: A, D, E Feedback: The predominant risk factors for SCI include young age, male gender, and alcohol and drug use. Ethnicity and travel are not risk factors.

Signs and symptoms of circulatory overload include all except: A. Bradycardia B. Tightness of the chest C. Pulmonary edema D. Elevated blood pressure

Ans: A. Bradycardia

What laboratory data is the most accurate indicator that a client with acute kidney injury has met the expected outcomes? A. Decreased serum creatinine B. Decreased neutrophil count C. Decreased lymphocyte count D. Decreased blood urea nitrogen levels (BUN)

Ans: A. Decreased serum creatinine

What intervention is not included in the care plan for a patient following a thyroidectomy? A. Hemorrhage is most likely to occur past 24 hours after surgery B. Laryngeal stridor C. Place a patient in a semi-fowler's position- not the answer D. Respiratory distress

Ans: A. Hemorrhage is most likely to occur past 24 hours after surgery.

What is not an extrinsic factor for the increased destruction of RBC's (hemolysis)? A. Membrane abnormalities B. Antibiotics C. Physical trauma D. Antibodies

Ans: A. Membrane abnormalities Rationale: Membrane abnormalities are an intrinsic factor.

What is the priority nursing intervention in the care of an older patient with a history of diverticular disease and pernicious anemia? A. Preventing falls B. Monitoring intake and output C. Turning the patient every 2 hours D. Encouraging a diet high in Vitamin B12

Ans: A. Preventing falls

What is the name of a carpopedal spasm induced when blood flow to the arm is occluded using a blood pressure cuff or tourniquet, causing ischemia to the distal nerves; suggestive sign of hypocalcemia? A. Trousseau's sign B. Chvostek's sign C. Hypophysectomy D. Blood flow spasm test

Ans: A. Trousseau's sign

Complications of chronic pyelonephritis include end-stage kidney disease. A. True B. False

Ans: A. True

Management of Hepatitis A usually occurs in the home A. True B. False

Ans: A. True

Myxedema is an accumulation of mucopolysoradies in the subq and interstitial tissues. A. True B. False

Ans: A. True

Risk for hepatitis B includes hemodialysis treatment. A. True B. False

Ans: A. True

The patient with chronic pyelonephritis usually has no symptoms of infection unless an acute exacerbation occurs. A. True B. False

Ans: A. True

Manifestations of hypothyroidism include all of the below, except? A. Weight loss B. Amenorrhea C. Decreased heart rate D. Cognitive changes

Ans: A. Weight loss Rationale: A symptom of hypothyroidism is weight gain, not weight loss.

Which type of leukemia is a result of damage to immature cells? A. Chronic lymphocytic leukemia B. Acute myeloid leukemia C. Chronic myeloid leukemia D. Acute lymphadenopathy leukemia

Ans: According to Garriga, it's A. I understood it as D. She gave us the point but idk what the true correct one was

Emergency care of the patient during myxedema coma includes? (Select all that apply.) A. Cover the patient with warm blankets B. Turn every two hours C. Institute Aspiration Precautions D. Check the patient's temperature hourly.

Ans: All of the above

13. A 6-year-old is admitted to the ED after being rescued from a pond after falling through the ice while ice skating. What action should the nurse perform while rewarming the patient? A) Assessing the patient's oral temperature frequently B) Ensuring continuous ECG monitoring C) Massaging the patient's skin surfaces to promote circulation D) Administering bronchodilators by nebulizer

Ans: B Feedback: A hypothermic patient requires continuous ECG monitoring and assessment of core temperatures with an esophageal probe, bladder, or rectal thermometer. Massage is not performed and bronchodilators would normally be insufficient to meet the patient's respiratory needs.

17. A patient on the critical care unit is postoperative day 1 following kidney transplantation from a living donor. The nurse's most recent assessments indicate that the patient is producing copious quantities of dilute urine. What is the nurse's most appropriate response? A) Assess the patient for further signs or symptoms of rejection. B) Recognize this as an expected finding. C) Inform the primary care provider of this finding. D) Administer exogenous antidiuretic hormone as ordered.

Ans: B Feedback: A kidney from a living donor related to the patient usually begins to function immediately after surgery and may produce large quantities of dilute urine. This is not suggestive of rejection and treatment is not warranted. There is no obvious need to report this finding.

17. The nurse is planning the care of a patient with hyperthyroidism. What should the nurse specify in the patients meal plan? A) A clear liquid diet, high in nutrients B) Small, frequent meals, high in protein and calories C) Three large, bland meals a day D) A diet high in fiber and plant-sourced fat

Ans: B Feedback: A patient with hyperthyroidism has an increased appetite. The patient should be counseled to consume several small, well-balanced meals. High-calorie, high-protein foods are encouraged. A clear liquid diet would not satisfy the patients caloric or hunger needs. A diet rich in fiber and fat should be avoided because these foods may lead to GI upset or increase peristalsis.

40. A patient's electronic health record states that the patient receives regular transfusions of factor IX. The nurse would be justified in suspecting that this patient has what diagnosis? A) Leukemia B) Hemophilia C) Hypoproliferative anemia D) Hodgkin's lymphoma

Ans: B Feedback: Administration of clotting factors is used to treat diseases where these factors are absent or insufficient; hemophilia is among the most common of these diseases. Factor IX is not used in the treatment of leukemia, lymphoma, or anemia.

32. A patient is being treated for bites that she suffered during an assault. After the bites have been examined and documented by a forensic examiner, the nurse should perform what action? A) Apply a dressing saturated with chlorhexidine. B) Wash the bites with soap and water. C) Arrange for the patient to receive a hepatitis B vaccination. D) Assess the patient's immunization history.

Ans: B Feedback: After forensic evidence has been gathered, cleansing with soap and water is necessary, followed by the administration of antibiotics and tetanus toxoid as prescribed. The patient's immunization history does not directly influence the course of treatment and hepatitis B vaccination is not indicated. Chlorhexidine bandages are not recommended.

25. A patient is scheduled for a splenectomy. During discharge education, what teaching point should the nurse prioritize? A) The importance of adhering to prescribed immunosuppressant therapy B) The need to report any signs or symptoms of infection promptly C) The need to ensure adequate folic acid, iron, and vitamin B12 intake D) The importance of limiting activity postoperatively to prevent hemorrhage

Ans: B Feedback: After splenectomy, the patient is instructed to seek prompt medical attention if even relatively minor symptoms of infection occur. Often, patients with high platelet counts have even higher counts after splenectomy, which can predispose them to serious thrombotic or hemorrhagic problems. However, this increase is usually transient and therefore often does not warrant additional treatment. Dietary modifications are not normally necessary and immunosuppressants would be strongly contraindicated.

26. A patient with a diagnosis of esophageal varices has undergone endoscopy to gauge the progression of this complication of liver disease. Following the completion of this diagnostic test, what nursing intervention should the nurse perform? A) Keep patient NPO until the results of test are known. B) Keep patient NPO until the patients gag reflex returns. C) Administer analgesia until post-procedure tenderness is relieved. D) Give the patient a cold beverage to promote swallowing ability.

Ans: B Feedback: After the examination, fluids are not given until the patients gag reflex returns. Lozenges and gargles may be used to relieve throat discomfort if the patients physical condition and mental status permit. The result of the test is known immediately. Food and fluids are contraindicated until the gag reflex returns.

11. A participant in a health fair has asked the nurse about the role of drugs in liver disease. What health promotion teaching has the most potential to prevent drug-induced hepatitis? A) Finish all prescribed courses of antibiotics, regardless of symptom resolution. B) Adhere to dosing recommendations of OTC analgesics. C) Ensure that expired medications are disposed of safely. D) Ensure that pharmacists regularly review drug regimens for potential interactions.

Ans: B Feedback: Although any medication can affect liver function, use of acetaminophen (found in many over-the-counter medications used to treat fever and pain) has been identified as the leading cause of acute liver failure. Finishing prescribed antibiotics and avoiding expired medications are unrelated to this disease. Drug interactions are rarely the cause of drug-induced hepatitis.

34. A patient with a history of progressively worsening fatigue is undergoing a comprehensive assessment which includes test of renal function relating to erythropoiesis. When assessing the oxygen transport ability of the blood, the nurse should prioritize the review of what blood value? A) Hematocrit B) Hemoglobin C) Erythrocyte sedimentation rate (ESR) D) Serum creatinine

Ans: B Feedback: Although historically hematocrit has been the blood test of choice when assessing a patient for anemia, use of the hemoglobin level rather than hematocrit is currently recommended, because that measurement is a better assessment of the oxygen transport ability of the blood. ESR and creatinine levels are not indicative of oxygen transport ability.

31. A 71-year-old patient with ESKD has been told by the physician that it is time to consider hemodialysis until a transplant can be found. The patient tells the nurse she is not sure she wants to undergo a kidney transplant. What would be an appropriate response for the nurse to make? A) "The decision is certainly yours to make, but be sure not to make a mistake." B) "Kidney transplants in patients your age are as successful as they are in younger patients." C) "I understand your hesitancy to commit to a transplant surgery. Success is comparatively rare." D) "Have you talked this over with your family?"

Ans: B Feedback: Although there is no specific age limitation for renal transplantation, concomitant disorders (e.g., coronary artery disease, peripheral vascular disease) have made it a less common treatment for the elderly. However, the outcome is comparable to that of younger patients. The other listed options either belittle the patient or give the patient misinformation.

32. A patient is being treated in the ICU after a medical error resulted in an acute hemolytic transfusion reaction. What was the etiology of this patient's adverse reaction? A) Antibodies to donor leukocytes remained in the blood. B) The donor blood was incompatible with that of the patient. C) The patient had a sensitivity reaction to a plasma protein in the blood. D) The blood was infused too quickly and overwhelmed the patient's circulatory system.

Ans: B Feedback: An acute hemolytic reaction occurs when the donor blood is incompatible with that of the recipient. In the case of a febrile nonhemolytic reaction, antibodies to donor leukocytes remain in the unit of blood or blood component. An allergic reaction is a sensitivity reaction to a plasma protein within the blood component. Hypervolemia does not cause an acute hemolytic reaction.

12. The nurse caring for a patient with suspected renal dysfunction calculates that the patient's weight has increased by 5 pounds in the past 24 hours. The nurse estimates that the patient has retained approximately how much fluid? A) 1,300 mL of fluid in 24 hours B) 2,300 mL of fluid in 24 hours C) 3,100 mL of fluid in 24 hours D) 5,000 mL of fluid in 24 hours

Ans: B Feedback: An increase in body weight commonly accompanies edema. To calculate the approximate weight gain from fluid retention, remember that 1 kg of weight gain equals approximately 1,000 mL of fluid. Five lbs = 2.27 kg = 2,270 mL.

20. A patients burns have required a homograft. During the nurses most recent assessment, the nurse observes that the graft is newly covered with purulent exudate. What is the nurses most appropriate response? A) Perform mechanical dbridement to remove the exudate and prevent further infection. B) Inform the primary care provider promptly because the graft may need to be removed. C) Perform range of motion exercises to increase perfusion to the graft site and facilitate healing. D) Document this finding as an expected phase of graft healing.

Ans: B Feedback: An infected graft may need to be removed, thus the care provider should be promptly informed. ROM exercises will not resolve this problem and the nurse would not independently perform debridement.

16. An elderly woman found with a head injury on the floor of her home is subsequently admitted to the neurologic ICU. What is the best rationale for the following physician orders: elevate the HOB; keep the head in neutral alignment with no neck flexion or head rotation; avoid sharp hip flexion? A) To decrease cerebral arterial pressure B) To avoid impeding venous outflow C) To prevent flexion contractures D) To prevent aspiration of stomach contents

Ans: B Feedback: Any activity or position that impedes venous outflow from the head may contribute to increased volume inside the skull and possibly increase ICP. Cerebral arterial pressure will be affected by the balance between oxygen and carbon dioxide. Flexion contractures are not a priority at this time. Stomach contents could still be aspirated in this position.

34. A patient diagnosed with MS has been admitted to the medical unit for treatment of an MS exacerbation. Included in the admission orders is baclofen (Lioresal). What should the nurse identify as an expected outcome of this treatment? A) Reduction in the appearance of new lesions on the MRI B) Decreased muscle spasms in the lower extremities C) Increased muscle strength in the upper extremities D) Decreased severity and duration of exacerbations

Ans: B Feedback: Baclofen, a g-aminobutyric acid (GABA) agonist, is the medication of choice in treating spasms. It can be administered orally or by intrathecal injection. Avonex and Betaseron reduce the appearance of new lesions on the MRI. Corticosteroids limit the severity and duration of exacerbations. Anticholinesterase agents increase muscle strength in the upper extremities.

5. The nurse is caring for a patient with Addisons disease who is scheduled for discharge. When teaching the patient about hormone replacement therapy, the nurse should address what topic? A) The possibility of precipitous weight gain B) The need for lifelong steroid replacement C) The need to match the daily steroid dose to immediate symptoms D) The importance of monitoring liver function

Ans: B Feedback: Because of the need for lifelong replacement of adrenal cortex hormones to prevent addisonian crises, the patient and family members receive explicit education about the rationale for replacement therapy and proper dosage. Doses are not adjusted on a short-term basis. Weight gain and hepatotoxicity are not common adverse effects.

30. A patient on the medical unit is receiving a unit of PRBCs. Difficult IV access has necessitated a slow infusion rate and the nurse notes that the infusion began 4 hours ago. What is the nurse's most appropriate action? A) Apply an icepack to the blood that remains to be infused. B) Discontinue the remainder of the PRBC transfusion and inform the physician. C) Disconnect the bag of PRBCs, cool for 30 minutes and then administer. D) Administer the remaining PRBCs by the IV direct (IV push) route.

Ans: B Feedback: Because of the risk of infection, a PRBC transfusion should not exceed 4 hours. Remaining blood should not be transfused, even if it is cooled. Blood is not administered by the IV direct route.

16. A patient is being treated for the effects of a longstanding vitamin B12 deficiency. What aspect of the patient's health history would most likely predispose her to this deficiency? A) The patient has irregular menstrual periods. B) The patient is a vegan. C) The patient donated blood 60 days ago. D) The patient frequently smokes marijuana.

Ans: B Feedback: Because vitamin B12 is found only in foods of animal origin, strict vegetarians may ingest little vitamin B12. Irregular menstrual periods, marijuana use, and blood donation would not precipitate a vitamin B12 deficiency.

24. The nurse is caring for a patient scheduled for renal angiography following a motor vehicle accident. What patient preparation should the nurse most likely provide before this test? A) Administration of IV potassium chloride B) Administration of a laxative C) Administration of Gastrografin D) Administration of a 24-hour urine test

Ans: B Feedback: Before the procedure, a laxative may be prescribed to evacuate the colon so that unobstructed x-rays can be obtained. A 24-hour urine test is not necessary prior to the procedure. Gastrografin and potassium chloride are not administered prior to renal angiography.

15. A 13-year-old is being admitted to the ED after falling from a roof and sustaining blunt abdominal injuries. To assess for internal injury in the patient's peritoneum, the nurse should anticipate what diagnostic test? A) Radiograph B) Computed tomography (CT) scan C) Complete blood count (CBC) D) Barium swallow

Ans: B Feedback: CT scan of the abdomen, diagnostic peritoneal lavage, and abdominal ultrasound are appropriate diagnostic tools to assess intra-abdominal injuries. X-rays do not yield sufficient data and a CBC would not reveal the presence of intraperitoneal injury.

38. A patient with MS has developed dysphagia as a result of cranial nerve dysfunction. What nursing action should the nurse consequently perform? A) Arrange for the patient to receive a low residue diet. B) Position the patient upright during feeding. C) Suction the patient following each meal. D) Withhold liquids until the patient has finished eating.

Ans: B Feedback: Correct, upright positioning is necessary to prevent aspiration in the patient with dysphagia. There is no need for a low-residue diet and suctioning should not be performed unless there is an apparent need. Liquids do not need to be withheld during meals in order to prevent aspiration.

19. A 44-year-old male patient has been exposed to severe amount of radiation after a leak in a reactor plant. When planning this patient's care, the nurse should implement what action? A) The patient should be scrubbed with alcohol and iodine. B) The patient should be carefully protected from infection. C) The patient's immunization status should be promptly assessed. D) The patient's body hair should be removed to prevent secondary contamination.

Ans: B Feedback: Damage to the hematopoietic system following radiation exposure creates a serious risk for infection. There is no need to remove the patient's hair and the patient's immunization status is not significant. Alcohol and iodine are ineffective against radiation.

21. The nurse is providing care for an older adult who has a hematologic disorder. What age-related change in hematologic function should the nurse integrate into care planning? A) Bone marrow in older adults produces a smaller proportion of healthy, functional blood cells. B) Older adults are less able to increase blood cell production when demand suddenly increases. C) Stem cells in older adults eventually lose their ability to differentiate. D) The ratio of plasma to erythrocytes and lymphocytes increases with age.

Ans: B Feedback: Due to a variety of factors, when an older person needs more blood cells, the bone marrow may not be able to increase production of these cells adequately. Stem cell activity continues throughout the lifespan, although at a somewhat decreased rate. The proportion of functional cells does not greatly decrease and the relative volume of plasma does not change significantly.

20. The nurse is coordinating the care of victims who arrive at the ED after a radiation leak at a nearby nuclear plant. What would be the first intervention initiated when victims arrive at the hospital? A) Administer prophylactic antibiotics. B) Survey the victims using a radiation survey meter. C) Irrigate victims' open wounds. D) Perform soap and water decontamination.

Ans: B Feedback: Each patient arriving at the hospital should first be surveyed with the radiation survey meter for external contamination and then directed toward the decontamination area as needed. This survey should precede decontamination efforts or irrigation of wounds. Antibiotics are not indicated.

24. A nurse is caring for a patient who has been admitted for the treatment of advanced cirrhosis. What assessment should the nurse prioritize in this patients plan of care? A) Measurement of abdominal girth and body weight B) Assessment for variceal bleeding C) Assessment for signs and symptoms of jaundice D) Monitoring of results of liver function testing

Ans: B Feedback: Esophageal varices are a major cause of mortality in patients with uncompensated cirrhosis. Consequently, this should be a focus of the nurses assessments and should be prioritized over the other listed assessments, even though each should be performed.

9. A patient with a history of major depression is brought to the ED by her parents. Which of the following nursing actions is most appropriate? A) Noting that symptoms of physical illness are not relevant to the current diagnosis B) Asking the patient if she has ever thought about taking her own life C) Conducting interviews in a brief and direct manner D) Arranging for the patient to spend time alone to consider her feelings

Ans: B Feedback: Establishing if the patient has suicidal thoughts or intents helps identify the level of depression and intervention. Physical symptoms are relevant and should be explored. Allow the patient to express feelings, and conduct the interview at a comfortable pace for the patient. Never leave the patient alone, because suicide is usually committed in solitude.

3. The nurse is caring for a patient with multiple sclerosis (MS). The patient tells the nurse the hardest thing to deal with is the fatigue. When teaching the patient how to reduce fatigue, what action should the nurse suggest? A) Taking a hot bath at least once daily B) Resting in an air-conditioned room whenever possible C) Increasing the dose of muscle relaxants D) Avoiding naps during the day

Ans: B Feedback: Fatigue is a common symptom of patients with MS. Lowering the body temperature by resting in an air-conditioned room may relieve fatigue; however, extreme cold should be avoided. A hot bath or shower can increase body temperature, producing fatigue. Muscle relaxants, prescribed to reduce spasticity, can cause drowsiness and fatigue. Planning for frequent rest periods and naps can relieve fatigue. Other measures to reduce fatigue in the patient with MS include treating depression, using occupational therapy to learn energy conservation techniques, and reducing spasticity.

22. The nurse is caring for a patient who has just returned from the ERCP removal of gallstones. The nurse should monitor the patient for signs of what complications? A) Pain and peritonitis B) Bleeding and perforation C) Acidosis and hypoglycemia D) Gangrene of the gallbladder and hyperglycemia

Ans: B Feedback: Following ERCP removal of gallstones, the patient is observed closely for bleeding, perforation, and the development of pancreatitis or sepsis. Blood sugar alterations, gangrene, peritonitis, and acidosis are less likely complications.

20. An individual has accidentally cut his hand, immediately initiating the process of hemostasis. Following vasoconstriction, what event in the process of hemostasis will take place? A) Fibrin will be activated at the bleeding site. B) Platelets will aggregate at the injury site. C) Thromboplastin will form a clot. D) Prothrombin will be converted to thrombin.

Ans: B Feedback: Following vasoconstriction, circulating platelets aggregate at the site and adhere to the vessel and to one another, forming an unstable hemostatic plug. Events involved in the clotting cascade take place subsequent to this initial platelet action.

27. A nurse has participated in organizing a blood donation drive at a local community center. Which of the following individuals would most likely be disallowed from donating blood? A) A man who is 81 years of age B) A woman whose blood pressure is 88/51 mm Hg C) A man who donated blood 4 months ago D) A woman who has type 1 diabetes

Ans: B Feedback: For potential blood donors, systolic arterial BP should be 90 to 180 mm Hg, and the diastolic pressure should be 50 to 100 mm Hg. There is no absolute upper age limit. Donation 4 months ago does not preclude safe repeat donation and diabetes is not a contraindication.

5. The nursing educator is reviewing the signs and symptoms of heat stroke with a group of nurses who provide care in a desert region. The educator should describe what sign or symptom? A) Hypertension with a wide pulse pressure B) Anhidrosis C) Copious diuresis D) Cheyne-Stokes respirations

Ans: B Feedback: Heat stroke is manifested by anhidrosis confusion, bizarre behavior, coma, elevated body temperature, hot dry skin, tachypnea, hypotension, and tachycardia. This health problem is not associated with anhidrosis or Cheyne-Stokes respirations.

22. A client's health history reveals daily consumption of two to three bottles of wine. The nurse should plan assessments and interventions in light of the patient's increased risk for what hematologic disorder? A) Leukemia B) Anemia C) Thrombocytopenia D) Lymphoma

Ans: B Feedback: Heavy alcohol use is associated with numerous health problems, including anemia. Leukemia and lymphoma are not associated with alcohol use; RBC levels are typically affected more than platelet levels.

2. A patient is brought to the trauma center by ambulance after sustaining a high cervical spinal cord injury 1 hours ago. Endotracheal intubation has been deemed necessary and the nurse is preparing to assist. What nursing diagnosis should the nurse associate with this procedure? A) Risk for impaired skin integrity B) Risk for injury C) Risk for autonomic dysreflexia D) Risk for suffocation

Ans: B Feedback: If endotracheal intubation is necessary, extreme care is taken to avoid flexing or extending the patients neck, which can result in extension of a cervical injury. Intubation does not directly cause autonomic dysreflexia and the threat to skin integrity is a not a primary concern. Intubation does not carry the potential to cause suffocation.

20. Following a spinal cord injury a patient is placed in halo traction. While performing pin site care, the nurse notes that one of the traction pins has become detached. The nurse would be correct in implementing what priority nursing action? A) Complete the pin site care to decrease risk of infection. B) Notify the neurosurgeon of the occurrence. C) Stabilize the head in a lateral position. D) Reattach the pin to prevent further head trauma.

Ans: B Feedback: If one of the pins became detached, the head is stabilized in neutral position by one person while another notifies the neurosurgeon. Reattaching the pin as a nursing intervention would not be done due to risk of increased injury. Pin site care would not be a priority in this instance. Prevention of neurologic injury is the priority.

14. A patient has had a laparoscopic cholecystectomy. The patient is now complaining of right shoulder pain. What should the nurse suggest to relieve the pain? A) Aspirin every 4 to 6 hours as ordered B) Application of heat 15 to 20 minutes each hour C) Application of an ice pack for no more than 15 minutes D) Application of liniment rub to affected area

Ans: B Feedback: If pain occurs in the right shoulder or scapular area (from migration of the CO2 used to insufflate the abdominal cavity during the procedure), the nurse may recommend use of a heating pad for 15 to 20 minutes hourly, walking, and sitting up when in bed. Aspirin would constitute a risk for bleeding.

19. A nurse is caring for a patient with a blocked bile duct from a tumor. What manifestation of obstructive jaundice should the nurse anticipate? A) Watery, blood-streaked diarrhea B) Orange and foamy urine C) Increased abdominal girth D) Decreased cognition

Ans: B Feedback: If the bile duct is obstructed, the bile will be reabsorbed into the blood and carried throughout the entire body. It is excreted in the urine, which becomes deep orange and foamy. Bloody diarrhea, ascites, and cognitive changes are not associated with obstructive jaundice.

23. A patient is being treated for AKI and the patient daily weights have been ordered. The nurse notes a weight gain of 3 pounds over the past 48 hours. What nursing diagnosis is suggested by this assessment finding? A) Imbalanced nutrition: More than body requirements B) Excess fluid volume C) Sedentary lifestyle D) Adult failure to thrive

Ans: B Feedback: If the patient with AKI gains or does not lose weight, fluid retention should be suspected. Short-term weight gain is not associated with excessive caloric intake or a sedentary lifestyle. Failure to thrive is not associated with weight gain.

40. A patient with chronic kidney disease is completing an exchange during peritoneal dialysis. The nurse observes that the peritoneal fluid is draining slowly and that the patient's abdomen is increasing in girth. What is the nurse's most appropriate action? A) Advance the catheter 2 to 4 cm further into the peritoneal cavity. B) Reposition the patient to facilitate drainage. C) Aspirate from the catheter using a 60-mL syringe. D) Infuse 50 mL of additional dialysate.

Ans: B Feedback: If the peritoneal fluid does not drain properly, the nurse can facilitate drainage by turning the patient from side to side or raising the head of the bed. The catheter should never be pushed further into the peritoneal cavity. It would be unsafe to aspirate or to infuse more dialysate.

27. A nurse is undergoing debriefing with the critical incident stress management (CISM) team after participating in the response to a disaster. During this process, the nurse will do which of the following? A) Evaluate the care that he or she provided during the disaster. B) Discuss own emotional responses to the disaster. C) Explore the ethics of the care provided during the disaster. D) Provide suggestions for improving the emergency operations plan.

Ans: B Feedback: In debriefing, participants are asked about their emotional reactions to the incident, what symptoms they may be experiencing (e.g., flashbacks, difficulty sleeping, intrusive thoughts), and other psychological ramifications. The EOP and the care the nurse provided are not evaluated.

24. A nurse is triaging patients after a chemical leak at a nearby fertilizer factory. The guiding principle of this activity is what? A) Assigning a high priority to the most critical injuries B) Doing the greatest good for the greatest number of people C) Allocating resources to the youngest and most critical D) Allocating resources on a first come, first served basis

Ans: B Feedback: In nondisaster situations, health care workers assign a high priority and allocate the most resources to those who are the most critically ill. However, in a disaster, when health care providers are faced with a large number of casualties, the fundamental principle guiding resource allocation is to do the greatest good for the greatest number of people. A first come, first served approach is unethical.

15. The nurse is caring for a patient who describes changes in his voiding patterns. The patient states, "I feel the urge to empty my bladder several times an hour and when the urge hits me I have to get to the restroom quickly. But when I empty my bladder, there doesn't seem to be a great deal of urine flow." What would the nurse expect this patient's physical assessment to reveal? A) Hematuria B) Urine retention C) Dehydration D) Renal failure

Ans: B Feedback: Increased urinary urgency and frequency coupled with decreasing urine volumes strongly suggest urine retention. Hematuria may be an accompanying symptom, but is likely related to a urinary tract infection secondary to the retention of urine. Dehydration and renal failure both result in a decrease in urine output, but the patient with these conditions does not have normal urine production and decreased or minimal flow of urine to the bladder. The symptoms of urgency and frequency do not accompany renal failure and dehydration due to decreased urine production.

8. A patient with spinal cord injury has a nursing diagnosis of altered mobility and the nurse recognizes the increased the risk of deep vein thrombosis (DVT). Which of the following would be included as an appropriate nursing intervention to prevent a DVT from occurring? A) Placing the patient on a fluid restriction as ordered B) Applying thigh-high elastic stockings C) Administering an antifibrinolyic agent D) Assisting the patient with passive range of motion (PROM) exercises

Ans: B Feedback: It is important to promote venous return to the heart and prevent venous stasis in a patient with altered mobility. Applying elastic stockings will aid in the prevention of a DVT. The patient should not be placed on fluid restriction because a dehydrated state will increase the risk of clotting throughout the body. Antifibrinolytic agents cause the blood to clot, which is absolutely contraindicated in this situation. PROM exercises are not an effective protection against the development of DVT.

39. A group of military nurses are reviewing the care of victims of biochemical terrorist attacks. The nurses should identify what agents as having the shortest latency? A) Viral agents B) Nerve agents C) Pulmonary agents D) Blood agents

Ans: B Feedback: Latency is the time from absorption to the appearance of signs and symptoms. Sulfur mustards and pulmonary agents have the longest latency, whereas vesicants, nerve agents, and cyanide produce signs and symptoms within seconds.

4. A nurse is caring for patients exposed to a terrorist attack involving chemicals. The nurse has been advised that personal protective equipment must be worn in order to give the highest level of respiratory protection with a lesser level of skin and eye protection. What level protection is this considered? A) Level A B) Level B C) Level C D) Level D

Ans: B Feedback: Level B personal protective equipment provides the highest level of respiratory protection, with a lesser level of skin and eye protection. Level A provides the highest level of respiratory, mucous membrane, skin, and eye protection. Level C incorporates the use of an air-purified respirator, a chemical resistant coverall with splash hood, chemical resistant gloves, and boots. Level D is the same as a work uniform.

1. The nurse manager in the ED receives information that a local chemical plant has had a chemical leak. This disaster is assigned a status of level II. What does this classification indicate? A) First responders can manage the situation. B) Regional efforts and aid from surrounding communities can manage the situation. C) Statewide or federal assistance is required. D) The area must be evacuated immediately.

Ans: B Feedback: Level II disasters indicate that regional efforts and aid from the surrounding communities will be able to manage the situation. Local efforts are likely to be overwhelmed, while state and federal assistance are not likely necessary. The disaster level does not indicate the necessity of evacuation.

4. A patient has experienced an electrical burn and has developed thick eschar over the burn site. Which of the following topical antibacterial agents will the nurse expect the physician to order for the wound? A) Silver sulfadiazine 1% (Silvadene) water-soluble cream B) Mafenide acetate 10% (Sulfamylon) hydrophilic-based cream C) Silver nitrate 0.5% aqueous solution D) Acticoat

Ans: B Feedback: Mafenide acetate 10% hydrophilic-based cream is the agent of choice when there is a need to penetrate thick eschar. Silver products do not penetrate eschar; Acticoat is a type of silver dressing.

6. The staff educator is precepting a nurse new to the critical care unit when a patient with a T2 spinal cord injury is admitted. The patient is soon exhibiting manifestations of neurogenic shock. In addition to monitoring the patient closely, what would be the nurses most appropriate action? A) Prepare to transfuse packed red blood cells. B) Prepare for interventions to increase the patients BP. C) Place the patient in the Trendelenberg position. D) Prepare an ice bath to lower core body temperature.

Ans: B Feedback: Manifestations of neurogenic shock include decreased BP and heart rate. Cardiac markers would be expected to rise in cardiogenic shock. Transfusion, repositioning, and ice baths are not indicated interventions.

15. While assisting with the surgical removal of an adrenal tumor, the OR nurse is aware that the patients vital signs may change upon manipulation of the tumor. What vital sign changes would the nurse expect to see? A) Hyperthermia and tachypnea B) Hypertension and heart rate changes C) Hypotension and hypothermia D) Hyperthermia and bradycardia

Ans: B Feedback: Manipulation of the tumor during surgical excision may cause release of stored epinephrine and norepinephrine, with marked increases in BP and changes in heart rate. The use of sodium nitroprusside and alpha-adrenergic blocking agents may be required during and after surgery. While other vital sign changes may occur related to surgical complications, the most common changes are related to hypertension and changes in the heart rate.

13. The nurse is describing normal RBC physiology to a patient who has a diagnosis of anemia. The nurse should explain that the RBCs consist primarily of which of the following? A) Plasminogen B) Hemoglobin C) Hematocrit D) Fibrin

Ans: B Feedback: Mature erythrocytes consist primarily of hemoglobin, which contains iron and makes up 95% of the cell mass. RBCs are not made of fibrin or plasminogen. Hematocrit is a measure of RBC volume in whole blood.

10. The nurse is preparing the patient for mechanical debridement and informs the patient that this will involve which of the following procedures? A) A spontaneous separation of dead tissue from the viable tissue B) Removal of eschar until the point of pain and bleeding occurs C) Shaving of burned skin layers until bleeding, viable tissue is revealed D) Early closure of the wound

Ans: B Feedback: Mechanical debridement can be achieved through the use of surgical scissors, scalpels, or forceps to remove the eschar until the point of pain and bleeding occurs. Mechanical debridement can also be accomplished through the use of topical enzymatic debridement agents. The spontaneous separation of dead tissue from the viable tissue is an example of natural debridement. Shaving the burned skin layers and early wound closure are examples of surgical debridement.

10. A nurse is caring for a patient who has been scheduled for endoscopic retrograde cholangiopancreatography (ERCP) the following day. When providing anticipatory guidance for this patient, the nurse should describe what aspect of this diagnostic procedure? A) The need to protect the incision postprocedure B) The use of moderate sedation C) The need to infuse 50% dextrose during the procedure D) The use of general anesthesia

Ans: B Feedback: Moderate sedation, not general anesthesia, is used during ERCP. D50 is not administered and the procedure does not involve the creation of an incision.

32. A nurse is caring for a patient with cirrhosis secondary to heavy alcohol use. The nurses most recent assessment reveals subtle changes in the patients cognition and behavior. What is the nurses most appropriate response? A) Ensure that the patients sodium intake does not exceed recommended levels. B) Report this finding to the primary care provider due to the possibility of hepatic encephalopathy. C) Inform the primary care provider that the patient should be assessed for alcoholic hepatitis. D) Implement interventions aimed at ensuring a calm and therapeutic care environment.

Ans: B Feedback: Monitoring is an essential nursing function to identify early deterioration in mental status. The nurse monitors the patients mental status closely and reports changes so that treatment of encephalopathy can be initiated promptly. This change in status is likely unrelated to sodium intake and would not signal the onset of hepatitis. A supportive care environment is beneficial, but does not address the patients physiologic deterioration.

39. A patient who has been taking corticosteroids for several months has been experiencing muscle wasting. The patient has asked the nurse for suggestions to address this adverse effect. What should the nurse recommend? A) Activity limitation to conserve energy B) Consumption of a high-protein diet C) Use of OTC vitamin D and calcium supplements D) Passive range-of-motion exercises

Ans: B Feedback: Muscle wasting can be partly addressed through increased protein intake. Passive ROM exercises maintain flexibility, but do not build muscle mass. Vitamin D and calcium supplements do not decrease muscle wasting. Activity limitation would exacerbate the problem.

31. A patient is admitted to the ED with an apparent overdose of IV heroin. After stabilizing the patient's cardiopulmonary status, the nurse should prepare to perform what intervention? A) Administer a bolus of lactated Ringer's. B) Administer naloxone hydrochloride (Narcan). C) Insert an indwelling urinary catheter. D) Perform a focused neurologic assessment.

Ans: B Feedback: Narcan is an opioid antagonist that is administered for the treatment of narcotic overdoses. There is no definitive need for a urinary catheter or for a bolus of lactated Ringer's. The patient's basic neurologic status should be ascertained during the rapid assessment, but a detailed examination would be take precedence over administration of an antidote.

25. A patient on the medical unit has a documented history of polycystic kidney disease (PKD). What principle should guide the nurse's care of this patient? A) The disease is self-limiting and cysts usually resolve spontaneously in the fifth or sixth decade of life. B) The patient's disease is incurable and the nurse's interventions will be supportive. C) The patient will eventually require surgical removal of his or her renal cysts. D) The patient is likely to respond favorably to lithotripsy treatment of the cysts.

Ans: B Feedback: PKD is incurable and care focuses on support and symptom control. It is not self-limiting and is not treated surgically or with lithotripsy.

36. A nurse has entered the room of a patient with cirrhosis and found the patient on the floor. The patient states that she fell when transferring to the commode. The patients vital signs are within reference ranges and the nurse observes no apparent injuries. What is the nurses most appropriate action? A) Remove the patients commode and supply a bedpan. B) Complete an incident report and submit it to the unit supervisor. C) Have the patient assessed by the physician due to the risk of internal bleeding. D) Perform a focused abdominal assessment in order to rule out injury.

Ans: C Feedback: A fall would necessitate thorough medical assessment due to the patients risk of bleeding. The nurses abdominal assessment is an appropriate action, but is not wholly sufficient to rule out internal injury. Medical assessment is a priority over removing the commode or filling out an incident report, even though these actions are appropriate.

22. After a radiation exposure, a patient has been assessed and determined to be a possible survivor. Following the resolution of the patient's initial symptoms, the care team should anticipate what event? A) A return to full health B) Internal bleeding C) A latent phase D) Massive tissue necrosis

Ans: C Feedback: A latent phase commonly follows the prodromal phase of radiation exposure. The patient is deemed a possible survivor, not a probable survivor, so an immediate return to health is unlikely. However, internal bleeding and massive tissue necrosis would not be expected in a patient categorized as a possible survivor.

A nurse is caring for client with sickle cell disease (SCD) reviews the client's laboratory work. Which finding should the nurse report to the provider? A. Creatinine: 2.9 mg/dl B. Hematocrit: 30% C. Sodium: 147 mEq/L D. White blood cell count: 12,000/mm3

ANS A. Creatinine: 2.9 mg/dl

During assessment, the nurse notices a systolic murmur on a client with anemia. The nurse interprets this finding correlates with which underlying pathophysiological mechanism? A. Increased quantity and speed of low-viscosity blood through valves B. Structural abnormality of heart valves from anemia C. High viscosity of blood circulating through the cardiac structures D. Decreased blood flow through the vascular system

ANS A. Increased quantity and speed of low-viscosity blood through valves

Signs and symptoms of a bacterial transfusion reaction include all except: A. Diarrhea B. Hypertension C. Vomiting D. Flushed Skin

ANS B. Hypertension

What are the signs and symptoms of an allergic blood transfusion reaction (Select all that apply)? A. Swelling of the tongue B. Pulmonary edema C. Pruritus D. Urticaria

ANS: A, B, C, D

The nurse is teaching a newly diagnosed Amyotrophic Lateral Sclerosis (ALS) patient about the disease process for ALS. Which is the best answer as it applies to ALS? A. ALS is a disease of unknown cause in which there is a loss of motor neurons in the anterior horn of the spinal cord and the motor nuclei of the lower brain stem. B. ALS is an autoimmune disorder affecting the myoneural junction. C. ALS is an autoimmune attack of the peripheral nerve myelin. D. ALS is an immune-mediated, progressive demyelinating disease of the CNS.

ANS: A. ALS is a disease of unknown cause in which there is a loss of motor neurons in the anterior horn of the spinal cord and the motor nuclei of the lower brain stem.

Fresh-frozen plasma (FFP) has been ordered for a hospital patient. Prior to administration of this blood product, the nurse should prioritize what patient education? A. Signs and symptoms of a transfusion reaction B. Infection risks associated with FFP administration C. Physiologic functions of plasma D. Strategies for managing transfusion-associated anxiety

ANS: A. Signs and symptoms of a transfusion reaction

The Hepatitis B may be transmitted through breaks in the skin. A. True B. False

ANS: A. True Rationale: The Hepatitis B virus (HBV) may be transmitted to others through blood and body fluid contact. HBV is spread when blood or body fluids from an infected person enters the body of a person who is not infected, through breaks in the skin or through the moist linings of the eyes, nose, mouth, and genitalia.

Is the following statement true or false? Polycythemia is a form of anemia. A. True B. False

ANS: B. False

Management of hepatitis includes low calorie protein diet of less than 1g/kg/day. A. True B. False

ANS: B. False Rationale: It should be 1-1.5g/kg/day

Age related changes in hepatobiliary system include all except: A. Decreased weight and size of liver B. Increased blood flow C. Atypical presentation of biliary disease D. More rapid progression of Hepatitis C

ANS: B. Increased blood flow.

The nurse is completing documentation for a client with acute kidney injury who is being discharged today. The nurse notices that the client has a serum potassium level of 5.8mEq/L. Which is the priority nursing action? A. Asking the client to drink an extra 500mL of water B. Encouraging the client to eat potassium-binding foods C. Checking the remaining values on electrolyte panel and informing the primary care provider of all results prior to discharge. D. Evaluating client's muscle strength and muscle irritability.

ANS: C. Checking the remaining values on electrolyte panel and informing the primary care provider of all results prior to discharge.

A nurse is caring for a patient who is in the diuresis phase of acute kidney injury (AKI). The nurse should closely monitor the patient for what complication during this phase? A. Hypokalemia B. Hypocalcemia C. Dehydration D. Acute flank pain

ANS: C. Dehydration

The nurse caring for a client undergoing a hemodialysis procedure places high priority on evaluating the client frequently for what common complication during the treatment? A. Hyperglycemia B. Infection and fever C. Hypotension D. Dialysis dementia

ANS: C. Hypotension

Oversecretion (hypersecretion) of the anterior pituitary gland results in what endocrine disorder? A. Hypothyroidism B. Cushing syndrome C. Hyperthyroidism D. Addison's disease

ANS: Cushing syndrome

The nurse is assessing a patient suspected of having developed acute glomerulonephritis. The nurse should expect to address what clinical manifestation that is characteristic of this health problem? A. Precipitous decrease in serum creatinine levels B. Glucosuria C. Hypotension unresolved by fluid administration D. Hematuria

ANS: D. Hematuria Rationale: The primary presenting feature of acute glomerulonephritis is hematuria (blood in the urine), which may be microscopic (identifiable through microscopic examination) or macroscopic or gross (visible to the eye). Proteinuria, primarily albumin, which is present, is due to increased permeability of the glomerular membrane. Blood urea nitrogen (BUN) and serum creatinine levels may rise as urine output drops. Some degree of edema and hypertension is noted in most patients.

A client is to receive a unit of packed red blood cells (PRBCs) as ordered by the attending physician. Immediately prior to obtaining the blood product from the facility blood bank, what client assessment should the nurse make? A. Skin color B. Hemoglobin level C. Urine output D. Vital signs

ANS: D. Vital Signs

12. A patient has been diagnosed with a lymphoid stem cell defect. This patient has the potential for a problem involving which of the following? A) Plasma cells B) Neutrophils C) Red blood cells D) Platelets

Ans: A Feedback: A defect in a myeloid stem cell can cause problems with erythrocyte, leukocyte, and platelet production. In contrast, a defect in the lymphoid stem cell can cause problems with T or B lymphocytes, plasma cells (a more differentiated form of B lymphocyte), or natural killer (NK) cells.

39. Dipstick testing of an older adult patient's urine indicates the presence of protein. Which of the following statements is true of this assessment finding? A) This finding needs to be considered in light of other forms of testing. B) This finding is a risk factor for urinary incontinence. C) This finding is likely the result of an age-related physiologic change. D) This result confirms that the patient has diabetes.

Ans: A Feedback: A dipstick examination, which can detect from 30 to 1000 mg/dL of protein, should be used as a screening test only, because urine concentration, pH, hematuria, and radiocontrast materials all affect the results. Proteinuria is not diagnostic of diabetes and it is neither an age-related change nor a risk factor for incontinence.

28. A nurse at a blood donation clinic has completed the collection of blood from a woman. The woman states that she feels lightheaded and she appears visibly pale. What is the nurse's most appropriate action? A) Help her into a sitting position with her head lowered below her knees. B) Administer supplementary oxygen by nasal prongs. C) Obtain a full set of vital signs. D) Inform a physician or other primary care provider.

Ans: A Feedback: A donor who appears pale or complains of faintness should immediately lie down or sit with the head lowered below the knees. He or she should be observed for another 30 minutes. There is no immediate need for a physician's care. Supplementary oxygen may be beneficial, but may take too much time to facilitate before a syncopal episode. Repositioning must precede assessment of vital signs.

27. A patient has a recent diagnosis of chronic pancreatitis and is undergoing diagnostic testing to determine pancreatic islet cell function. The nurse should anticipate what diagnostic test? A) Glucose tolerance test B) ERCP C) Pancreatic biopsy D) Abdominal ultrasonography

Ans: A Feedback: A glucose tolerance test evaluates pancreatic islet cell function and provides necessary information for making decisions about surgical resection of the pancreas. This specific clinical information is not provided by ERCP, biopsy, or ultrasound.

29. A public health nurse has reviewed local data about the incidence and prevalence of burn injuries in the community. These data are likely to support what health promotion effort? A) Education about home safety B) Education about safe storage of chemicals C) Education about workplace health threats D) Education about safe driving

Ans: A Feedback: A large majority of burns occur in the home setting; educational interventions should address this epidemiologic trend.

14. A patient with a C5 spinal cord injury is tetraplegic. After being moved out of the ICU, the patient complains of a severe throbbing headache. What should the nurse do first? A) Check the patients indwelling urinary catheter for kinks to ensure patency. B) Lower the HOB to improve perfusion. C) Administer analgesia. D) Reassure the patient that headaches are expected after spinal cord injuries.

Ans: A Feedback: A severe throbbing headache is a common symptom of autonomic dysreflexia, which occurs after injuries to the spinal cord above T6. The syndrome is usually brought on by sympathetic stimulation, such as bowel and bladder distention. Lowering the HOB can increase ICP. Before administering analgesia, the nurse should check the patients catheter, record vital signs, and perform an abdominal assessment. A severe throbbing headache is a dangerous symptom in this patient and is not expected.

37. A patient with MS has been admitted to the hospital following an acute exacerbation. When planning the patients care, the nurse addresses the need to enhance the patients bladder control. What aspect of nursing care is most likely to meet this goal? A) Establish a timed voiding schedule. B) Avoid foods that change the pH of urine. C) Perform intermittent catheterization q6h. D) Administer anticholinergic drugs as ordered.

Ans: A Feedback: A timed voiding schedule addresses many of the challenges with urinary continence that face the patient with MS. Interventions should be implemented to prevent the need for catheterization and anticholinergics are not normally used.

8. A nurse is preparing a patient diagnosed with benign prostatic hypertrophy (BPH) for a lower urinary tract cystoscopic examination. The nurse informs the patient that the most common temporary complication experienced after this procedure is what? A) Urinary retention B) Bladder perforation C) Hemorrhage D) Nausea

Ans: A Feedback: After a cystoscopic examination, the patient with obstructive pathology may experience urine retention if the instruments used during the examination caused edema. The nurse will carefully monitor the patient with prostatic hyperplasia for urine retention. Post-procedure, the patient will experience some hematuria, but is not at great risk for hemorrhage. Unless the condition is associated with another disorder, nausea is not commonly associated with this diagnostic study. Bladder perforation is rare.

24. A patient is admitted to the ED after being involved in a motor vehicle accident. The patient has multiple injuries. After establishing an airway and adequate ventilation, the ED team should prioritize what aspect of care? A) Control the patient's hemorrhage. B) Assess for cognitive effects of the injury. C) Splint the patient's fractures. D) Assess the patient's neurologic status.

Ans: A Feedback: After establishing airway and ventilation, the team should evaluate and restore cardiac output by controlling hemorrhage. This must precede neurologic assessments and treatment of skeletal injuries.

10. The nurse is caring for a patient postoperative day 4 following a kidney transplant. When assessing for potential signs and symptoms of rejection, what assessment should the nurse prioritize? A) Assessment of the quantity of the patient's urine output B) Assessment of the patient's incision C) Assessment of the patient's abdominal girth D) Assessment for flank or abdominal pain

Ans: A Feedback: After kidney transplantation, the nurse should perform all of the listed assessments. However, oliguria is considered to be more suggestive of rejection than changes to the patient's abdomen or incision.

7. A nurse on a burn unit is caring for a patient in the acute phase of burn care. While performing an assessment during this phase of burn care, the nurse recognizes that airway obstruction related to upper airway edema may occur up to how long after the burn injury? A) 2 days B) 3 days C) 5 days D) 1 week

Ans: A Feedback: Airway obstruction caused by upper airway edema can take as long as 48 hours to develop. Changes detected by x-ray and arterial blood gases may occur as the effects of resuscitative fluid and the chemical reaction of smoke ingredients with lung tissues become apparent.

19. A patient's most recent blood work reveals low levels of albumin. This assessment finding should suggest the possibility of what nursing diagnosis? A) Risk for imbalanced fluid volume related to low albumin B) Risk for infection related to low albumin C) Ineffective tissue perfusion related to low albumin D) Impaired skin integrity related to low albumin

Ans: A Feedback: Albumin is particularly important for the maintenance of fluid balance within the vascular system. Deficiencies nearly always manifest as fluid imbalances. Tissue oxygenation and skin integrity are not normally affected. Low albumin does not constitute a risk for infection.

21. A patient with suspected adrenal insufficiency has been ordered an adrenocorticotropic hormone (ACTH) stimulation test. Administration of ACTH caused a marked increase in cortisol levels. How should the nurse interpret this finding? A) The patients pituitary function is compromised. B) The patients adrenal insufficiency is not treatable. C) The patient has insufficient hypothalamic function. D) The patient would benefit from surgery.

Ans: A Feedback: An adrenal response to the administration of a stimulating hormone suggests inadequate production of the stimulating hormone. In this case, ACTH is produced by the pituitary and, consequently, pituitary hypofunction is suggested. Hypothalamic function is not relevant to the physiology of this problem. Treatment exists, although surgery is not likely indicated.

17. A nurse is caring for a 73-year-old patient with a urethral obstruction related to prostatic enlargement. When planning this patient's care, the nurse should be aware of the consequent risk of what complication? A) Urinary tract infection B) Enuresis C) Polyuria D) Proteinuria

Ans: A Feedback: An obstruction of the bladder outlet, such as in advanced benign prostatic hyperplasia, results in abnormally high voiding pressure with a slow, prolonged flow of urine. The urine may remain in the bladder, which increases the potential of a urinary tract infection. Older male patients are at risk for prostatic enlargement, which causes urethral obstruction and can result in hydronephrosis, renal failure, and urinary tract infections.

7. A patient has been scheduled for an ultrasound of the gallbladder the following morning. What should the nurse do in preparation for this diagnostic study? A) Have the patient refrain from food and fluids after midnight. B) Administer the contrast agent orally 10 to 12 hours before the study. C) Administer the radioactive agent intravenously the evening before the study. D) Encourage the intake of 64 ounces of water 8 hours before the study.

Ans: A Feedback: An ultrasound of the gallbladder is most accurate if the patient fasts overnight, so that the gallbladder is distended. Contrast and radioactive agents are not used when performing ultrasonography of the gallbladder, as an ultrasound is based on reflected sound waves.

18. A nurse is caring for a patient in the emergent/resuscitative phase of burn injury. During this phase, the nurse should monitor for evidence of what alteration in laboratory values? A) Sodium deficit B) Decreased prothrombin time (PT) C) Potassium deficit D) Decreased hematocrit

Ans: A Feedback: Anticipated fluid and electrolyte changes that occur during the emergent/resuscitative phase of burn injury include sodium deficit, potassium excess, base-bicarbonate deficit, and elevated hematocrit. PT does not typically decrease.

27. A patient is admitted to the burn unit after being transported from a facility 1000 miles away. The patient has burns to the groin area and circumferential burns to both upper thighs. When assessing the patients legs distal to the wound site, the nurse should be cognizant of the risk of what complication? A) Ischemia B) Referred pain C) Cellulitis D) Venous thromboembolism (VTE)

Ans: A Feedback: As edema increases, pressure on small blood vessels and nerves in the distal extremities causes an obstruction of blood flow and consequent ischemia. This complication is similar to compartment syndrome. Referred pain, cellulitis, and VTE are not noted complications that occur distal to the injury site.

3. A patient with thyroid cancer has undergone surgery and a significant amount of parathyroid tissue has been removed. The nurse caring for the patient should prioritize what question when addressing potential complications? A) Do you feel any muscle twitches or spasms? B) Do you feel flushed or sweaty? C) Are you experiencing any dizziness or lightheadedness? D) Are you having any pain that seems to be radiating from your bones?

Ans: A Feedback: As the blood calcium level falls, hyperirritability of the nerves occurs, with spasms of the hands and feet and muscle twitching. This is characteristic of hypoparathyroidism. Flushing, diaphoresis, dizziness, and pain are atypical signs of the resulting hypocalcemia.

32. The nurse has identified the nursing diagnosis of "risk for infection" in a patient who undergoes peritoneal dialysis. What nursing action best addresses this risk? A) Maintain aseptic technique when administering dialysate. B) Wash the skin surrounding the catheter site with soap and water prior to each exchange. C) Add antibiotics to the dialysate as ordered. D) Administer prophylactic antibiotics by mouth or IV as ordered.

Ans: A Feedback: Aseptic technique is used to prevent peritonitis and other infectious complications of peritoneal dialysis. It is not necessary to cleanse the skin with soap and water prior to each exchange. Antibiotics may be added to dialysate to treat infection, but they are not used to prevent infection.

29. The school nurse has been called to the football field where player is immobile on the field after landing awkwardly on his head during a play. While awaiting an ambulance, what action should the nurse perform? A) Ensure that the player is not moved. B) Obtain the players vital signs, if possible. C) Perform a rapid assessment of the players range of motion. D) Assess the players reflexes.

Ans: A Feedback: At the scene of the injury, the patient must be immobilized on a spinal (back) board, with the head and neck maintained in a neutral position, to prevent an incomplete injury from becoming complete. This is a priority over determining the patients vital signs. It would be inappropriate to test ROM or reflexes.

20. During a health education session, a participant has asked about the hepatitis E virus. What prevention measure should the nurse recommend for preventing infection with this virus? A) Following proper hand-washing techniques B) Avoiding chemicals that are toxic to the liver C) Wearing a condom during sexual contact D) Limiting alcohol intake

Ans: A Feedback: Avoiding contact with the hepatitis E virus through good hygiene, including hand-washing, is the major method of prevention. Hepatitis E is transmitted by the fecaloral route, principally through contaminated water in areas with poor sanitation. Consequently, none of the other listed preventative measures is indicated.

30. The nurse is caring for a patient whose spinal cord injury has caused recent muscle spasticity. What medication should the nurse expect to be ordered to control this? A) Baclofen (Lioresal) B) Dexamethasone (Decadron) C) Mannitol (Osmitrol) D) Phenobarbital (Luminal)

Ans: A Feedback: Baclofen is classified as an antispasmodic agent in the treatment of muscles spasms related to spinal cord injury. Decadron is an anti-inflammatory medication used to decrease inflammation in both SCI and head injury. Mannitol is used to decrease cerebral edema in patients with head injury. Phenobarbital is an anticonvulsant that is used in the treatment of seizure activity.

18. The nurse is caring for a patient who is hospitalized with an exacerbation of MS. To ensure the patients safety, what nursing action should be performed? A) Ensure that suction apparatus is set up at the bedside. B) Pad the patients bed rails. C) Maintain bed rest whenever possible. D) Provide several small meals each day.

Ans: A Feedback: Because of the patients risk of aspiration, it is important to have a suction apparatus at hand. Bed rest should be generally be minimized, not maximized, and there is no need to pad the patients bed rails or to provide multiple small meals.

35. A patient with Cushing syndrome as a result of a pituitary tumor has been admitted for a transsphenoidal hypophysectomy. What would be most important for the nurse to monitor before, during, and after surgery? A) Blood glucose B) Assessment of urine for blood C) Weight D) Oral temperature

Ans: A Feedback: Before, during, and after this surgery, blood glucose monitoring and assessment of stools for blood are carried out. The patients blood sugar is more likely to be volatile than body weight or temperature. Hematuria is not a common complication.

5. A nurse is caring for a patient with cancer of the liver whose condition has required the insertion of a percutaneous biliary drainage system. The nurses most recent assessment reveals the presence of dark green fluid in the collection container. What is the nurses best response to this assessment finding? A) Document the presence of normal bile output. B) Irrigate the drainage system with normal saline as ordered. C) Aspirate a sample of the drainage for culture. D) Promptly report this assessment finding to the primary care provider.

Ans: A Feedback: Bile is usually a dark green or brownish-yellow color, so this would constitute an expected assessment finding, with no other action necessary.

37. A patient who is being treated in the hospital for a spinal cord injury is advocating for the removal of his urinary catheter, stating that he wants to try to resume normal elimination. What principle should guide the care teams decision regarding this intervention? A) Urinary retention can have serious consequences in patients with SCIs. B) Urinary function is permanently lost following an SCI. C) Urinary catheters should not remain in place for more than 7 days. D) Overuse of urinary catheters can exacerbate nerve damage.

Ans: A Feedback: Bladder distention, a major cause of autonomic dysreflexia, can also cause trauma. For this reason, removal of a urinary catheter must be considered with caution. Extended use of urinary catheterization is often necessary following SCI. The effect of a spinal cord lesion on urinary function depends on the level of the injury. Catheter use does not cause nerve damage, although it is a major risk factor for UTIs.

33. The nurse is caring for a patient who has returned to the postsurgical suite after post-anesthetic recovery from a nephrectomy. The nurse's most recent hourly assessment reveals a significant drop in level of consciousness and BP as well as scant urine output over the past hour. What is the nurse's best response? A) Assess the patient for signs of bleeding and inform the physician. B) Monitor the patient's vital signs every 15 minutes for the next hour. C) Reposition the patient and reassess vital signs. D) Palpate the patient's flanks for pain and inform the physician.

Ans: A Feedback: Bleeding may be suspected when the patient experiences fatigue and when urine output is less than 30 mL/h. The physician must be made aware of this finding promptly. Palpating the patient's flanks would cause intense pain that is of no benefit to assessment.

35. A burn patient is transitioning from the acute phase of the injury to the rehabilitation phase. The patient tells the nurse, I cant wait to have surgery to reconstruct my face so I look normal again. What would be the nurses best response? A) Thats something that you and your doctor will likely talk about after your scars mature. B) That is something for you to talk to your doctor about because its not a nursing responsibility. C) I know this is really important to you, but you have to realize that no one can make you look like you used to. D) Unfortunately, its likely that you will have most of these scars for the rest of your life.

Ans: A Feedback: Burn reconstruction is a treatment option after all scars have matured and is discussed within the first few years after injury. Even though this is not a nursing responsibility, the nurse should still respond appropriately to the patients query. It is true that the patient will not realistically look like he or she used to, but this does not instill hope.

12. Renal failure can have prerenal, renal, or postrenal causes. A patient with acute kidney injury is being assessed to determine where, physiologically, the cause is. If the cause is found to be prerenal, which condition most likely caused it? A) Heart failure B) Glomerulonephritis C) Ureterolithiasis D) Aminoglycoside toxicity

Ans: A Feedback: By causing inadequate renal perfusion, heart failure can lead to prerenal failure. Glomerulonephritis and aminoglycoside toxicity are renal causes, and ureterolithiasis is a postrenal cause.

28. A man survived a workplace accident that claimed the lives of many of his colleagues several months ago. The man has recently sought care for the treatment of depression. How should the nurse best understand the man's current mental health problem? A) The man is experiencing a common response following a disaster. B) The man fails to appreciate the fact that he survived the disaster. C) The man most likely feels guilty about his actions during the disaster. D) The man's depression most likely predated the disaster.

Ans: A Feedback: Depression is a common response to disaster. It does not suggest that the patient feels guilty about his actions or that he does not appreciate the fact that he survived. It is possible, but less likely, that the patient was depressed prior to the disaster.

24. An adult patient has been admitted to the medical unit for the treatment of acute pancreatitis. What nursing action should be included in this patients plan of care? A) Measure the patients abdominal girth daily. B) Limit the use of opioid analgesics. C) Monitor the patient for signs of dysphagia. D) Encourage activity as tolerated.

Ans: A Feedback: Due to the risk of ascites, the nurse should monitor the patients abdominal girth. There is no specific need to avoid the use of opioids or to monitor for dysphagia, and activity is usually limited.

4. The nurse is assessing a patient's bladder by percussion. The nurse elicits dullness after the patient has voided. How should the nurse interpret this assessment finding? A) The patient's bladder is not completely empty. B) The patient has kidney enlargement. C) The patient has a ureteral obstruction. D) The patient has a fluid volume deficit.

Ans: A Feedback: Dullness to percussion of the bladder following voiding indicates incomplete bladder emptying. Enlargement of the kidneys can be attributed to numerous conditions such as polycystic kidney disease or hydronephrosis and is not related to bladder fullness. Dehydration and ureteral obstruction are not related to bladder fullness; in fact, these conditions result in decreased flow of urine to the bladder.

25. A nurse has been called for duty during a response to a natural disaster. In this context of care, the nurse should expect to do which of the following? A) Practice outside of her normal area of clinical expertise. B) Perform interventions that are not based on assessment data. C) Prioritize psychosocial needs over physiologic needs. D) Prioritize the interests of older adults over younger patients.

Ans: A Feedback: During a disaster, nurses may be asked to perform duties outside their areas of expertise and may take on responsibilities normally held by physicians or advanced practice nurses.

28. Following an addisonian crisis, a patients adrenal function has been gradually regained. The nurse should ensure that the patient knows about the need for supplementary glucocorticoid therapy in which of the following circumstances? A) Episodes of high psychosocial stress B) Periods of dehydration C) Episodes of physical exertion D) Administration of a vaccine

Ans: A Feedback: During stressful procedures or significant illnesses, additional supplementary therapy with glucocorticoids is required to prevent addisonian crisis. Physical activity, dehydration and vaccine administration would not normally be sufficiently demanding such to require glucocorticoids.

9. A patient has been admitted to a burn intensive care unit with extensive full-thickness burns over 25% of the body. After ensuring cardiopulmonary stability, what would be the nurses immediate, priority concern when planning this patients care? A) Fluid status B) Risk of infection C) Nutritional status D) Psychosocial coping

Ans: A Feedback: During the early phase of burn care, the nurse is most concerned with fluid resuscitation, to correct large-volume fluid loss through the damaged skin. Infection control and early nutritional support are important, but fluid resuscitation is an immediate priority. Coping is a higher priority later in the recovery period.

17. A patient with a T2 injury is in spinal shock. The nurse will expect to observe what assessment finding? A) Absence of reflexes along with flaccid extremities B) Positive Babinskis reflex along with spastic extremities C) Hyperreflexia along with spastic extremities D) Spasticity of all four extremities

Ans: A Feedback: During the period immediately following a spinal cord injury, spinal shock occurs. In spinal shock, all reflexes are absent and the extremities are flaccid. When spinal shock subsides, the patient demonstrates a positive Babinskis reflex, hyperreflexia, and spasticity of all four extremities.

40. A patient is admitted to the neurologic ICU with a C4 spinal cord injury. When writing the plan of care for this patient, which of the following nursing diagnoses would the nurse prioritize in the immediate care of this patient? A) Risk for impaired skin integrity related to immobility and sensory loss B) Impaired physical mobility related to loss of motor function C) Ineffective breathing patterns related to weakness of the intercostal muscles D) Urinary retention related to inability to void spontaneously

Ans: C Feedback: A nursing diagnosis related to breathing pattern would be the priority for this patient. A C4 spinal cord injury will require ventilatory support, due to the diaphragm and intercostals being affected. The other nursing diagnoses would be used in the care plan, but not designated as a higher priority than ineffective breathing patterns.

38. An 83-year-old patient is brought in by ambulance from a long-term care facility. The patient's symptoms are weakness, lethargy, incontinence, and a change in mental status. The nurse knows that emergencies in older adults may be more difficult to manage. Why would this be true? A) Older adults may have an altered response to treatment. B) Older adults are often reluctant to adhere to prescribed treatment. C) Older adults have difficulty giving a health history. D) Older adults often stigmatize their peers who use the ED.

Ans: A Feedback: Emergencies in this age group may be more difficult to manage because elderly patients may have an atypical presentation, an altered response to treatment, a greater risk of developing complications, or a combination of these factors. The elderly patient may perceive the emergency as a crisis signaling the end of an independent lifestyle or even resulting in death. Stigmatization and nonadherence to treatment are not commonly noted. Older adults do not necessarily have difficulty giving a health history.

23. A nurse works in a walk-in clinic. The nurse recognizes that certain patients are at higher risk for different disorders than other patients. What patient is at a greater risk for the development of hypothyroidism? A) A 75-year-old female patient with osteoporosis B) A 50-year-old male patient who is obese C) A 45-year-old female patient who used oral contraceptives D) A 25-year-old male patient who uses recreational drugs

Ans: A Feedback: Even though osteoporosis is not a risk factor for hypothyroidism, the condition occurs most frequently in older women.

3. A patient in the emergent/resuscitative phase of a burn injury has had blood work and arterial blood gases drawn. Upon analysis of the patients laboratory studies, the nurse will expect the results to indicate what? A) Hyperkalemia, hyponatremia, elevated hematocrit, and metabolic acidosis B) Hypokalemia, hypernatremia, decreased hematocrit, and metabolic acidosis C) Hyperkalemia, hypernatremia, decreased hematocrit, and metabolic alkalosis D) Hypokalemia, hyponatremia, elevated hematocrit, and metabolic alkalosis

Ans: A Feedback: Fluid and electrolyte changes in the emergent/resuscitative phase of a burn injury include hyperkalemia related to the release of potassium into the extracellular fluid, hyponatremia from large amounts of sodium lost in trapped edema fluid, hemoconcentration that leads to an increased hematocrit, and loss of bicarbonate ions that results in metabolic acidosis.

31. The nurse is planning the care of a patient with a T1 spinal cord injury. The nurse has identified the diagnosis of risk for impaired skin integrity. How can the nurse best address this risk? A) Change the patients position frequently. B) Provide a high-protein diet. C) Provide light massage at least daily. D) Teach the patient deep breathing and coughing exercises.

Ans: A Feedback: Frequent position changes are among the best preventative measures against pressure ulcers. A high-protein diet can benefit wound healing, but does not necessarily prevent skin breakdown. Light massage and deep breathing do not protect or restore skin integrity.

14. A group of medical nurses are being certified in their response to potential bioterrorism. The nurses learn that if a patient is exposed to the smallpox virus he or she becomes contagious at what time? A) 6 to 12 hours after exposure B) When pustules form C) After a rash appears D) When the patient becomes febrile

Ans: C Feedback: A patient is contagious after a rash develops, which initially develops on the face, mouth, pharynx, and forearms. The patient exposed to the smallpox virus is not contagious immediately after exposure; only when pustules form, or with a body temperature of 38°C.

19. A backcountry skier has been airlifted to the ED after becoming lost and developing hypothermia and frostbite. How should the nurse best manage the patient's frostbite? A) Immerse affected extremities in water slightly above normal body temperature. B) Immerse the patient's frostbitten extremities in the warmest water the patient can tolerate. C) Gently massage the patient's frozen extremities in between water baths. D) Perform passive range-of-motion exercises of the affected extremities to promote circulation.

Ans: A Feedback: Frozen extremities are usually placed in a 37°C to 40°C (98.6°F to 104°F) circulating bath for 30- to 40-minute spans. To avoid further mechanical injury, the body part is not handled. Massage is contraindicated.

29. A group of nurses have attended an inservice on the prevention of occupationally acquired diseases that affect healthcare providers. What action has the greatest potential to reduce a nurses risk of acquiring hepatitis C in the workplace? A) Disposing of sharps appropriately and not recapping needles B) Performing meticulous hand hygiene at the appropriate moments in care C) Adhering to the recommended schedule of immunizations D) Wearing an N95 mask when providing care for patients on airborne precautions

Ans: A Feedback: HCV is bloodborne. Consequently, prevention of needlestick injuries is paramount. Hand hygiene, immunizations and appropriate use of masks are important aspects of overall infection control, but these actions do not directly mitigate the risk of HCV.

6. A patient is brought to the ED by ambulance after swallowing highly acidic toilet bowl cleaner 2 hours earlier. The patient is alert and oriented. What is the care team's most appropriate treatment? A) Administering syrup of ipecac B) Performing a gastric lavage C) Giving milk to drink D) Referring to psychiatry

Ans: C Feedback: A patient who has swallowed an acidic substance, such as toilet bowl cleaner, may be given milk or water to drink for dilution. Gastric lavage must be performed within 1 hour of ingestion. A psychiatric consult may be considered once the patient is physically stable and it is deemed appropriate by the physician. Syrup of ipecac is no longer used in clinical settings.

32. A patient is scheduled for diagnostic testing to address prolonged signs and symptoms of genitourinary dysfunction. What signs and symptoms are particularly suggestive of urinary tract disease? Select all that apply. A) Petechiae B) Pain C) Gastrointestinal symptoms D) Changes in voiding E) Jaundice

Ans: B, C, D Feedback: Dysfunction of the kidney can produce a complex array of symptoms throughout the body. Pain, changes in voiding, and gastrointestinal symptoms are particularly suggestive of urinary tract disease. Jaundice and petechiae are not associated with genitourinary health problems.

22. A patient is admitted to the ED complaining of abdominal pain. Further assessment of the abdomen reveals signs of peritoneal irritation. What assessment findings would corroborate this diagnosis? Select all that apply. A) Ascites B) Rebound tenderness C) Changes in bowel sounds D) Muscular rigidity E) Copious diarrhea

Ans: B, C, D Feedback: Signs of peritoneal irritation include abdominal distention, involuntary guarding, tenderness, pain, muscular rigidity, or rebound tenderness along with changes in bowel sounds. Diarrhea and ascites are not signs of peritoneal irritation.

9. A patient with chronic pancreatitis had a pancreaticojejunostomy created 3 months ago for relief of pain and to restore drainage of pancreatic secretions. The patient has come to the office for a routine postsurgical appointment. The patient is frustrated that the pain has not decreased. What is the most appropriate initial response by the nurse? A) The majority of patients who have a pancreaticojejunostomy have their normal digestion restored but do not achieve pain relief. B) Pain relief occurs by 6 months in most patients who undergo this procedure, but some people experience a recurrence of their pain. C) Your physician will likely want to discuss the removal of your gallbladder to achieve pain relief. D) You are probably not appropriately taking the medications for your pancreatitis and pain, so we will need to discuss your medication regimen in detail.

Ans: B Feedback: Pain relief from a pancreaticojejunostomy often occurs by 6 months in more than 85% of the patients who undergo this procedure, but pain returns in a substantial number of patients as the disease progresses. This patient had surgery 3 months ago; the patient has 3 months before optimal benefits of the procedure may be experienced. There is no obvious indication for gallbladder removal and nonadherence is not the most likely factor underlying the pain.

19. The nurse is caring for a patient who is rapidly progressing toward brain death. The nurse should be aware of what cardinal signs of brain death? Select all that apply. A) Absence of pain response B) Apnea C) Coma D) Absence of brain stem reflexes E) Absence of deep tendon reflexes

Ans: B, C, D Feedback: The three cardinal signs of brain death upon clinical examination are coma, the absence of brain stem reflexes, and apnea. Absences of pain response and deep tendon reflexes are not necessarily indicative of brain death.

37. The nurse is caring for a patient at risk for an addisonian crisis. For what associated signs and symptoms should the nurse monitor the patient? Select all that apply. A) Epistaxis B) Pallor C) Rapid respiratory rate D) Bounding pulse E) Hypotension

Ans: B, C, E Feedback: The patient at risk is monitored for signs and symptoms indicative of addisonian crisis, which can include shock; hypotension; rapid, weak pulse; rapid respiratory rate; pallor; and extreme weakness. Epistaxis and a bounding pulse are not symptoms or signs of an addisonian crisis.

34. A patient who was burned in a workplace accident has completed the acute phase of treatment and the plan of care has been altered to prioritize rehabilitation. What nursing action should be prioritized during this phase of treatment? A) Monitoring fluid and electrolyte imbalances B) Providing education to the patient and family C) Treating infection D) Promoting thermoregulation

Ans: B Feedback: Patient and family education is a priority during rehabilitation. There should be no fluid and electrolyte imbalances in the rehabilitation phase. The presence of impaired thermoregulation or infection would suggest that the patient is still in the acute phase of burn recovery.

30. A patient is scheduled for a diagnostic MRI of the lower urinary system. What pre-procedure education should the nurse include? A) The need to be NPO for 12 hours prior to the test B) Relaxation techniques to apply during the test C) The need for conscious sedation prior to the test D) The need to limit fluid intake to 1 liter in the 24 hours before the test

Ans: B Feedback: Patient preparation should include teaching relaxation techniques because the patient needs to remain still during an MRI. The patient does not normally need to be NPO or fluid-restricted before the test and conscious sedation is not usually implemented.

21. A nurse who is taking care of a patient with burns is asked by a family member why the patient is losing so much weight. The patient is currently in the intermediate phase of recovery. What would be the nurses most appropriate response to the family member? A) Hes on a calorie-restricted diet in order to divert energy to wound healing. B) His body has consumed his fat deposits for fuel because his calorie intake is lower than normal. C) He actually hasnt lost weight. Instead, theres been a change in the distribution of his body fat. D) He lost many fluids while he was being treated in the emergency phase of burn care.

Ans: B Feedback: Patients lose a great deal of weight during recovery from severe burns. Reserve fat deposits are catabolized as a result of hypermetabolism. Patients are not placed on a calorie restriction during recovery and fluid losses would not account for weight loss later in the recovery period. Changes in the overall distribution of body fat do not occur.

3. A patient is brought to the ED by ambulance with a gunshot wound to the abdomen. The nurse knows that the most common hollow organ injured in this type of injury is what? A) Liver B) Small bowel C) Stomach D) Large bowel

Ans: B Feedback: Penetrating abdominal wounds have a high incidence of injury to hollow organs, especially the small bowel. The liver is also injured frequently, but it is a solid organ.

7. The nurse is describing the role of plasminogen in the clotting cascade. Where in the body is plasminogen present? A) Myocardial muscle tissue B) All body fluids C) Cerebral tissue D) Venous and arterial vessel walls

Ans: B Feedback: Plasminogen, which is present in all body fluids, circulates with fibrinogen. Plasminogen is found in body fluids, not tissue.

39. A home care nurse is performing a visit to a patients home to perform wound care following the patients hospital treatment for severe burns. While interacting with the patient, the nurse should assess for evidence of what complication? A) Psychosis B) Post-traumatic stress disorder C) Delirium D) Vascular dementia

Ans: B Feedback: Post-traumatic stress disorder (PTSD) is the most common psychiatric disorder in burn survivors, with a prevalence that may be as high as 45%. As a result, it is important for the nurse to assess for this complication of burn injuries. Psychosis, delirium, and dementia are not among the noted psychiatric and psychosocial complications of burns.

19. The nurse is caring for a patient with acute glomerular inflammation. When assessing for the characteristic signs and symptoms of this health problem, the nurse should include which assessments? Select all that apply. A) Percuss for pain in the right lower abdominal quadrant. B) Assess for the presence of peripheral edema. C) Auscultate the patient's apical heart rate for dysrhythmias. D) Assess the patient's BP. E) Assess the patient's orientation and judgment.

Ans: B, D Feedback: Most patients with acute glomerular inflammation have some degree of edema and hypertension. Dysrhythmias, RLQ pain, and changes in mental status are not among the most common manifestations of acute glomerular inflammation.

25. A patient has been admitted to the critical care unit with a diagnosis of thyroid storm. What interventions should the nurse include in this patients immediate care? Select all that apply. A) Administering diuretics to prevent fluid overload B) Administering beta blockers to reduce heart rate C) Administering insulin to reduce blood glucose levels D) Applying interventions to reduce the patients temperature E) Administering corticosteroids

Ans: B, D Feedback: Thyroid storm necessitates interventions to reduce heart rate and temperature. Diuretics, insulin, and steroids are not indicated to address the manifestations of this health problem.

Which of the following is not a possible manifestation for a patient diagnosed with anemia? A. Malaise B. Constipation C. Pica D. Tongue changes

Ans: B. Constipation Rationale: Constipation is a side effect of iron supplementation, not anemia itself.

A 23-year-old African American male with a history of sickle cell disease had an emergency open reduction and internal fixation of his right femur after a car crash. What is the initial postoperative nursing priority? A. Treating the patient's pain B. Ensuring adequate IV hydration C. Examining the surgical incision for signs of infection D. Titrating oxygen to an Sp02 of >98%

Ans: B. Ensuring adequate IV hydration

18. A patient is scheduled for a CT scan of the abdomen with contrast. The patient has a baseline creatinine level of 2.3 mg/dL. In preparing this patient for the procedure, the nurse anticipates what orders? A) Monitor the patient's electrolyte values every hour before the procedure. B) Preprocedure hydration and administration of acetylcysteine C) Hemodialysis immediately prior to the CT scan D) Obtain a creatinine clearance by collecting a 24-hour urine specimen.

Ans: B Feedback: Radiocontrast-induced nephropathy is a major cause of hospital-acquired acute kidney injury. Baseline levels of creatinine greater than 2 mg/dL identify the patient as being high risk. Preprocedure hydration and prescription of acetylcysteine (Mucomyst) the day prior to the test is effective in prevention. The nurse would not monitor the patient's electrolytes every hour preprocedure. Nothing in the scenario indicates the need for hemodialysis. A creatinine clearance is not necessary prior to a CT scan with contrast.

What stage of chronic kidney disease (CKD) is characterized by mild chronic kidney disease? A. Stage 0 B. Stage 2 C. Stage 3 D. Stage 6

Ans: B. Stage 2

What is not a sign and symptom of thyroid storm? A. Chest pain B. Weight gain C. Palpitations D. High fever

Ans: B. Weight gain Rationale: Weight gain is a symptom of hypothyroidism. Thyroid storm is a crisis derived from hyperthyroidism, of which a key symptom is weight loss.

8. While developing an emergency operations plan (EOP), the committee is discussing the components of the EOP. During the post-incident response of an emergency operations plan, what activity will take place? A) Deciding when the facility will go from disaster response to daily activities B) Conducting practice drills for the community and facility C) Conducting a critique and debriefing for all involved in the incident D) Replacing the resources in the facility

Ans: C Feedback: A post-incident response includes critiquing and debriefing all parties involved immediately and at later dates. It does not include the decision to go from disaster response to daily activities; it does not include practice drills; and it does not include replacement of resources in the facility.

40. A nurse who provides care on a burn unit is preparing to apply a patients ordered topical antibiotic ointment. What action should the nurse perform when administering this medication? A) Apply the new ointment without disturbing the existing layer of ointment. B) Apply the ointment using a sterile tongue depressor. C) Apply a layer of ointment approximately 1/16 inch thick. D) Gently irrigate the wound bed after applying the antibiotic ointment.

Ans: C Feedback: After removing the old ointment from the wound bed, the nurse should apply a layer of ointment 1/16-inch thick using clean gloves. The wound would not be irrigated after application of new ointment.

14. A triage nurse in the emergency department (ED) receives a phone call from a frantic father who saw his 4-year-old child tip a pot of boiling water onto her chest. The father has called an ambulance. What would the nurse in the ED receiving the call instruct the father to do? A) Cover the burn with ice and secure with a towel. B) Apply butter to the area that is burned. C) Immerse the child in a cool bath. D) Avoid touching the burned area under any circumstances.

Ans: C Feedback: After the flames or heat source have been removed or extinguished, the burned area and adherent clothing are soaked with cool water briefly to cool the wound and halt the burning process. Cool water is the best first-aid measure. Ice and butter are contraindicated. Appropriate first aid necessitates touchingthe burn.

36. A nurse has had contact with a patient who developed smallpox and became febrile after a terrorist attack. This nurse will require what treatment? A) Watchful waiting B) Treatment with colony-stimulating factors (CSFs) C) Vaccination D) Treatment with ceftriaxone

Ans: C Feedback: All people who have had household or face-to-face contact with a patient with small pox after the fever begins should be vaccinated within 4 days to prevent infection and death. Watchful waiting would be inappropriate and CSFs are not used for treatment. Vaccination, rather than antibiotics, is the treatment of choice.

28. A patient is brought to the ED by friends. The friends tell the nurse that the patient was using cocaine at a party. On arrival to the ED the patient is in visible distress with an axillary temperature of 40.1ºC (104.2°F). What would be the priority nursing action for this patient? A) Monitor cardiovascular effects. B) Administer antipyretics. C) Ensure airway and ventilation. D) Prevent seizure activity.

Ans: C Feedback: Although all of the listed actions may be necessary for this patient's care, the priority is to establish a patent airway and adequate ventilation.

1. The ED nurse is caring for a patient who has been brought in by ambulance after sustaining a fall at home. What physical assessment finding is suggestive of a basilar skull fracture? A) Epistaxis B) Periorbital edema C) Bruising over the mastoid D) Unilateral facial numbness

Ans: C Feedback: An area of ecchymosis (bruising) may be seen over the mastoid (Battles sign) in a basilar skull fracture. Numbness, edema, and epistaxis are not directly associated with a basilar skull fracture.

33. A patient has been admitted to the medical unit with signs and symptoms that are suggestive of anthrax infection. The nurse should anticipate what intervention? A) Administration of acyclovir B) Hematopoietic stem cell transplantation (HSCT) C) Administration of penicillin D) Hemodialysis

Ans: C Feedback: Anthrax infection is treated with penicillin. Acyclovir is ineffective because anthrax is a bacterium. Dialysis and HSCT are not indicated.

28. A patient experienced a 33% TBSA burn 72 hours ago. The nurse observes that the patients hourly urine output has been steadily increasing over the past 24 hours. How should the nurse best respond to this finding? A) Obtain an order to reduce the rate of the patients IV fluid infusion. B) Report the patients early signs of acute kidney injury (AKI). C) Recognize that the patient is experiencing an expected onset of diuresis. D) Administer sodium chloride as ordered to compensate for this fluid loss.

Ans: C Feedback: As capillaries regain integrity, 48 or more hours after the burn, fluid moves from the interstitial to the intravascular compartment and diuresis begins. This is an expected development and does not require a reduction in the IV infusion rate or the administration of NaCl. Diuresis is not suggestive of AKI.

24. A patient with severe burns is admitted to the intensive care unit to stabilize and begin fluid resuscitation before transport to the burn center. The nurse should monitor the patient closely for what signs of the onset of burn shock? A) Confusion B) High fever C) Decreased blood pressure D) Sudden agitation

Ans: C Feedback: As fluid loss continues and vascular volume decreases, cardiac output continues to decrease and the blood pressure drops, marking the onset of burn shock. Shock and the accompanying hemodynamic changes are not normally accompanied by confusion, fever, or agitation.

11. A patient with a partial-thickness burn injury had Biobrane applied 2 weeks ago. The nurse notices that the Biobrane is separating from the burn wound. What is the nurses most appropriate intervention? A) Reinforce the Biobrane dressing with another piece of Biobrane. B) Remove the Biobrane dressing and apply a new dressing. C) Trim away the separated Biobrane. D) Notify the physician for further emergency-related orders.

Ans: C Feedback: As the Biobrane gradually separates, it is trimmed, leaving a healed wound. When the Biobrane dressing adheres to the wound, the wound remains stable and the Biobrane can remain in place for 3 to 4 weeks. There is no need to reinforce the Biobrane nor to remove it and apply a new dressing. There is not likely any need to notify the physician for further orders.

39. Which of the following circumstances would most clearly warrant autologous blood donation? A) The patient has type-O blood. B) The patient has sickle cell disease or a thalassemia. C) The patient has elective surgery pending. D) The patient has hepatitis C.

Ans: C Feedback: Autologous blood donation is useful for many elective surgeries where the potential need for transfusion is high. Type-O blood, hepatitis, sickle cell disease, and thalassemia are not clear indications for autologous donation.

11. A patient has undergone a laparoscopic cholecystectomy and is being prepared for discharge home. When providing health education, the nurse should prioritize which of the following topics? A) Management of fluid balance in the home setting B) The need for blood glucose monitoring for the next week C) Signs and symptoms of intra-abdominal complications D) Appropriate use of prescribed pancreatic enzymes

Ans: C Feedback: Because of the early discharge following laparoscopic cholecystectomy, the patient needs thorough education in the signs and symptoms of complications. Fluid balance is not typically a problem in the recovery period after laparoscopic cholecystectomy. There is no need for blood glucose monitoring or pancreatic enzymes.

30. A patient has been admitted to the critical care unit with a diagnosis of toxic hepatitis. When planning the patients care, the nurse should be aware of what potential clinical course of this health problem? Place the following events in the correct sequence. 1. Fever rises. 2. Hematemesis. 3. Clotting abnormalities. 4. Vascular collapse. 5. Coma. A) 1, 2, 5, 4, 3 B) 1, 2, 3, 4, 5 C) 2, 3, 1, 4, 5 D) 3, 1, 2, 5, 4

Ans: B Feedback: Recovery from acute toxic hepatitis is rapid if the hepatotoxin is identified early and removed or if exposure to the agent has been limited. Recovery is unlikely if there is a prolonged period between exposure and onset of symptoms. There are no effective antidotes. The fever rises; the patient becomes toxic and prostrated. Vomiting may be persistent, with the emesis containing blood. Clotting abnormalities may be severe, and hemorrhages may appear under the skin. The severe GI symptoms may lead to vascular collapse. Delirium, coma, and seizures develop, and within a few days the patient may die of fulminant hepatic failure unless he or she receives a liver transplant.

2. The nurse observes that the family members of a patient who was injured in an accident are blaming each other for the circumstances leading up to the accident. The nurse appropriately lets the family members express their feelings of responsibility, while explaining that there was probably little they could do to prevent the injury. In what stage of crisis is this family? A) Anxiety and denial B) Remorse and guilt C) Anger D) Grief

Ans: B Feedback: Remorse and guilt are natural processes of the stages of a crisis and should be facilitated for the family members to process the crisis. The family's sense of blame and responsibility are more suggestive of guilt than anger, grief, or anxiety.

9. A patient with renal failure secondary to diabetic nephropathy has been admitted to the medical unit. What is the most life-threatening effect of renal failure for which the nurse should monitor the patient? A) Accumulation of wastes B) Retention of potassium C) Depletion of calcium D) Lack of BP control

Ans: B Feedback: Retention of potassium is the most life-threatening effect of renal failure. Aldosterone causes the kidney to excrete potassium, in contrast to aldosterone's effects on sodium described previously. Acidbase balance, the amount of dietary potassium intake, and the flow rate of the filtrate in the distal tubule also influence the amount of potassium secreted into the urine. Hypocalcemia, the accumulation of wastes, and lack of BP control are complications associated with renal failure, but do not have same level of threat to the patient's well-being as hyperkalemia.

21. A patient with recurrent urinary tract infections has just undergone a cystoscopy and complains of slight hematuria during the first void after the procedure. What is the nurse's most appropriate action? A) Administer a STAT dose of vitamin K, as ordered. B) Reassure the patient that this is not unexpected and then monitor the patient for further bleeding. C) Promptly inform the physician of this assessment finding. D) Position the patient supine and insert a Foley catheter, as ordered.

Ans: B Feedback: Some burning on voiding, blood-tinged urine, and urinary frequency from trauma to the mucous membranes can be expected after cystoscopy. The nurse should explain this to the patient and ensure that the bleeding resolves. No clear need exists to report this finding and it does not warrant insertion of a Foley catheter or vitamin K administration.

7. A football player is thought to have sustained an injury to his kidneys from being tackled from behind. The ER nurse caring for the patient reviews the initial orders written by the physician and notes that an order to collect all voided urine and send it to the laboratory for analysis. The nurse understands that this nursing intervention is important for what reason? A) Hematuria is the most common manifestation of renal trauma and blood losses may be microscopic, so laboratory analysis is essential. B) Intake and output calculations are essential and the laboratory will calculate the precise urine output produced by this patient. C) A creatinine clearance study may be ordered at a later time and the laboratory will hold all urine until it is determined if the test will be necessary. D) There is great concern about electrolyte imbalances and the laboratory will monitor the urine for changes in potassium and sodium concentrations.

Ans: A Feedback: Hematuria is the most common manifestation of renal trauma; its presence after trauma suggests renal injury. Hematuria may not occur, or it may be detectable only on microscopic examination. All urine should be saved and sent to the laboratory for analysis to detect RBCs and to evaluate the course of bleeding. Measuring intake and output is not a function of the laboratory. The laboratory does not save urine to test creatinine clearance at a later time. The laboratory does not monitor the urine for sodium or potassium concentrations.

36. A patient is receiving the first of two ordered units of PRBCs. Shortly after the initiation of the transfusion, the patient complains of chills and experiences a sharp increase in temperature. What is the nurse's priority action? A) Position the patient in high Fowler's. B) Discontinue the transfusion. C) Auscultate the patient's lungs. D) Obtain a blood specimen from the patient.

Ans: B Feedback: Stopping the transfusion is the first step in any suspected transfusion reaction. This must precede other assessments and interventions, including repositioning, chest auscultation, and collecting specimens.

9. The nurse coming on shift on the medical unit is taking a report on four patients. What patient does the nurse know is at the greatest risk of developing ESKD? A) A patient with a history of polycystic kidney disease B) A patient with diabetes mellitus and poorly controlled hypertension C) A patient who is morbidly obese with a history of vascular disorders D) A patient with severe chronic obstructive pulmonary disease

Ans: B Feedback: Systemic diseases, such as diabetes mellitus (leading cause); hypertension; chronic glomerulonephritis; pyelonephritis; obstruction of the urinary tract; hereditary lesions, such as in polycystic kidney disease; vascular disorders; infections; medications; or toxic agents may cause ESKD. A patient with more than one of these risk factors is at the greatest risk for developing ESKD. Therefore, the patient with diabetes and hypertension is likely at highest risk for ESKD.

13. A 45-year-old man with diabetic nephropathy has ESKD and is starting dialysis. "What should the nurse teach the patient about hemodialysis? A) "Hemodialysis is a treatment option that is usually required three times a week." B) "Hemodialysis is a program that will require you to commit to daily treatment." C) "This will require you to have surgery and a catheter will need to be inserted into your abdomen." D) "Hemodialysis is a treatment that is used for a few months until your kidney heals and starts to produce urine again."

Ans: A Feedback: Hemodialysis is the most commonly used method of dialysis. Patients receiving hemodialysis must undergo treatment for the rest of their lives or until they undergo successful kidney transplantation. Treatments usually occur three times a week for at least 3 to 4 hours per treatment.

18. A nurse is reviewing the trend of a patients scores on the Glasgow Coma Scale (GCS). This allows the nurse to gauge what aspect of the patients status? A) Reflex activity B) Level of consciousness C) Cognitive ability D) Sensory involvement

Ans: B Feedback: The Glasgow Coma Scale (GCS) examines three responses related to LOC: eye opening, best verbal response, and best motor response.

7. A patient is admitted to the ED with suspected alcohol intoxication. The ED nurse is aware of the need to assess for conditions that can mimic acute alcohol intoxication. In light of this need, the nurse should perform what action? A) Check the patient's blood glucose level. B) Assess for a documented history of major depression. C) Determine whether the patient has ingested a corrosive substance. D) Arrange for assessment of serum potassium levels.

Ans: A Feedback: Hypoglycemia can mimic alcohol intoxication and should be assessed in a patient suspected of alcohol intoxication. Potassium imbalances, depression, and poison ingestion are not noted to mimic the characteristic signs and symptoms of alcohol intoxication.

4. A patients abdominal ultrasound indicates cholelithiasis. When the nurse is reviewing the patients laboratory studies, what finding is most closely associated with this diagnosis? A) Increased bilirubin B) Decreased serum cholesterol C) Increased blood urea nitrogen (BUN) D) Decreased serum alkaline phosphatase level

Ans: A Feedback: If the flow of blood is impeded, bilirubin, a pigment derived from the breakdown of red blood cells, does not enter the intestines. As a result, bilirubin levels in the blood increase. Cholesterol, BUN, and alkaline phosphatase levels are not typically affected.

35. The nurse is reviewing the electronic health record of a patient with a history of incontinence. The nurse reads that the physician assessed the patient's deep tendon reflexes. What condition of the urinary/renal system does this assessment address? A) Renal calculi B) Bladder dysfunction C) Benign prostatic hyperplasia (BPH) D) Recurrent urinary tract infections (UTIs)

Ans: B Feedback: The deep tendon reflexes of the knee are examined for quality and symmetry. This is an important part of testing for neurologic causes of bladder dysfunction, because the sacral area, which innervates the lower extremities, is in the same peripheral nerve area responsible for urinary continence. Neurologic function does not directly influence the course of renal calculi, BPH or UTIs.

8. A patient has sustained a severe burn injury and is thought to have an impaired intestinal mucosal barrier. Since this patient is considered at an increased risk for infection, what intervention will best assist in avoiding increased intestinal permeability and prevent early endotoxin translocation? A) Early enteral feeding B) Administration of prophylactic antibiotics C) Bowel cleansing procedures D) Administration of stool softeners

Ans: A Feedback: If the intestinal mucosa receives some type of protection against permeability change, infection could be avoided. Early enteral feeding is one step to help avoid this increased intestinal permeability and prevent early endotoxin translocation. Antibiotics are seldom prescribed prophylactically because of the risk of promoting resistant strains of bacteria. A bowel cleansing procedure would not be ordered for this patient. The administration of stool softeners would not assist in avoiding increased intestinal permeability and prevent early endotoxin translocation.

15. A patient with a liver mass is undergoing a percutaneous liver biopsy. What action should the nurse perform when assisting with this procedure? A) Position the patient on the right side with a pillow under the costal margin after the procedure. B) Administer 1 unit of albumin 90 minutes before the procedure as ordered. C) Administer at least 1 unit of packed red blood cells as ordered the day before the scheduled procedure. D) Confirm that the patients electrolyte levels have been assessed prior to the procedure.

Ans: A Feedback: Immediately after a percutaneous liver biopsy, assist the patient to turn onto the right side and place a pillow under the costal margin. Prior administration of albumin or PRBCs is unnecessary. Coagulation tests should be performed, but electrolyte analysis is not necessary.

23. A hospital's emergency operations plan has been enacted following an industrial accident. While one nurse performs the initial triage, what should other emergency medical services personnel do? A) Perform life-saving measures. B) Classify patients according to acuity. C) Provide health promotion education. D) Modify the emergency operations plan.

Ans: A Feedback: In an emergency, patients are immediately tagged and transported or given life-saving interventions. One person performs the initial triage while other emergency medical services (EMS) personnel perform life-saving measures and transport patients. Health promotion is not a priority during the acute stage of the crisis. Classifying patients is the task of the triage nurse. EMS personnel prioritize life-saving measures; they do not modify the operations plan.

38. A nurse on a solid organ transplant unit is planning the care of a patient who will soon be admitted upon immediate recovery following liver transplantation. What aspect of nursing care is the nurses priority? A) Implementation of infection-control measures B) Close monitoring of skin integrity and color C) Frequent assessment of the patients psychosocial status D) Administration of antiretroviral medications

Ans: A Feedback: Infection control is paramount following liver transplantation. This is a priority over skin integrity and psychosocial status, even though these are valid areas of assessment and intervention. Antiretrovirals are not indicated.

14. A patient is being discharged after a liver transplant and the nurse is performing discharge education. When planning this patients continuing care, the nurse should prioritize which of the following risk diagnoses? A) Risk for Infection Related to Immunosuppressant Use B) Risk for Injury Related to Decreased Hemostasis C) Risk for Unstable Blood Glucose Related to Impaired Gluconeogenesis D) Risk for Contamination Related to Accumulation of Ammonia

Ans: A Feedback: Infection is the leading cause of death after liver transplantation. Pulmonary and fungal infections are common; susceptibility to infection is increased by the immunosuppressive therapy that is needed to prevent rejection. This risk exceeds the threats of injury and unstable blood glucose. The diagnosis of Risk for Contamination relates to environmental toxin exposure.

15. An older adult client is exhibiting many of the characteristic signs and symptoms of iron deficiency. In addition to a complete blood count, what diagnostic assessment should the nurse anticipate? A) Stool for occult blood B) Bone marrow biopsy C) Lumbar puncture D) Urinalysis

Ans: A Feedback: Iron deficiency in the adult generally indicates blood loss (e.g., from bleeding in the GI tract or heavy menstrual flow). Bleeding in the GI tract can be preliminarily identified by testing stool for the presence of blood. A bone marrow biopsy would not be undertaken for the sole purpose of investigating an iron deficiency. Lumbar puncture and urinalysis would not be clinically relevant.

37. A patient is brought to the ED by two police officers. The patient was found unconscious on the sidewalk, with his face and hands covered in blood. At present, the patient is verbally abusive and is fighting the staff in the ED, but appears medically stable. The decision is made to place the patient in restraints. What action should the nurse perform when the patient is restrained? A) Frequently assess the patient's skin integrity. B) Inform the patient that he is likely to be charged with assault. C) Avoid interacting with the patient until the restraints are removed. D) Take the opportunity to perform a full physical assessment.

Ans: A Feedback: It is important to assess skin integrity when physical restraints are used. Criminal charges are not the responsibility of the nurse and the nurse should still interact with the patient. A full physical assessment, however, would likely be delayed until the patient is not combative.

17. A patient has developed hepatic encephalopathy secondary to cirrhosis and is receiving care on the medical unit. The patients current medication regimen includes lactulose (Cephulac) four times daily. What desired outcome should the nurse relate to this pharmacologic intervention? A) Two to 3 soft bowel movements daily B) Significant increase in appetite and food intake C) Absence of nausea and vomiting D) Absence of blood or mucus in stool

Ans: A Feedback: Lactulose (Cephulac) is administered to reduce serum ammonia levels. Two or three soft stools per day are desirable; this indicates that lactulose is performing as intended. Lactulose does not address the patients appetite, symptoms of nausea and vomiting, or the development of blood and mucus in the stool.

6. The nurse is working with a patient who is newly diagnosed with MS. What basic information should the nurse provide to the patient? A) MS is a progressive demyelinating disease of the nervous system. B) MS usually occurs more frequently in men. C) MS typically has an acute onset. D) MS is sometimes caused by a bacterial infection.

Ans: A Feedback: MS is a chronic, degenerative, progressive disease of the central nervous system, characterized by the occurrence of small patches of demyelination in the brain and spinal cord. The cause of MS is not known, and the disease affects twice as many women as men.

37. A patient with ongoing back pain, nausea, and abdominal bloating has been diagnosed with cholecystitis secondary to gallstones. The nurse should anticipate that the patient will undergo what intervention? A) Laparoscopic cholecystectomy B) Methyl tertiary butyl ether (MTBE) infusion C) Intracorporeal lithotripsy D) Extracorporeal shock wave therapy (ESWL)

Ans: A Feedback: Most of the nonsurgical approaches, including lithotripsy and dissolution of gallstones, provide only temporary solutions to gallstone problems and are infrequently used in the United States. Cholecystectomy is the preferred treatment.

13. A patient has been diagnosed with advanced stage breast cancer and will soon begin aggressive treatment. What assessment findings would most strongly suggest that the patient may have developed liver metastases? A) Persistent fever and cognitive changes B) Abdominal pain and hepatomegaly C) Peripheral edema unresponsive to diuresis D) Spontaneous bleeding and jaundice

Ans: B Feedback: The early manifestations of malignancy of the liver include paina continuous dull ache in the right upper quadrant, epigastrium, or back. Weight loss, loss of strength, anorexia, and anemia may also occur. The liver may be enlarged and irregular on palpation. Jaundice is present only if the larger bile ducts are occluded by the pressure of malignant nodules in the hilum of the liver. Fever, cognitive changes, peripheral edema, and bleeding are atypical signs.

22. The physician has ordered a fluid deprivation test for a patient suspected of having diabetes insipidus. During the test, the nurse should prioritize what assessments? A) Temperature and oxygen saturation B) Heart rate and BP C) Breath sounds and bowel sounds D) Color, warmth, movement, and sensation of extremities

Ans: B Feedback: The fluid deprivation test is carried out by withholding fluids for 8 to 12 hours or until 3% to 5% of the body weight is lost. The patients condition needs to be monitored frequently during the test, and the test is terminated if tachycardia, excessive weight loss, or hypotension develops. Consequently, BP and heart rate monitoring are priorities over the other listed assessments.

29. The nurse is teaching a patient with Guillain-Barr syndrome about the disease. The patient asks how he can ever recover if demyelination of his nerves is occurring. What would be the nurses best response? A) Guillain-Barr spares the Schwann cell, which allows for remyelination in the recovery phase of the disease. B) In Guillain-Barr, Schwann cells replicate themselves before the disease destroys them, so remyelination is possible. C) I know you understand that nerve cells do not remyelinate, so the physician is the best one to answer your question. D) For some reason, in Guillain-Barr, Schwann cells become activated and take over the remyelination process.

Ans: A Feedback: Myelin is a complex substance that covers nerves, providing insulation and speeding the conduction of impulses from the cell body to the dendrites. The cell that produces myelin in the peripheral nervous system is the Schwann cell. In Guillain-Barr syndrome, the Schwann cell is spared, allowing for remyelination in the recovery phase of the disease. The nurse should avoid downplaying the patients concerns by wholly deferring to the physician.

15. A patient is being treated in the ED following a terrorist attack. The patient is experiencing visual disturbances, nausea, vomiting, and behavioral changes. The nurse suspects this patient has been exposed to what chemical agent? A) Nerve agent B) Pulmonary agent C) Vesicant D) Blood agent

Ans: A Feedback: Nerve agent exposure results in visual disturbances, nausea and vomiting, forgetfulness, irritability, and impaired judgment. This presentation is not suggestive of vesicants, pulmonary agents, or blood agents.

38. A patient has been exposed to a nerve agent in a biochemical terrorist attack. This type of agent bonds with acetylcholinesterase, so that acetylcholine is not inactivated. What is the pathologic effect of this type of agent? A) Hyperstimulation of the nerve endings B) Temporary deactivation of the nerve endings C) Binding of the nerve endings D) Destruction of the nerve endings

Ans: A Feedback: Nerve agents can be inhaled or absorbed percutaneously or subcutaneously. These agents bond with acetylcholinesterase, so that acetylcholine is not inactivated; the adverse result is continuous stimulation (hyperstimulation) of the nerve endings. Nerve endings are not deactivated, bound, or destroyed.

26. A nurse is participating in the planning of a hospital's emergency operations plan. The nurse is aware of the potential for ethical dilemmas during a disaster or other emergency. Ethical dilemmas in these contexts are best addressed by which of the following actions? A) Having an ethical framework in place prior to an emergency B) Allowing staff to provide care anonymously during an emergency C) Assuring staff that they are not legally accountable for care provided during an emergency D) Teaching staff that principles of ethics do not apply in an emergency situation

Ans: A Feedback: Nurses can plan for the ethical dilemmas they may face during disasters by establishing a framework for evaluating ethical questions before they arise and by identifying and exploring possible responses to difficult clinical situations. Ethical principles do not become wholly irrelevant in emergencies. Care cannot be given anonymously and accountability for practice always exists, even in an emergency.

26. A patient with a diagnosis of respiratory acidosis is experiencing renal compensation. What function does the kidney perform to assist in restoring acidbase balance? A) Sequestering free hydrogen ions in the nephrons B) Returning bicarbonate to the body's circulation C) Returning acid to the body's circulation D) Excreting bicarbonate in the urine

Ans: B Feedback: The kidney performs two major functions to assist in acidbase balance. The first is to reabsorb and return to the body's circulation any bicarbonate from the urinary filtrate; the second is to excrete acid in the urine. Retaining bicarbonate will counteract an acidotic state. The nephrons do not sequester free hydrogen ions.

4. A patient has been brought to the ED with multiple trauma after a motor vehicle accident. After immediate threats to life have been addressed, the nurse and trauma team should take what action? A) Perform a rapid physical assessment. B) Initiate health education. C) Perform diagnostic imaging. D) Establish the circumstances of the accident.

Ans: A Feedback: Once immediate threats to life have been corrected, a rapid physical examination is done to identify injuries and priorities of treatment. Health education is initiated later in the care process and diagnostic imaging would take place after a rapid physical assessment. It is not the care team's responsibility to determine the circumstances of the accident.

35. The ED nurse admitting a patient with a history of depression is screening the patient for suicide risk. What assessment question should the nurse ask when screening the patient? A) "How would you describe your mood over the past few days?" B) "Have you ever thought about taking your own life?" C) "How do you think that your life is most likely to end?" D) "How would you rate the severity of your depression right now on a 10-point scale?"

Ans: B Feedback: The nurse should address the patient's possible plans for suicide in a direct yet empathic manner. The nurse should avoid oblique or indirect references to suicide and should not limit questions to the patient's depression.

14. A male patient with multiple injuries is brought to the ED by ambulance. He has had his airway stabilized and is breathing on his own. The ED nurse does not see any active bleeding, but should suspect internal hemorrhage based on what finding? A) Absence of bruising at contusion sites B) Rapid pulse and decreased capillary refill C) Increased BP with narrowed pulse pressure D) Sudden diaphoresis

Ans: B Feedback: The nurse would anticipate that the pulse would increase and BP would decrease. Urine output would also decrease. An absence of bruising and the presence of diaphoresis would not suggest internal hemorrhage.

36. A patient with a cholelithiasis has been scheduled for a laparoscopic cholecystectomy. Why is laparoscopic cholecystectomy preferred by surgeons over an open procedure? A) Laparoscopic cholecystectomy poses fewer surgical risks than an open procedure. B) Laparoscopic cholecystectomy can be performed in a clinic setting, while an open procedure requires an OR. C) A laparoscopic approach allows for the removal of the entire gallbladder. D) A laparoscopic approach can be performed under conscious sedation.

Ans: A Feedback: Open surgery has largely been replaced by laparoscopic cholecystectomy (removal of the gallbladder through a small incision through the umbilicus). As a result, surgical risks have decreased, along with the length of hospital stay and the long recovery period required after standard surgical cholecystectomy. Both approaches allow for removal of the entire gallbladder and must be performed under general anesthetic in an operating theater.

31. A patient has been diagnosed with acute pancreatitis. The nurse is addressing the diagnosis of Acute Pain Related to Pancreatitis. What pharmacologic intervention is most likely to be ordered for this patient? A) Oral oxycodone B) IV hydromorphone (Dilaudid) C) IM meperidine (Demerol) D) Oral naproxen (Aleve)

Ans: B Feedback: The pain of acute pancreatitis is often very severe and pain relief may require parenteral opioids such as morphine, fentanyl (Sublimaze), or hydromorphone (Dilaudid). There is no clinical evidence to support the use of meperidine for pain relief in pancreatitis. Opioids are preferred over NSAIDs.

25. An emergency department nurse has just received a patient with burn injuries brought in by ambulance. The paramedics have started a large-bore IV and covered the burn in cool towels. The burn is estimated as covering 24% of the patients body. How should the nurse best address the pathophysiologic changes resulting from major burns during the initial burn-shock period? A) Administer IV fluids B) Administer broad-spectrum antibiotics C) Administer IV potassium chloride D) Administer packed red blood cells

Ans: A Feedback: Pathophysiologic changes resulting from major burns during the initial burn-shock period include massive fluid losses. Addressing these losses is a major priority in the initial phase of treatment. Antibiotics and PRBCs are not normally administered. Potassium chloride would exacerbate the patients hyperkalemia.

37. A patient with a new diagnosis of amyotrophic lateral sclerosis (ALS) is overwhelmed by his diagnosis and the known complications of the disease. How can the patient best make known his wishes for care as his disease progresses? A) Prepare an advance directive. B) Designate a most responsible physician (MRP) early in the course of the disease. C) Collaborate with representatives from the Amyotrophic Lateral Sclerosis Association. D) Ensure that witnesses are present when he provides instruction.

Ans: A Feedback: Patients with ALS are encouraged to complete an advance directive or living will to preserve their autonomy in decision making. None of the other listed actions constitutes a legally binding statement of end-of-life care.

14. A patient with ESKD receives continuous ambulatory peritoneal dialysis. The nurse observes that the dialysate drainage fluid is cloudy. What is the nurse's most appropriate action? A) Inform the physician and assess the patient for signs of infection. B) Flush the peritoneal catheter with normal saline. C) Remove the catheter promptly and have the catheter tip cultured. D) Administer a bolus of IV normal saline as ordered.

Ans: A Feedback: Peritonitis is the most common and serious complication of peritoneal dialysis. The first sign of peritonitis is cloudy dialysate drainage fluid, so prompt reporting to the primary care provider and rapid assessment for other signs of infection are warranted. Administration of an IV bolus is not necessary or appropriate and the physician would determine whether removal of the catheter is required. Flushing the catheter does not address the risk for infection.

26. The nurse's brief review of a patient's electronic health record indicates that the patient regularly undergoes therapeutic phlebotomy. Which of the following rationales for this procedure is most plausible? A) The patient may chronically produce excess red blood cells. B) The patient may frequently experience a low relative plasma volume. C) The patient may have impaired stem cell function. D) The patient may previously have undergone bone marrow biopsy.

Ans: A Feedback: Persistently elevated hematocrit is an indication for therapeutic phlebotomy. It is not used to address excess or deficient plasma volume and is not related to stem cell function. Bone marrow biopsy is not an indication for therapeutic phlebotomy.

2. A man suffers a leg wound which causes minor blood loss. As a result of bleeding, the process of primary hemostasis is activated. What occurs in primary hemostasis? A) Severed blood vessels constrict. B) Thromboplastin is released. C) Prothrombin is converted to thrombin. D) Fibrin is lysed.

Ans: A Feedback: Primary hemostasis involves the severed vessel constricting and platelets collecting at the injury site. Secondary hemostasis occurs when thromboplastin is released, prothrombin converts to thrombin, and fibrin is lysed.

34. The ED staff has been notified of the imminent arrival of a patient who has been exposed to chlorine. The nurse should anticipate the need to address what nursing diagnosis? A) Impaired gas exchange B) Decreased cardiac output C) Chronic pain D) Excess fluid volume

Ans: A Feedback: Pulmonary agents, such as phosgene and chlorine, destroy the pulmonary membrane that separates the alveolus from the capillary bed, disrupting alveolar-capillary oxygen transport mechanisms. Capillary leakage results in fluid-filled alveoli and gas exchange ceases to occur. Pain is likely, but is acute rather than chronic. Fluid volume excess is unlikely to be a priority diagnosis and cardiac output will be secondarily affected by the pulmonary effects.

35. The nursing supervisor at the local hospital is advised that your hospital will be receiving multiple trauma victims from a blast that occurred at a local manufacturing plant. The paramedics call in a victim of the blast with injuries including a head injury and hemorrhage. What phase of blast injury should the nurse expect to treat in this patient? A) Primary phase B) Secondary phase C) Tertiary phase D) Quaternary phase

Ans: A Feedback: Pulmonary barotraumas, including pulmonary contusions; head injuries, including concussion, other severe brain injuries; tympanic membrane rupture, middle ear injury; abdominal hollow organ perforation; and hemorrhage are all injuries that can occur in the primary phase of a blast. These particular injuries are not characteristic of the subsequent phases.

20. The nurse is developing a plan of care for a patient with Guillain-Barr syndrome. Which of the following interventions should the nurse prioritize for this patient? A) Using the incentive spirometer as prescribed B) Maintaining the patient on bed rest C) Providing aids to compensate for loss of vision D) Assessing frequently for loss of cognitive function

Ans: A Feedback: Respiratory function can be maximized with incentive spirometry and chest physiotherapy. Nursing interventions toward enhancing physical mobility should be utilized. Nursing interventions are aimed at preventing a deep vein thrombosis. Guillain-Barr syndrome does not affect cognitive function or vision.

36. What should the nurse teach a patient on corticosteroid therapy in order to reduce the patients risk of adrenal insufficiency? A) Take the medication late in the day to mimic the bodys natural rhythms. B) Always have enough medication on hand to avoid running out. C) Skip up to 2 doses in cases of illness involving nausea. D) Take up to 1 extra dose per day during times of stress.

Ans: B Feedback: The patient and family should be informed that acute adrenal insufficiency and underlying symptoms will recur if corticosteroid therapy is stopped abruptly without medical supervision. The patient should be instructed to have an adequate supply of the corticosteroid medication always available to avoid running out. Doses should not be skipped or added without explicit instructions to do so. Corticosteroids should normally be taken in the morning to mimic natural rhythms.

28. A patient is brought to the renal unit from the PACU status post resection of a renal tumor. Which of the following nursing actions should the nurse prioritize in the care of this patient? A) Increasing oral intake B) Managing postoperative pain C) Managing dialysis D) Increasing mobility

Ans: B Feedback: The patient requires frequent analgesia during the postoperative period and assistance with turning, coughing, use of incentive spirometry, and deep breathing to prevent atelectasis and other pulmonary complications. Increasing oral intake and mobility are not priority nursing actions in the immediate postoperative care of this patient. Dialysis is not necessary following kidney surgery.

38. The nurse is caring for a patient after kidney surgery. The nurse is aware that bleeding is a major complication of kidney surgery and that if it goes undetected and untreated can result in hypovolemia and hemorrhagic shock in the patient. When assessing for bleeding, what assessment parameter should the nurse evaluate? A) Oral intake B) Pain intensity C) Level of consciousness D) Radiation of pain

Ans: C Feedback: Bleeding is a major complication of kidney surgery. If undetected and untreated, this can result in hypovolemia and hemorrhagic shock. The nurse's role is to observe for these complications, to report their signs and symptoms, and to administer prescribed parenteral fluids and blood and blood components. Monitoring of vital signs, skin condition, the urinary drainage system, the surgical incision, and the level of consciousness is necessary to detect evidence of bleeding, decreased circulating blood, and fluid volume and cardiac output. Bleeding is not normally evidenced by changes in pain or oral intake.

1. A patient with a hematologic disorder asks the nurse how the body forms blood cells. The nurse should describe a process that takes place where? A) In the spleen B) In the kidneys C) In the bone marrow D) In the liver

Ans: C Feedback: Bone marrow is the primary site for hematopoiesis. The liver and spleen may be involved during embryonic development or when marrow is destroyed. The kidneys release erythropoietin, which stimulates the marrow to increase production of red blood cells (RBCs). However, blood cells are not primarily formed in the spleen, kidneys, or liver.

32. A patient has just been diagnosed with chronic pancreatitis. The patient is underweight and in severe pain and diagnostic testing indicates that over 80% of the patients pancreas has been destroyed. The patient asks the nurse why the diagnosis was not made earlier in the disease process. What would be the nurses best response? A) The symptoms of pancreatitis mimic those of much less serious illnesses. B) Your body doesnt require pancreatic function until it is under great stress, so it is easy to go unnoticed. C) Chronic pancreatitis often goes undetected until a large majority of pancreatic function is lost. D) Its likely that your other organs were compensating for your decreased pancreatic function.

Ans: C Feedback: By the time symptoms occur in chronic pancreatitis, approximately 90% of normal acinar cell function (exocrine function) has been lost. Late detection is not usually attributable to the vagueness of symptoms. The pancreas contributes continually to homeostasis and other organs are unable to perform its physiologic functions.

20. A patient is admitted to the ICU after a motor vehicle accident. On the second day of the hospital admission, the patient develops acute kidney injury. The patient is hemodynamically unstable, but renal replacement therapy is needed to manage the patient's hypervolemia and hyperkalemia. Which of the following therapies will the patient's hemodynamic status best tolerate? A) Hemodialysis B) Peritoneal dialysis C) Continuous venovenous hemodialysis (CVVHD) D) Plasmapheresis

Ans: C Feedback: CVVHD facilitates the removal of uremic toxins and fluid. The hemodynamic effects of CVVHD are usually mild in comparison to hemodialysis, so CVVHD is best tolerated by an unstable patient. Peritoneal dialysis is not the best choice, as the patient may have sustained abdominal injuries during the accident and catheter placement would be risky. Plasmapheresis does not achieve fluid removal and electrolyte balance.

25. A community health nurse is caring for a patient whose multiple health problems include chronic pancreatitis. During the most recent visit, the nurse notes that the patient is experiencing severe abdominal pain and has vomited 3 times in the past several hours. What is the nurses most appropriate action? A) Administer a PRN dose of pancreatic enzymes as ordered. B) Teach the patient about the importance of abstaining from alcohol. C) Arrange for the patient to be transported to the hospital. D) Insert an NG tube, if available, and stay with the patient.

Ans: C Feedback: Chronic pancreatitis is characterized by recurring attacks of severe upper abdominal and back pain, accompanied by vomiting. The onset of these acute symptoms warrants hospital treatment. Pancreatic enzymes are not indicated and an NG tube would not be inserted in the home setting. Patient education is a later priority that may or may not be relevant.

20. The nurse is assessing a patient diagnosed with Graves disease. What physical characteristics of Graves disease would the nurse expect to find? A) Hair loss B) Moon face C) Bulging eyes D) Fatigue

Ans: C Feedback: Clinical manifestations of the endocrine disorder Graves disease include exophthalmos (bulging eyes) and fine tremor in the hands. Graves disease is not associated with hair loss, a moon face, or fatigue.

29. A 30 year-old female patient has been diagnosed with Cushing syndrome. What psychosocial nursing diagnosis should the nurse most likely prioritize when planning the patients care? A) Decisional conflict related to treatment options B) Spiritual distress related to changes in cognitive function C) Disturbed body image related to changes in physical appearance D) Powerlessness related to disease progression

Ans: C Feedback: Cushing syndrome causes characteristic physical changes that are likely to result in disturbed body image. Decisional conflict and powerless may exist, but disturbed body image is more likely to be present. Cognitive changes take place in patients with Cushing syndrome, but these may or may not cause spiritual distress.

40. A 23-year-old woman is brought to the ED complaining of stomach cramps, nausea, vomiting, and diarrhea. The care team suspects food poisoning. What is the key to treatment in food poisoning? A) Administering IV antibiotics B) Assessing immunization status C) Determining the source and type of food poisoning D) Determining if anyone else in the family is ill

Ans: C Feedback: Determining the source and type of food poisoning is essential to treatment, and is more important than determining other sick family members. Antibiotics are not normally indicated and immunizations are not relevant to diagnosis or treatment of food poisoning.

9. The nurse caring for a patient with Cushing syndrome is describing the dexamethasone suppression test scheduled for tomorrow. What does the nurse explain that this test will involve? A) Administration of dexamethasone orally, followed by a plasma cortisol level every hour for 3 hours B) Administration of dexamethasone IV, followed by an x-ray of the adrenal glands C) Administration of dexamethasone orally at 11 PM, and a plasma cortisol level at 8 AM the next morning D) Administration of dexamethasone intravenously, followed by a plasma cortisol level 3 hours after the drug is administered

Ans: C Feedback: Dexamethasone (1 mg) is administered orally at 11 PM, and a plasma cortisol level is obtained at 8 AM the next morning. This test can be performed on an outpatient basis and is the most widely used and sensitive screening test for diagnosis of pituitary and adrenal causes of Cushing syndrome.

31. A patient has experienced burns to his upper thighs and knees. Following the application of new wound dressings, the nurse should perform what nursing action? A) Instruct the patient to keep the wound site in a dependent position. B) Administer PRN analgesia as ordered. C) Assess the patients peripheral pulses distal to the dressing. D) Assist with passive range of motion exercises to set the new dressing.

Ans: C Feedback: Dressings can impede circulation if they are wrapped too tightly. The peripheral pulses must be checked frequently and burned extremities elevated. Dependent positioning does not need to be maintained. PRN analgesics should be administered prior to the dressing change. ROM exercises do not normally follow a dressing change.

25. A patient with multiple trauma is brought to the ED by ambulance after a fall while rock climbing. What is a responsibility of the ED nurse in this patient's care? A) Intubating the patient B) Notifying family members C) Ensuring IV access D) Delivering specimens to the laboratory

Ans: C Feedback: ED nursing responsibilities include ensuring airway and IV access. Nurses are not normally responsible for notifying family members. Nurses collect specimens, but are not responsible for their delivery. Physicians or other team members with specialized training intubate the patient.

34. A patient is admitted to the ICU with acute pancreatitis. The patients family asks what causes acute pancreatitis. The critical care nurse knows that a majority of patients with acute pancreatitis have what? A) Type 1 diabetes B) An impaired immune system C) Undiagnosed chronic pancreatitis D) An amylase deficiency

Ans: C Feedback: Eighty percent of patients with acute pancreatitis have biliary tract disease or a history of long-term alcohol abuse. These patients usually have had undiagnosed chronic pancreatitis before their first episode of acute pancreatitis. Diabetes, an impaired immune function, and amylase deficiency are not specific precursors to acute pancreatitis.

15. A nurse is teaching a patient with a partial-thickness wound how to wear his elastic pressure garment. How would the nurse instruct the patient to wear this garment? A) 4 to 6 hours a day for 6 months B) During waking hours for 2 to 3 months after the injury C) Continuously D) At night while sleeping for a year after the injury

Ans: C Feedback: Elastic pressure garments are worn continuously (i.e., 23 hours a day).

17. A patient is experiencing respiratory insufficiency and cannot maintain spontaneous respirations. The nurse suspects that the physician will perform which of the following actions? A) Insert an oropharyngeal airway. B) Perform the jaw thrust maneuver. C) Perform endotracheal intubation. D) Perform a cricothyroidotomy.

Ans: C Feedback: Endotracheal tubes are used in cases when the patient cannot be ventilated with an oropharyngeal airway, which is used in patients who are breathing spontaneously. The jaw thrust maneuver does not establish an airway and cricothyroidotomy would be performed as a last resort.

5. A nurse who provides care in a walk-in clinic assesses a wide range of individuals. The nurse should identify which of the following patients as having the highest risk for chronic pancreatitis? A) A 45-year-old obese woman with a high-fat diet B) An 18-year-old man who is a weekend binge drinker C) A 39-year-old man with chronic alcoholism D) A 51-year-old woman who smokes one-and-a-half packs of cigarettes per day

Ans: C Feedback: Excessive and prolonged consumption of alcohol accounts for approximately 70% to 80% of all cases of chronic pancreatitis.

22. A patient is complaining of genitourinary pain shortly after returning to the unit from a scheduled cystoscopy. What intervention should the nurse perform? A) Encourage mobilization. B) Apply topical lidocaine to the patient's meatus, as ordered. C) Apply moist heat to the patient's lower abdomen. D) Apply an ice pack to the patient's perineum.

Ans: C Feedback: Following cystoscopy, moist heat to the lower abdomen and warm sitz baths are helpful in relieving pain and relaxing the muscles. Ice, lidocaine, and mobilization are not recommended interventions.

4. A patient with Guillain-Barr syndrome has experienced a sharp decline in vital capacity. What is the nurses most appropriate action? A) Administer bronchodilators as ordered. B) Remind the patient of the importance of deep breathing and coughing exercises. C) Prepare to assist with intubation. D) Administer supplementary oxygen by nasal cannula.

Ans: C Feedback: For the patient with Guillain-Barr syndrome, mechanical ventilation is required if the vital capacity falls, making spontaneous breathing impossible and tissue oxygenation inadequate. Each of the other listed actions is likely insufficient to meet the patients oxygenation needs.

20. The nurse is caring for a patient who had a brush biopsy 12 hours ago. The presence of what assessment finding should prompt the nurse to notify the physician? A) Scant hematuria B) Renal colic C) Temperature 100.2°F orally D) Infiltration of the patient's intravenous catheter

Ans: C Feedback: Hematuria and renal colic are common and expected findings after the performance of a renal brush biopsy. The physician should be notified of the patient's body temperature, which likely indicates the onset of an infectious process. IV infiltration does not warrant notification of the primary care physician.

11. The nurse is caring for a patient in acute kidney injury. Which of the following complications would most clearly warrant the administration of polystyrene sulfonate (Kayexalate)? A) Hypernatremia B) Hypomagnesemia C) Hyperkalemia D) Hypercalcemia

Ans: C Feedback: Hyperkalemia, a common complication of acute kidney injury, is life-threatening if immediate action is not taken to reverse it. The administration of polystyrene sulfonate reduces serum potassium levels.

20. A patient with amyotrophic lateral sclerosis (ALS) is being visited by the home health nurse who is creating a care plan. What nursing diagnosis is most likely for a patient with this condition? A) Chronic confusion B) Impaired urinary elimination C) Impaired verbal communication D) Bowel incontinence

Ans: C Feedback: Impaired communication is an appropriate nursing diagnosis; the voice in patients with ALS assumes a nasal sound and articulation becomes so disrupted that speech is unintelligible. Intellectual function is marginally impaired in patients with late ALS. Usually, the anal and bladder sphincters are intact because the spinal nerves that control muscles of the rectum and urinary bladder are not affected.

26. A student nurse is caring for a patient who has a diagnosis of acute pancreatitis and who is receiving parenteral nutrition. The student should prioritize which of the following assessments? A) Fluid output B) Oral intake C) Blood glucose levels D) BUN and creatinine levels

Ans: C Feedback: In addition to administering enteral or parenteral nutrition, the nurse monitors serum glucose levels every 4 to 6 hours. Output should be monitored but in most cases it is not more important than serum glucose levels. A patient on parenteral nutrition would have no oral intake to monitor. Blood sugar levels are more likely to be unstable than indicators of renal function.

10. A group of disaster survivors is working with the critical incident stress management (CISM) team. Members of this team should be guided by what goal? A) Determining whether the incident was managed effectively B) Educating survivors on potential coping strategies for future disasters C) Providing individuals with education about recognizing stress reactions D) Determining if individuals responded appropriately during the incident

Ans: C Feedback: In defusing, patients are given information about recognizing stress reactions and how to deal with handling the stress they may experience. Debriefing involves asking patients about their current emotional coping and symptoms, following up, and identifying patients who require further assessment and assistance in dealing with the stress experienced. The CISM team does not focus primarily on the management of the incident or on providing skills for future incidents.

15. A patient is admitted to the neurologic ICU with a spinal cord injury. When assessing the patient the nurse notes there is a sudden depression of reflex activity in the spinal cord below the level of injury. What should the nurse suspect? A) Epidural hemorrhage B) Hypertensive emergency C) Spinal shock D) Hypovolemia

Ans: C Feedback: In spinal shock, the reflexes are absent, BP and heart rate fall, and respiratory failure can occur. Hypovolemia, hemorrhage, and hypertension do not cause this sudden change in neurologic function.

32. A patient suffering from blast lung has been admitted to the hospital and is exhibiting signs and symptoms of an air embolus. What is the nurse's most appropriate action? A) Place the patient in the Trendelenberg position. B) Assess the patient's airway and begin chest compressions. C) Position the patient in the prone, left lateral position. D) Encourage the patient to perform deep breathing and coughing exercises.

Ans: C Feedback: In the event of an air embolus, the patient should be placed immediately in the prone left lateral position to prevent migration of the embolus and will require emergent treatment in a hyperbaric chamber. Chest compressions, deep breathing, and coughing would exacerbate the patient's condition. Trendelenberg positioning is not recommended.

16. A patient has returned to the floor after having a thyroidectomy for thyroid cancer. The nurse knows that sometimes during thyroid surgery the parathyroid glands can be injured or removed. What laboratory finding may be an early indication of parathyroid gland injury or removal? A) Hyponatremia B) Hypophosphatemia C) Hypocalcemia D) Hypokalemia

Ans: C Feedback: Injury or removal of the parathyroid glands may produce a disturbance in calcium metabolism and result in a decline of calcium levels (hypocalcemia). As the blood calcium levels fall, hyperirritability of the nerves occurs, with spasms of the hands and feet and muscle twitching. This group of symptoms is known as tetany and must be reported to the physician immediately, because laryngospasm may occur and obstruct the airway. Hypophosphatemia, hyponatremia, and hypokalemia are not expected responses to parathyroid injury or removal. In fact, parathyroid removal or injury that results in hypocalcemia may lead to hyperphosphatemia.

29. The family of a patient in the ICU diagnosed with acute pancreatitis asks the nurse why the patient has been moved to an air bed. What would be the nurses best response? A) Air beds allow the care team to reposition her more easily while shes on bed rest. B) Air beds are far more comfortable than regular beds and shell likely have to be on bed rest a long time. C) The bed automatically moves, so shes less likely to develop pressure sores while shes in bed. D) The bed automatically moves, so she is likely to have less pain.

Ans: C Feedback: It is important to turn the patient every 2 hours; use of specialty beds may be indicated to prevent skin breakdown. The rationale for a specialty bed is not related to repositioning, comfort, or ease of movement.

31. A previously healthy adults sudden and precipitous decline in health has been attributed to fulminant hepatic failure, and the patient has been admitted to the intensive care unit. The nurse should be aware that the treatment of choice for this patient is what? A) IV administration of immune globulins B) Transfusion of packed red blood cells and fresh-frozen plasma (FFP) C) Liver transplantation D) Lobectomy

Ans: C Feedback: Liver transplantation carries the highest potential for the resolution of fulminant hepatic failure. This is preferred over other interventions, such as pharmacologic treatments, transfusions, and surgery.

40. A nurse is giving an educational class to members of the local disaster team. What should the nurse instruct members of the disaster team to do in a chemical bioterrorist attack? A) Cover their eyes. B) Put on a personal protective equipment mask. C) Stand up. D) Crawl to an exit.

Ans: C Feedback: Most chemicals are heavier than air, except for hydrogen cyanide. Therefore, in the presence of most chemicals, people should stand up to avoid heavy exposure because the chemical will sink toward the floor or ground. For this reason, covering their eyes, putting on a PPE mask, or crawling to an exit will not decrease exposure.

3. A patient with portal hypertension has been admitted to the medical floor. The nurse should prioritize which of the following assessments related to the manifestations of this health problem? A) Assessment of blood pressure and assessment for headaches and visual changes B) Assessments for signs and symptoms of venous thromboembolism C) Daily weights and abdominal girth measurement D) Blood glucose monitoring q4h

Ans: C Feedback: Obstruction to blood flow through the damaged liver results in increased blood pressure (portal hypertension) throughout the portal venous system. This can result in varices and ascites in the abdominal cavity. Assessments related to ascites are daily weights and abdominal girths. Portal hypertension is not synonymous with cardiovascular hypertension and does not create a risk for unstable blood glucose or VTE.

2. The nurse is caring for acutely ill patient. What assessment finding should prompt the nurse to inform the physician that the patient may be exhibiting signs of acute kidney injury (AKI)? A) The patient is complains of an inability to initiate voiding. B) The patient's urine is cloudy with a foul odor. C) The patient's average urine output has been 10 mL/hr for several hours. D) The patient complains of acute flank pain.

Ans: C Feedback: Oliguria (<500 mL/d of urine) is the most common clinical situation seen in AKI. Flank pain and inability to initiate voiding are not characteristic of AKI. Cloudy, foul-smelling urine is suggestive of a urinary tract infection.

19. A 33-year-old patient presents at the clinic with complaints of weakness, incoordination, dizziness, and loss of balance. The patient is hospitalized and diagnosed with MS. What sign or symptom, revealed during the initial assessment, is typical of MS? A) Diplopia, history of increased fatigue, and decreased or absent deep tendon reflexes B) Flexor spasm, clonus, and negative Babinskis reflex C) Blurred vision, intention tremor, and urinary hesitancy D) Hyperactive abdominal reflexes and history of unsteady gait and episodic paresthesia in both legs

Ans: C Feedback: Optic neuritis, leading to blurred vision, is a common early sign of MS, as is intention tremor (tremor when performing an activity). Nerve damage can cause urinary hesitancy. In MS, deep tendon reflexes are increased or hyperactive. A positive Babinskis reflex is found in MS. Abdominal reflexes are absent with MS.

17. The nurse's review of a patient's most recent blood work reveals a significant increase in the number of band cells. The nurse's subsequent assessment should focus on which of the following? A) Respiratory function B) Evidence of decreased tissue perfusion C) Signs and symptoms of infection D) Recent changes in activity tolerance

Ans: C Feedback: Ordinarily, band cells account for only a small percentage of circulating granulocytes, although their percentage can increase greatly under conditions in which neutrophil production increases, such as infection. This finding is not suggestive of problems with oxygenation and subsequent activity intolerance.

31. Results of a patient's 24-hour urine sample indicate osmolality of 510 mOsm/kg, which is within reference range. What conclusion can the nurse draw from this assessment finding? A) The patient's kidneys are capable of maintaining acidbase balance. B) The patient's kidneys reabsorb most of the potassium that the patient ingests. C) The patient's kidneys can produce sufficiently concentrated urine. D) The patient's kidneys are producing sufficient erythropoietin.

Ans: C Feedback: Osmolality is the most accurate measurement of the kidney's ability to dilute and concentrate urine. Osmolality is not a direct indicator of renal function as it relates to erythropoietin synthesis or maintenance of acid-base balance. It does not indicate the maintenance of healthy levels of potassium, the vast majority of which is excreted.

30. A patient is receiving care in the intensive care unit for acute pancreatitis. The nurse is aware that pancreatic necrosis is a major cause of morbidity and mortality in patients with acute pancreatitis. Consequently, the nurse should assess for what signs or symptoms of this complication? A) Sudden increase in random blood glucose readings B) Increased abdominal girth accompanied by decreased level of consciousness C) Fever, increased heart rate and decreased blood pressure D) Abdominal pain unresponsive to analgesics

Ans: C Feedback: Pancreatic necrosis is a major cause of morbidity and mortality in patients with acute pancreatitis because of resulting hemorrhage, septic shock, and multiple organ dysfunction syndrome (MODS). Signs of shock would include hypotension, tachycardia and fever. Each of the other listed changes in status warrants intervention, but none is clearly suggestive of an onset of pancreatic necrosis.

39. The nurse recognizes that a patient with a SCI is at risk for muscle spasticity. How can the nurse best prevent this complication of an SCI? A) Position the patient in a high Fowlers position when in bed. B) Support the knees with a pillow when the patient is in bed. C) Perform passive ROM exercises as ordered. D) Administer NSAIDs as ordered.

Ans: C Feedback: Passive ROM exercises can prevent muscle spasticity following SCI. NSAIDs are not used for this purpose. Pillows and sitting upright do not directly address the patients risk of muscle spasticity.

12. A patient is admitted to the neurologic ICU with a spinal cord injury. In writing the patients care plan, the nurse specifies that contractures can best be prevented by what action? A) Repositioning the patient every 2 hours B) Initiating range-of-motion exercises (ROM) as soon as the patient initiates C) Initiating (ROM) exercises as soon as possible after the injury D) Performing ROM exercises once a day

Ans: C Feedback: Passive ROM exercises should be implemented as soon as possible after injury. It would be inappropriate to wait for the patient to first initiate exercises. Toes, metatarsals, ankles, knees, and hips should be put through a full ROM at least four, and ideally five, times daily. Repositioning alone will not prevent contractures.

16. A nurse is caring for a patient with hepatic encephalopathy. The nurses assessment reveals that the patient exhibits episodes of confusion, is difficult to arouse from sleep and has rigid extremities. Based on these clinical findings, the nurse should document what stage of hepatic encephalopathy? A) Stage 1 B) Stage 2 C) Stage 3 D) Stage 4

Ans: C Feedback: Patients in the third stage of hepatic encephalopathy exhibit the following symptoms: stuporous, difficult to arouse, sleeps most of the time, exhibits marked confusion, incoherent in speech, asterixis, increased deep tendon reflexes, rigidity of extremities, marked EEG abnormalities. Patients in stages 1 and 2 exhibit clinical symptoms that are not as advanced as found in stage 3, and patients in stage 4 are comatose. In stage 4, there is an absence of asterixis, absence of deep tendon reflexes, flaccidity of extremities, and EEG abnormalities.

37. Fresh-frozen plasma (FFP) has been ordered for a hospital patient. Prior to administration of this blood product, the nurse should prioritize what patient education? A) Infection risks associated with FFP administration B) Physiologic functions of plasma C) Signs and symptoms of a transfusion reaction D) Strategies for managing transfusion-associated anxiety

Ans: C Feedback: Patients should be educated about signs and symptoms of transfusion reactions prior to administration of any blood product. In most cases, this is priority over education relating to infection. Anxiety may be an issue for some patients, but transfusion reactions are a possibility for all patients. Teaching about the functions of plasma is not likely a high priority.

24. A patient has been scheduled for a bone marrow biopsy and admits to the nurse that she is worried about the pain involved with the procedure. What patient education is most accurate? A) You'll be given painkillers before the test, so there won't likely be any pain? B) You'll feel some pain when the needle enters your skin, but none when the needle enters the bone because of the absence of nerves in bone. C) Most people feel some brief, sharp pain when the needle enters the bone. D) I'll be there with you, and I'll try to help you keep your mind off the pain.

Ans: C Feedback: Patients typically feel a pressure sensation as the needle is advanced into position. The actual aspiration always causes sharp, but brief pain, resulting from the suction exerted as the marrow is aspirated into the syringe; the patient should be warned about this. Stating, I'll try to help you keep your mind off the pain may increase the patient's fears of pain, because this does not help the patient know what to expect.

18. There has been a radiation-based terrorist attack and a patient is experiencing vomiting, diarrhea, and shock after the attack. How will the patient's likelihood of survival be characterized? A) Probable B) Possible C) Improbable D) Extended

Ans: C Feedback: Patients who experience vomiting, diarrhea, and shock after radiation exposure are categorized as improbable survival, because they are demonstrating symptoms of exposure levels of more than 800 rads of total body-penetrating irradiation.

4. The nurse is caring for a patient with a diagnosis of Addisons disease. What sign or symptom is most closely associated with this health problem? A) Truncal obesity B) Hypertension C) Muscle weakness D) Moon face

Ans: C Feedback: Patients with Addisons disease demonstrate muscular weakness, anorexia, gastrointestinal symptoms, fatigue, emaciation, dark pigmentation of the skin, and hypotension. Patients with Cushing syndrome demonstrate truncal obesity, moon face, acne, abdominal striae, and hypertension.

22. A patient with liver disease has developed jaundice; the nurse is collaborating with the patient to develop a nutritional plan. The nurse should prioritize which of the following in the patients plan? A) Increased potassium intake B) Fluid restriction to 2 L per day C) Reduction in sodium intake D) High-protein, low-fat diet

Ans: C Feedback: Patients with ascites require a sharp reduction in sodium intake. Potassium intake should not be correspondingly increased. There is no need for fluid restriction or increased protein intake.

8. A nurse caring for a patient with diabetes insipidus is reviewing laboratory results. What is an expected urinalysis finding? A) Glucose in the urine B) Albumin in the urine C) Highly dilute urine D) Leukocytes in the urine

Ans: C Feedback: Patients with diabetes insipidus produce an enormous daily output of very dilute, water-like urine with a specific gravity of 1.001 to 1.005. The urine contains no abnormal substances such as glucose or albumin. Leukocytes in the urine are not related to the condition of diabetes insipidus, but would indicate a urinary tract infection, if present in the urine.

29. A patient's low hemoglobin level has necessitated transfusion of PRBCs. Prior to administration, what action should the nurse perform? A) Have the patient identify his or her blood type in writing. B) Ensure that the patient has granted verbal consent for transfusion. C) Assess the patient's vital signs to establish baselines. D) Facilitate insertion of a central venous catheter.

Ans: C Feedback: Prior to a transfusion, the nurse must take the patient's temperature, pulse, respiration, and BP to establish a baseline. Written consent is required and the patient's blood type is determined by type and cross match, not by the patient's self-declaration. Peripheral venous access is sufficient for blood transfusion.

17. A nurse is planning the care of a 28-year-old woman hospitalized with a diagnosis of myasthenia gravis. What approach would be most appropriate for the care and scheduling of diagnostic procedures for this patient? A) All at one time, to provide a longer rest period B) Before meals, to stimulate her appetite C) In the morning, with frequent rest periods D) Before bedtime, to promote rest

Ans: C Feedback: Procedures should be spaced to allow for rest in between. Procedures should be avoided before meals, or the patient may be too exhausted to eat. Procedures should be avoided near bedtime if possible.

40. A nurse is caring for a patient with severe hemolytic jaundice. Laboratory tests show free bilirubin to be 24 mg/dL. For what complication is this patient at risk? A) Chronic jaundice B) Pigment stones in portal circulation C) Central nervous system damage D) Hepatomegaly

Ans: C Feedback: Prolonged jaundice, even if mild, predisposes to the formation of pigment stones in the gallbladder, and extremely severe jaundice (levels of free bilirubin exceeding 20 to 25 mg/dL) poses a risk for CNS damage. There are not specific risks of hepatomegaly or chronic jaundice resulting from high bilirubin.

27. The critical care nurse is admitting a patient in myasthenic crisis to the ICU. The nurse should prioritize what nursing action in the immediate care of this patient? A) Suctioning secretions B) Facilitating ABG analysis C) Providing ventilatory assistance D) Administering tube feedings

Ans: C Feedback: Providing ventilatory assistance takes precedence in the immediate management of the patient with myasthenic crisis. It may be necessary to suction secretions and/or provide tube feedings, but they are not the priority for this patient. ABG analysis will be done, but this is not the priority.

25. Diagnostic testing of an adult patient reveals renal glycosuria. The nurse should recognize the need for the patient to be assessed for what health problem? A) Diabetes insipidus B) Syndrome of inappropriate antidiuretic hormone secretion (SIADH) C) Diabetes mellitus D) Renal carcinoma

Ans: C Feedback: Renal glycosuria can occur on its own as a benign condition. It also occurs in poorly controlled diabetes, the most common condition that causes the blood glucose level to exceed the kidney's reabsorption capacity. Glycosuria is not associated with SIADH, diabetes insipidus, or renal carcinoma.

6. A patient has a glomerular filtration rate (GFR) of 43 mL/min/1.73 m2. Based on this GFR, the nurse interprets that the patient's chronic kidney disease is at what stage? A) Stage 1 B) Stage 2 C) Stage 3 D) Stage 4

Ans: C Feedback: Stages of chronic renal failure are based on the GFR. Stage 3 is defined by a GFR in the range of 30 to 59 mL/min/1.73 m2. This is considered a moderate decrease in GFR.

12. Diagnostic testing has revealed that a patients hepatocellular carcinoma (HCC) is limited to one lobe. The nurse should anticipate that this patients plan of care will focus on what intervention? A) Cryosurgery B) Liver transplantation C) Lobectomy D) Laser hyperthermia

Ans: C Feedback: Surgical resection is the treatment of choice when HCC is confined to one lobe of the liver and the function of the remaining liver is considered adequate for postoperative recovery. Removal of a lobe of the liver (lobectomy) is the most common surgical procedure for excising a liver tumor. While cryosurgery and liver transplantation are other surgical options for management of liver cancer, these procedures are not performed at the same frequency as a lobectomy. Laser hyperthermia is a nonsurgical treatment for liver cancer.

2. The current phase of a patients treatment for a burn injury prioritizes wound care, nutritional support, and prevention of complications such as infection. Based on these care priorities, the patient is in what phase of burn care? A) Emergent B) Immediate resuscitative C) Acute D) Rehabilitation

Ans: C Feedback: The acute or intermediate phase of burn care follows the emergent/resuscitative phase and begins 48 to 72 hours after the burn injury. During this phase, attention is directed toward continued assessment and maintenance of respiratory and circulatory status, fluid and electrolyte balance, and gastrointestinal function. Infection prevention, burn wound care (i.e., wound cleaning, topical antibacterial therapy, wound dressing, dressing changes, wound dbridement, and wound grafting), pain management, and nutritional support are priorities at this stage. Priorities during the emergent or immediate resuscitative phase include first aid, prevention of shock and respiratory distress, detection and treatment of concomitant injuries, and initial wound assessment and care. The priorities during the rehabilitation phase include prevention of scars and contractures, rehabilitation, functional and cosmetic reconstruction, and psychosocial counseling.

21. A nurse is creating a care plan for a patient with acute pancreatitis. The care plan includes reduced activity. What rationale for this intervention should be cited in the care plan? A) Bed rest reduces the patients metabolism and reduces the risk of metabolic acidosis. B) Reduced activity protects the physical integrity of pancreatic cells. C) Bed rest lowers the metabolic rate and reduces enzyme production. D) Inactivity reduces caloric need and gastrointestinal motility.

Ans: C Feedback: The acutely ill patient is maintained on bed rest to decrease the metabolic rate and reduce the secretion of pancreatic and gastric enzymes. Staying in bed does not release energy from the body to fight the disease.

13. The nurse is performing a focused genitourinary and renal assessment of a patient. Where should the nurse assess for pain at the costovertebral angle? A) At the umbilicus and the right lower quadrant of the abdomen B) At the suprapubic region and the umbilicus C) At the lower border of the 12th rib and the spine D) At the 7th rib and the xyphoid process

Ans: C Feedback: The costovertebral angle is the angle formed by the lower border of the 12th rib and the spine. Renal dysfunction may produce tenderness over the costovertebral angle.

29. A nurse is caring for a patient who is in the diuresis phase of AKI. The nurse should closely monitor the patient for what complication during this phase? A) Hypokalemia B) Hypocalcemia C) Dehydration D) Acute flank pain

Ans: C Feedback: The diuresis period is marked by a gradual increase in urine output, which signals that glomerular filtration has started to recover. The patient must be observed closely for dehydration during this phase; if dehydration occurs, the uremic symptoms are likely to increase. Excessive losses of potassium and calcium are not typical during this phase, and diuresis does not normally result in pain.

8. A patient who had surgery for gallbladder disease has just returned to the postsurgical unit from postanesthetic recovery. The nurse caring for this patient knows to immediately report what assessment finding to the physician? A) Decreased breath sounds B) Drainage of bile-colored fluid onto the abdominal dressing C) Rigidity of the abdomen D) Acute pain with movement

Ans: C Feedback: The location of the subcostal incision will likely cause the patient to take shallow breaths to prevent pain, which may result in decreased breath sounds. The nurse should remind patients to take deep breaths and cough to expand the lungs fully and prevent atelectasis. Acute pain is an expected assessment finding following surgery; analgesics should be administered for pain relief. Abdominal splinting or application of an abdominal binder may assist in reducing the pain. Bile may continue to drain from the drainage tract after surgery, which will require frequent changes of the abdominal dressing. Increased abdominal tenderness and rigidity should be reported immediately to the physician, as it may indicate bleeding from an inadvertent puncture or nicking of a major blood vessel during the surgical procedure.

6. The nurse is teaching a patient that the body needs iodine for the thyroid to function. What food would be the best source of iodine for the body? A) Eggs B) Shellfish C) Table salt D) Red meat

Ans: C Feedback: The major use of iodine in the body is by the thyroid. Iodized table salt is the best source of iodine.

29. A nurse is aware of the high incidence and prevalence of fluid volume deficit among older adults. What related health education should the nurse provide to an older adult? A) If possible, try to drink at least 4 liters of fluid daily. B) Ensure that you avoid replacing water with other beverages. C) Remember to drink frequently, even if you don't feel thirsty. D) Make sure you eat plenty of salt in order to stimulate thirst.

Ans: C Feedback: The nurse emphasizes the need to drink throughout the day even if the patient does not feel thirsty, because the thirst stimulation is decreased. Four liters of daily fluid intake is excessive and fluids other than water are acceptable in most cases. Additional salt intake is not recommended as a prompt for increased fluid intake.

39. A nurse is providing discharge education to a patient who has undergone a laparoscopic cholecystectomy. During the immediate recovery period, the nurse should recommend what foods? A) High-fiber foods B) Low-purine, nutrient-dense foods C) Low-fat foods high in proteins and carbohydrates D) Foods that are low-residue and low in fat

Ans: C Feedback: The nurse encourages the patient to eat a diet that is low in fats and high in carbohydrates and proteins immediately after surgery. There is no specific need to increase fiber or avoid purines. A low-residue diet is not indicated.

36. A patient is brought to the ED by family members who tell the nurse that the patient has been exhibiting paranoid, agitated behavior. What should the nurse do when interacting with this patient? A) Keep the patient in a confined space. B) Use therapeutic touch appropriately. C) Give the patient honest answers about likely treatment. D) Attempt to convince the patient that his or her fears are unfounded.

Ans: C Feedback: The nurse should offer appropriate and honest explanations in order to foster rapport and trust. Confinement is likely to cause escalation, as is touching the patient. The nurse should not normally engage in trying to convince the patient that his or her fears are unjustified, as this can also cause escalation.

35. The nurse is creating an education plan for a patient who underwent a nephrectomy for the treatment of a renal tumor. What should the nurse include in the teaching plan? A) The importance of increased fluid intake B) Signs and symptoms of rejection C) Inspection and care of the incision D) Techniques for preventing metastasis

Ans: C Feedback: The nurse teaches the patient to inspect and care for the incision and perform other general postoperative care, including activity and lifting restrictions, driving, and pain management. There would be no need to teach the signs or symptoms of rejection as there has been no transplant. Increased fluid intake is not normally recommended and the patient has minimal control on the future risk for metastasis.

7. A patient is prescribed corticosteroid therapy. What would be priority information for the nurse to give the patient who is prescribed long-term corticosteroid therapy? A) The patients diet should be low protein with ample fat. B) The patient may experience short-term changes in cognition. C) The patient is at an increased risk for developing infection. D) The patient is at a decreased risk for development of thrombophlebitis and thromboembolism.

Ans: C Feedback: The patient is at increased risk of infection and masking of signs of infection. The cardiovascular effects of corticosteroid therapy may result in development of thrombophlebitis or thromboembolism. Diet should be high in protein with limited fat. Changes in appearance usually disappear when therapy is no longer necessary. Cognitive changes are not common adverse effects.

6. A patient who has undergone liver transplantation is ready to be discharged home. Which outcome of health education should the nurse prioritize? A) The patient will obtain measurement of drainage from the T-tube. B) The patient will exercise three times a week. C) The patient will take immunosuppressive agents as required. D) The patient will monitor for signs of liver dysfunction.

Ans: C Feedback: The patient is given written and verbal instructions about immunosuppressive agent doses and dosing schedules. The patient is also instructed on steps to follow to ensure that an adequate supply of medication is available so that there is no chance of running out of the medication or skipping a dose. Failure to take medications as instructed may precipitate rejection. The nurse would not teach the patient to measure drainage from a T-tube as the patient wouldnt go home with a T-tube. The nurse may teach the patient about the need to exercise or what the signs of liver dysfunction are, but the nurse would not stress these topics over the immunosuppressive drug regimen.

11. You are a floor nurse caring for a patient with alcohol withdrawal syndrome. What would be an appropriate nursing action to minimize the potential for hallucinations? A) Engage the patient in a process of health education. B) Administer opioid analgesics as ordered. C) Place the patient in a private, well-lit room. D) Provide television or a radio as therapeutic distraction

Ans: C Feedback: The patient should be placed in a quiet single room with lights on and in a calm nonstressful environment. TV and radio stimulation should be avoided. Analgesics are not normally necessary, and would potentially contribute to hallucinations. Health education would be inappropriate while the patient is experiencing acute withdrawal.

28. A patient has been admitted to the hospital for the treatment of chronic pancreatitis. The patient has been stabilized and the nurse is now planning health promotion and educational interventions. Which of the following should the nurse prioritize? A) Educating the patient about expectations and care following surgery B) Educating the patient about the management of blood glucose after discharge C) Educating the patient about postdischarge lifestyle modifications D) Educating the patient about the potential benefits of pancreatic transplantation

Ans: C Feedback: The patients lifestyle (especially regarding alcohol use) is a major determinant of the course of chronic pancreatitis. The disease is not often managed by surgery and blood sugar monitoring is not necessarily indicated for every patient after hospital treatment. Transplantation is not an option.

20. A patient with a fractured femur presenting to the ED exhibits cool, moist skin, increased heart rate, and falling BP. The care team should consider the possibility of what complication of the patient's injuries? A) Myocardial infarction B) Hypoglycemia C) Hemorrhage D) Peritonitis

Ans: C Feedback: The signs and symptoms the patient is experiencing suggest a volume deficit from an internal bleed. That the symptoms follow an acute injury suggests hemorrhage rather than myocardial infarction or hypoglycemia. Peritonitis would be an unlikely result of a femoral fracture.

25. A patient with a diagnosis of cirrhosis has developed variceal bleeding and will imminently undergo variceal banding. What psychosocial nursing diagnosis should the nurse most likely prioritize during this phase of the patients treatment? A) Decisional Conflict B) Deficient Knowledge C) Death Anxiety D) Disturbed Thought Processes

Ans: C Feedback: The sudden hemorrhage that accompanies variceal bleeding is intensely anxiety-provoking. The nurse must address the patients likely fear of death, which is a realistic possibility. For most patients, anxiety is likely to be a more acute concern than lack of knowledge or decisional conflict. The patient may or may not experience disturbances in thought processes.

11. The nurse is caring for a patient suspected of having renal dysfunction. When reviewing laboratory results for this patient, the nurse interprets the presence of which substances in the urine as most suggestive of pathology? A) Potassium and sodium B) Bicarbonate and urea C) Glucose and protein D) Creatinine and chloride

Ans: C Feedback: The various substances normally filtered by the glomerulus, reabsorbed by the tubules, and excreted in the urine include sodium, chloride, bicarbonate, potassium, glucose, urea, creatinine, and uric acid. Within the tubule, some of these substances are selectively reabsorbed into the blood. Glucose is completely reabsorbed in the tubule and normally does not appear in the urine. However, glucose is found in the urine if the amount of glucose in the blood and glomerular filtrate exceeds the amount that the tubules are able to reabsorb. Protein molecules are also generally not found in the urine because amino acids are also filtered at the level of the glomerulus and reabsorbed so that it is not excreted in the urine.

14. The staff educator is giving a class for a group of nurses new to the renal unit. The educator is discussing renal biopsies. In what patient would the educator tell the new nurses that renal biopsies are contraindicated? A) A 64-year-old patient with chronic glomerulonephritis B) A 57-year-old patient with proteinuria C) A 42-year-old patient with morbid obesity D) A 16-year-old patient with signs of kidney transplant rejection

Ans: C Feedback: There are several contraindications to a kidney biopsy, including bleeding tendencies, uncontrolled hypertension, a solitary kidney, and morbid obesity. Indications for a renal biopsy include unexplained acute renal failure, persistent proteinuria or hematuria, transplant rejection, and glomerulopathies.

2. A nurse is performing an admission assessment of a patient with a diagnosis of cirrhosis. What technique should the nurse use to palpate the patients liver? A) Place hand under the right lower abdominal quadrant and press down lightly with the other hand. B) Place the left hand over the abdomen and behind the left side at the 11th rib. C) Place hand under right lower rib cage and press down lightly with the other hand. D) Hold hand 90 degrees to right side of the abdomen and push down firmly.

Ans: C Feedback: To palpate the liver, the examiner places one hand under the right lower rib cage and presses downward with light pressure with the other hand. The liver is not on the left side or in the right lower abdominal quadrant.

35. The nurse caring for a patient with a spinal cord injury notes that the patient is exhibiting early signs and symptoms of disuse syndrome. Which of the following is the most appropriate nursing action? A) Limit the amount of assistance provided with ADLs. B) Collaborate with the physical therapist and immobilize the patients extremities temporarily. C) Increase the frequency of ROM exercises. D) Educate the patient about the importance of frequent position changes.

Ans: C Feedback: To prevent disuse syndrome, ROM exercises must be provided at least four times a day, and care is taken to stretch the Achilles tendon with exercises. The patient is repositioned frequently and is maintained in proper body alignment whether in bed or in a wheelchair. The patient must be repositioned by caregivers, not just taught about repositioning. It is inappropriate to limit assistance for the sole purpose of preventing disuse syndrome.

32. A patient with a spinal cord injury has experienced several hypotensive episodes. How can the nurse best address the patients risk for orthostatic hypotension? A) Administer an IV bolus of normal saline prior to repositioning. B) Maintain bed rest until normal BP regulation returns. C) Monitor the patients BP before and during position changes. D) Allow the patient to initiate repositioning.

Ans: C Feedback: To prevent hypotensive episodes, close monitoring of vital signs before and during position changes is essential. Prolonged bed rest carries numerous risks and it is not possible to provide a bolus before each position change. Following the patients lead may or may not help regulate BP.

19. A nurse is developing a care plan for a patient with a partial-thickness burn, and determines that an appropriate goal is to maintain position of joints in alignment. What is the best rationale for this intervention? A) To prevent neuropathies B) To prevent wound breakdown C) To prevent contractures D) To prevent heterotopic ossification

Ans: C Feedback: To prevent the complication of contractures, the nurse will establish a goal to maintain position of joints in alignment. Gentle range of motion exercises and a consult to PT and OT for exercises and positioning recommendations are also appropriate interventions for the prevention of contractures. Joint alignment is not maintained specifically for preventing neuropathy, wound breakdown, or heterotopic ossification.

3. A patient has come to the OB/GYN clinic due to recent heavy menstrual flow. Because of the patient's consequent increase in RBC production, the nurse knows that the patient may need to increase her daily intake of what substance? A) Vitamin E B) Vitamin D C) Iron D) Magnesium

Ans: C Feedback: To replace blood loss, the rate of red cell production increases. Iron is incorporated into hemoglobin. Vitamins E and D and magnesium do not need to be increased when RBC production is increased.

26. The nurse is providing a health education workshop to a group of adults focusing on cancer prevention. The nurse should emphasize what action in order to reduce participants' risks of renal carcinoma? A) Avoiding heavy alcohol use B) Control of sodium intake C) Smoking cessation D) Adherence to recommended immunization schedules

Ans: C Feedback: Tobacco use is a significant risk factor for renal cancer, surpassing the significance of high alcohol and sodium intake. Immunizations do not address an individual's risk of renal cancer.

19. A patient admitted to the medical unit with impaired renal function is complaining of severe, stabbing pain in the flank and lower abdomen. The patient is being assessed for renal calculi. The nurse recognizes that the stone is most likely in what anatomic location? A) Meatus B) Bladder C) Ureter D) Urethra

Ans: C Feedback: Ureteral pain is characterized as a dull continuous pain that may be intense with voiding. The pain may be described as sharp or stabbing if the bladder is full. This type of pain is inconsistent with a stone being present in the bladder. Stones are not normally situated in the urethra or meatus.

18. A patient is brought by friends to the ED after being involved in a motor vehicle accident. The patient sustained blunt trauma to the abdomen. What nursing action would be most appropriate for this patient? A) Ambulate the patient to expel flatus. B) Place the patient in a high Fowler's position. C) Immobilize the patient on a backboard. D) Place the patient in a left lateral position.

Ans: C Feedback: When admitted for blunt trauma, patients must be immobilized until spinal injury is ruled out. Ambulation, side-lying, and upright positioning would be contraindicated until spinal injury is ruled out.

5. The nurse is caring for a patient receiving hemodialysis three times weekly. The patient has had surgery to form an arteriovenous fistula. What is most important for the nurse to be aware of when providing care for this patient? A) Using a stethoscope for auscultating the fistula is contraindicated. B) The patient feels best immediately after the dialysis treatment. C) Taking a BP reading on the affected arm can damage the fistula. D) The patient should not feel pain during initiation of dialysis.

Ans: C Feedback: When blood flow is reduced through the access for any reason (hypotension, application of BP cuff/tourniquet), the access site can clot. Auscultation of a bruit in the fistula is one way to determine patency. Typically, patients feel fatigued immediately after hemodialysis because of the rapid change in fluid and electrolyte status. Although the area over the fistula may have some decreased sensation, a needle stick is still painful.

2. A patient has been admitted to the post-surgical unit following a thyroidectomy. To promote comfort and safety, how should the nurse best position the patient? A) Side-lying (lateral) with one pillow under the head B) Head of the bed elevated 30 degrees and no pillows placed under the head C) Semi-Fowlers with the head supported on two pillows D) Supine, with a small roll supporting the neck

Ans: C Feedback: When moving and turning the patient, the nurse carefully supports the patients head and avoids tension on the sutures. The most comfortable position is the semi-Fowlers position, with the head elevated and supported by pillows.

16. The nurse is caring for a patient with a nursing diagnosis of deficient fluid volume. The nurse's assessment reveals a BP of 98/52 mm Hg. The nurse should recognize that the patient's kidneys will compensate by secreting what substance? A) Antidiuretic hormone (ADH) B) Aldosterone C) Renin D) Angiotensin

Ans: C Feedback: When the vasa recta detect a decrease in BP, specialized juxtaglomerular cells near the afferent arteriole, distal tubule, and efferent arteriole secrete the hormone renin. Renin converts angiotensinogen to angiotensin I, which is then converted to angiotensin II. The vasoconstriction causes the BP to increase. The adrenal cortex secretes aldosterone in response to stimulation by the pituitary gland, which in turn is in response to poor perfusion or increasing serum osmolality. The result is an increase in BP.

What is a manifestation of hyperthyroidism called? A. Myxedema crisis B. Thyroid storm C. Thyrotoxicosis D. Throidsoic crisis

Ans: C Thyrotoxicosis Rationale: Thyrotoxicosis means an excess of thyroid hormone in the body. Having this condition also means that you have a low level of thyroid stimulating hormone, TSH, in your bloodstream, because the pituitary gland senses that you have "enough" thyroid hormone.

22. An 84-year-old woman diagnosed with cancer is admitted to the oncology unit for surgical treatment. The patient has been on chemotherapeutic agents to decrease the tumor size prior to the planned surgery. The nurse caring for the patient is aware that what precipitating factors in this patient may contribute to AKI? Select all that apply. A) Anxiety B) Low BMI C) Age-related physiologic changes D) Chronic systemic disease E) NPO status

Ans: C, D Feedback: Changes in kidney function with normal aging increase the susceptibility of elderly patients to kidney dysfunction and renal failure. In addition, the presence of chronic, systemic diseases increases the risk of AKI. Low BMI and anxiety are not risk factors for acute renal disease. NPO status is not a risk, provided adequate parenteral hydration is administered.

38. A patient has been assessed for aldosteronism and has recently begun treatment. What are priority areas for assessment that the nurse should frequently address? Select all that apply. A) Pupillary response B) Creatinine and BUN levels C) Potassium level D) Peripheral pulses E) BP

Ans: C, E Feedback: Patients with aldosteronism exhibit a profound decline in the serum levels of potassium, and hypertension is the most prominent and almost universal sign of aldosteronism. Pupillary response, peripheral pulses, and renal function are not directly affected.

What is not a manifestation of hyperthyroidism? A. Thyrotoxicosis B. Decreased sensitivity to catecholamines C. Can occur after irradiation D. Can occur after a presence of a tumor

Ans: C. Can occur after irradiation. Rationale: Radioactive iodine therapy is used to damage part or all of the thyroid as a treatment for hyperthyroidism.

The nurse approaches a patient diagnosed with Cushing's syndrome. The patient is irritable and shoults to the nurse to leave the room. The nurse is aware that this is an expected change in patient's personality due to the disease affects on: A. Glucose tolerance B. Increased acid in GI tract C. Central Nervous System D. Infection Process

Ans: C. Central Nervous System

Decompensated hepatic cirrhosis include all except: A. Abdominal pain B. Muscle wasting C. Clubbing of fingers D. Gonadal atrophy

Ans: C. Clubbing of the fingers

What is an intervention in the management of myxedema? A. Avoid rapid overcooling B. Cautious fluid replacement C. Give levothyroxine by mouth D. Turning every two hours

Ans: C. Give levothyroxine by mouth

A critical care nurse learns during change of shift report that the assigned client experienced acute kidney injury following surgery. Which of the following causes should the nurse suspect? A. Vascular disease B. Urethral obstruction C. Hypovolemia D. Glomerulonephritis

Ans: C. Hypovolemia

The nurse in the hematology clinic is reviewing laboratory and assessment data for a client. Which finding is the best indication that the treatment is successful? A. The client is no longer cyanotic B. The client is more active C. The reticulocyte count is rising D. Stools are black

Ans: C. The reticulocyte count is rising

What lab value determines Levothyroxine dosage? A. T3 B. T4 C. CBC D. TSH

Ans: D

11. A patient who suffered a spinal cord injury is experiencing an exaggerated autonomic response. What aspect of the patients current health status is most likely to have precipitated this event? A) The patient received a blood transfusion. B) The patients analgesia regimen was recent changed. C) The patient was not repositioned during the night shift. D) The patients urinary catheter became occluded.

Ans: D Feedback: A distended bladder is the most common cause of autonomic dysreflexia. Infrequent positioning is a less likely cause, although pressure ulcers or tactile stimulation can cause it. Changes in mediations or blood transfusions are unlikely causes.

1. A patient is brought to the emergency department from the site of a chemical fire, where he suffered a burn that involves the epidermis, dermis, and the muscle and bone of the right arm. On inspection, the skin appears charred. Based on these assessment findings, what is the depth of the burn on the patients arm? A) Superficial partial-thickness B) Deep partial-thickness C) Full partial-thickness D) Full-thickness

Ans: D Feedback: A full-thickness burn involves total destruction of the epidermis and dermis and, in some cases, underlying tissue as well. Wound color ranges widely from white to red, brown, or black. The burned area is painless because the nerve fibers are destroyed. The wound can appear leathery; hair follicles and sweat glands are destroyed. Edema may also be present. Superficial partial-thickness burns involve the epidermis and possibly a portion of the dermis; the patient will experience pain that is soothed by cooling. Deep partial-thickness burns involve the epidermis, upper dermis, and portion of the deeper dermis; the patient will complain of pain and sensitivity to cold air. Full partial thickness is not a depth of burn.

7. The nurse is creating a plan of care for a patient who has a recent diagnosis of MS. Which of the following should the nurse include in the patients care plan? A) Encourage patient to void every hour. B) Order a low-residue diet. C) Provide total assistance with all ADLs. D) Instruct the patient on daily muscle stretching.

Ans: D Feedback: A patient diagnosed with MS should be encouraged to increase the fiber in his or her diet and void 30 minutes after drinking to help train the bladder. The patient should participate in daily muscle stretching to help alleviate and relax muscle spasms.

16. A patient has been treated in the hospital for an episode of acute pancreatitis. The patient has acknowledged the role that his alcohol use played in the development of his health problem, but has not expressed specific plans for lifestyle changes after discharge. What is the nurses most appropriate response? A) Educate the patient about the link between alcohol use and pancreatitis. B) Ensure that the patient knows the importance of attending follow-up appointments. C) Refer the patient to social work or spiritual care. D) Encourage the patient to connect with a community-based support group.

Ans: D Feedback: After the acute attack has subsided, some patients may be inclined to return to their previous drinking habits. The nurse provides specific information about resources and support groups that may be of assistance in avoiding alcohol in the future. Referral to Alcoholics Anonymous as appropriate or other support groups is essential. The patient already has an understanding of the effects of alcohol, and follow-up appointments will not necessarily result in lifestyle changes. Social work and spiritual care may or may not be beneficial.

37. The emergency response team is dealing with a radiation leak at the hospital. What action should be performed to prevent the spread of the contaminants? A) Floors must be scrubbed with undiluted bleach. B) Waste must be promptly incinerated. C) The ventilation system should be deactivated. D) Air ducts and vents should be sealed.

Ans: D Feedback: All air ducts and vents must be sealed to prevent spread. Waste is controlled through double-bagging and the use of plastic-lined containers outside of the facility rather than incineration. Bleach would be ineffective against radiation and the ventilation system may or may not be deactivated.

2. A 55-year-old man has been newly diagnosed with acute pancreatitis and admitted to the acute medical unit. How should the nurse most likely explain the pathophysiology of this patients health problem? A) Toxins have accumulated and inflamed your pancreas. B) Bacteria likely migrated from your intestines and became lodged in your pancreas. C) A virus that was likely already present in your body has begun to attack your pancreatic cells. D) The enzymes that your pancreas produces have damaged the pancreas itself.

Ans: D Feedback: Although the mechanisms causing pancreatitis are unknown, pancreatitis is commonly described as the autodigestion of the pancreas. Less commonly, toxic substances and microorganisms are implicated as the cause of pancreatitis.

16. A patient is admitted to the ED who has been exposed to a nerve agent. The nurse should anticipate the STAT administration of what drug? A) Amyl nitrate B) Dimercaprol C) Erythromycin D) Atropine

Ans: D Feedback: Atropine is administered when a patient is exposed to a nerve agent. Exposure to blood agents, such as cyanide, requires treatment with amyl nitrate, sodium nitrite, and sodium thiosulfate. Dimercaprol is administered IV for systemic toxicity and topically for skin lesions when exposed to vesicants. Erythromycin is an antibiotic, which is ineffective against nerve agents.

18. A nurse is educating a patient about the role of B lymphocytes. The nurse's description will include which of the following physiologic processes? A) Stem cell differentiation B) Cytokine production C) Phagocytosis D) Antibody production

Ans: D Feedback: B lymphocytes are capable of differentiating into plasma cells. Plasma cells, in turn, produce antibodies. Cytokines are produced by NK cells. Stem cell differentiation greatly precedes B lymphocyte production.

28. The nurse caring for a patient in ICU diagnosed with Guillain-Barr syndrome should prioritize monitoring for what potential complication? A) Impaired skin integrity B) Cognitive deficits C) Hemorrhage D) Autonomic dysfunction

Ans: D Feedback: Based on the assessment data, potential complications that may develop include respiratory failure and autonomic dysfunction. Skin breakdown, decreased cognition, and hemorrhage are not complications of Guillain-Barr syndrome.

27. A patient with esophageal varices is being cared for in the ICU. The varices have begun to bleed and the patient is at risk for hypovolemia. The patient has Ringers lactate at 150 cc/hr infusing. What else might the nurse expect to have ordered to maintain volume for this patient? A) Arterial line B) Diuretics C) Foley catheter D) Volume expanders

Ans: D Feedback: Because patients with bleeding esophageal varices have intravascular volume depletion and are subject to electrolyte imbalance, IV fluids with electrolytes and volume expanders are provided to restore fluid volume and replace electrolytes. Diuretics would reduce vascular volume. An arterial line and Foley catheter are likely to be ordered, but neither actively maintains the patients volume.

21. A patient who has been diagnosed with cholecystitis is being discharged home from the ED to be scheduled for surgery later. The patient received morphine during the present ED admission and is visibly drowsy. When providing health education to the patient, what would be the most appropriate nursing action? A) Give written instructions to patient. B) Give verbal instructions to one of the patient's family members. C) Telephone the patient the next day with verbal instructions. D) Give verbal and written instructions to patient and a family member.

Ans: D Feedback: Before discharge, verbal and written instructions for continuing care are given to the patient and the family or significant others. Discharge teaching is completed prior to the patient leaving the ED, so phoning the patient the next day in not acceptable.

10. A kidney biopsy has been scheduled for a patient with a history of acute renal failure. The patient asks the nurse why this test has been scheduled. What is the nurse's best response? A) A biopsy is routinely ordered for all patients with renal disorders. B) A biopsy is generally ordered following abnormal x-ray findings of the renal pelvis. C) A biopsy is often ordered for patients before they have a kidney transplant. D) A biopsy is sometimes necessary for diagnosing and evaluating the extent of kidney disease.

Ans: D Feedback: Biopsy of the kidney is used in diagnosing and evaluating the extent of kidney disease. Indications for biopsy include unexplained acute renal failure, persistent proteinuria or hematuria, transplant rejection, and glomerulopathies.

3. The nurse is caring for a patient with a history of systemic lupus erythematosus who has been recently diagnosed with end-stage kidney disease (ESKD). The patient has an elevated phosphorus level and has been prescribed calcium acetate to bind the phosphorus. The nurse should teach the patient to take the prescribed phosphorus-binding medication at what time? A) Only when needed B) Daily at bedtime C) First thing in the morning D) With each meal

Ans: D Feedback: Both calcium carbonate and calcium acetate are medications that bind with the phosphate and assist in excreting the phosphate from the body, in turn lowering the phosphate levels. Phosphate-binding medications must be administered with food to be effective.

27. A patient has been taking prednisone for several weeks after experiencing a hypersensitivity reaction. To prevent adrenal insufficiency, the nurse should ensure that the patient knows to do which of the following? A) Take the drug concurrent with levothyroxine (Synthroid). B) Take each dose of prednisone with a dose of calcium chloride. C) Gradually replace the prednisone with an OTC alternative. D) Slowly taper down the dose of prednisone, as ordered.

Ans: D Feedback: Corticosteroid dosages are reduced gradually (tapered) to allow normal adrenal function to return and to prevent steroid-induced adrenal insufficiency. There are no OTC substitutes for prednisone and neither calcium chloride nor levothyroxine addresses the risk of adrenal insufficiency.

19. A patient with hypofunction of the adrenal cortex has been admitted to the medical unit. What would the nurse most likely find when assessing this patient? A) Increased body temperature B) Jaundice C) Copious urine output D) Decreased BP

Ans: D Feedback: Decreased BP may occur with hypofunction of the adrenal cortex. Decreased function of the adrenal cortex does not affect the patients body temperature, urine output, or skin tone.

1. A nurse is caring for a patient with liver failure and is performing an assessment in the knowledge of the patients increased risk of bleeding. The nurse recognizes that this risk is related to the patients inability to synthesize prothrombin in the liver. What factor most likely contributes to this loss of function? A) Alterations in glucose metabolism B) Retention of bile salts C) Inadequate production of albumin by hepatocytes D) Inability of the liver to use vitamin K

Ans: D Feedback: Decreased production of several clotting factors may be partially due to deficient absorption of vitamin K from the GI tract. This probably is caused by the inability of liver cells to use vitamin K to make prothrombin. This bleeding risk is unrelated to the roles of glucose, bile salts, or albumin.

37. The nurse caring for a patient who is recovering from full-thickness burns is aware of the patients risk for contracture and hypertrophic scarring. How can the nurse best mitigate this risk? A) Apply skin emollients as ordered after granulation has occurred. B) Keep injured areas immobilized whenever possible to promote healing. C) Administer oral or IV corticosteroids as ordered. D) Encourage physical activity and range of motion exercises.

Ans: D Feedback: Exercise and the promotion of mobility can reduce the risk of contracture and hypertrophic scarring. Skin emollients are not normally used in the treatment of burns, and these do not prevent scarring. Steroids are not used to reduce scarring, as they also slow the healing process.

34. A patient with liver cancer is being discharged home with a biliary drainage system in place. The nurse should teach the patients family how to safely perform which of the following actions? A) Aspirating bile from the catheter using a syringe B) Removing the catheter when output is 15 mL in 24 hours C) Instilling antibiotics into the catheter D) Assessing the patency of the drainage catheter

Ans: D Feedback: Families should be taught to provide basic catheter care, including assessment of patency. Antibiotics are not instilled into the catheter and aspiration using a syringe is contraindicated. The family would not independently remove the catheter; this would be done by a member of the care team when deemed necessary.

26. A patients burns are estimated at 36% of total body surface area; fluid resuscitation has been ordered in the emergency department. After establishing intravenous access, the nurse should anticipate the administration of what fluid? A) 0.45% NaCl with 20 mEq/L KCl B) 0.45% NaCl with 40 mEq/L KCl C) Normal saline D) Lactated Ringers

Ans: D Feedback: Fluid resuscitation with lactated Ringers (LR) should be initiated using the American Burn Associations (ABA) fluid resuscitation formulas. LR is the crystalloid of choice because its composition and osmolality most closely resemble plasma and because use of normal saline is associated with hyperchloremic acidosis. Potassium chloride solutions would exacerbate the hyperkalemia that occurs following burn injuries.

10. A local public health nurse is informed that a cook in a local restaurant has been diagnosed with hepatitis A. What should the nurse advise individuals to obtain who ate at this restaurant and have never received the hepatitis A vaccine? A) The hepatitis A vaccine B) Albumin infusion C) The hepatitis A and B vaccines D) An immune globulin injection

Ans: D Feedback: For people who have not been previously vaccinated, hepatitis A can be prevented by the intramuscular administration of immune globulin during the incubation period, if given within 2 weeks of exposure. Administration of the hepatitis A vaccine will not protect the patient exposed to hepatitis A, as protection will take a few weeks to develop after the first dose of the vaccine. The hepatitis B vaccine provides protection again the hepatitis B virus, but plays no role in protection for the patient exposed to hepatitis A. Albumin confers no therapeutic benefit.

11. The nurse caring for a patient diagnosed with Guillain-Barr syndrome is planning care with regard to the clinical manifestations associated this syndrome. The nurses communication with the patient should reflect the possibility of what sign or symptom of the disease? A) Intermittent hearing loss B) Tinnitus C) Tongue enlargement D) Vocal paralysis

Ans: D Feedback: Guillain-Barr syndrome is a disorder of the vagus nerve. Clinical manifestations include vocal paralysis, dysphagia, and voice changes (temporary or permanent hoarseness). Hearing deficits, tinnitus, and tongue enlargement are not associated with the disease.

30. A patient with pheochromocytoma has been admitted for an adrenalectomy to be performed the following day. To prevent complications, the nurse should anticipate preoperative administration of which of the following? A) IV antibiotics B) Oral antihypertensives C) Parenteral nutrition D) IV corticosteroids

Ans: D Feedback: IV administration of corticosteroids (methylprednisolone sodium succinate [Solu-Medrol) may begin on the evening before surgery and continue during the early postoperative period to prevent adrenal insufficiency. Antibiotics, antihypertensives, and parenteral nutrition do not prevent adrenal insufficiency or other common complications of adrenalectomy.

14. The nurse educating a patient with anemia is describing the process of RBC production. When the patient's kidneys sense a low level of oxygen in circulating blood, what physiologic response is initiated? A) Increased stem cell synthesis B) Decreased respiratory rate C) Arterial vasoconstriction D) Increased production of erythropoietin

Ans: D Feedback: If the kidney detects low levels of oxygen, as occurs when fewer red cells are available to bind oxygen (i.e., anemia), erythropoietin levels increase. The body does not compensate with vasoconstriction, decreased respiration, or increased stem cell activity.

10. A triage nurse is talking to a patient when the patient begins choking on his lunch. The patient is coughing forcefully. What should the nurse do? A) Stand him up and perform the abdominal thrust maneuver from behind. B) Lay him down, straddle him, and perform the abdominal thrust maneuver. C) Leave him to get assistance. D) Stay with him and encourage him, but not intervene at this time.

Ans: D Feedback: If the patient is coughing, he should be able to dislodge the object or cause a complete obstruction. If complete obstruction occurs, the nurse should perform the abdominal thrust maneuver with the patient standing. If the patient is unconscious, the nurse should lay the patient down. A nurse should never leave a choking patient alone.

8. The nurse is caring for a patient who has developed scar tissue in many of the areas that normally produce blood cells. What organs can become active in blood cell production by the process of extramedullary hematopoiesis? A) Spleen and kidneys B) Kidneys and pancreas C) Pancreas and liver D) Liver and spleen

Ans: D Feedback: In adults with disease that causes marrow destruction, fibrosis, or scarring, the liver and spleen can also resume production of blood cells by a process known as extramedullary hematopoiesis. The kidneys and pancreas do not produce blood cells for the body.

24. A patient with a recent diagnosis of hypothyroidism is being treated for an unrelated injury. When administering medications to the patient, the nurse should know that the patients diminished thyroid function may have what effect? A) Anaphylaxis B) Nausea and vomiting C) Increased risk of drug interactions D) Prolonged duration of effect

Ans: D Feedback: In all patients with hypothyroidism, the effects of analgesic agents, sedatives, and anesthetic agents are prolonged. There is no direct increase in the risk of anaphylaxis, nausea, or drug interactions, although these may potentially result from the prolonged half-life of drugs.

40. A patient presents to the emergency department (ED) complaining of severe right upper quadrant pain. The patient states that his family doctor told him he had gallstones. The ED nurse should recognize what possible complication of gallstones? A) Acute pancreatitis B) Atrophy of the gallbladder C) Gallbladder cancer D) Gangrene of the gallbladder

Ans: D Feedback: In calculous cholecystitis, a gallbladder stone obstructs bile outflow. Bile remaining in the gallbladder initiates a chemical reaction; autolysis and edema occur; and the blood vessels in the gallbladder are compressed, compromising its vascular supply. Gangrene of the gallbladder with perforation may result. Pancreatitis, atrophy, and cancer of the gallbladder are not plausible complications.

12. The home care nurse is conducting patient teaching with a patient on corticosteroid therapy. To achieve consistency with the bodys natural secretion of cortisol, when would the home care nurse instruct the patient to take his or her corticosteroids? A) In the evening between 4 PM and 6 PM B) Prior to going to sleep at night C) At noon every day D) In the morning between 7 AM and 8 AM

Ans: D Feedback: In keeping with the natural secretion of cortisol, the best time of day for the total corticosteroid dose is in the morning from 7 to 8 AM. Large-dose therapy at 8 AM, when the adrenal gland is most active, produces maximal suppression of the gland. Also, a large 8 AM dose is more physiologic because it allows the body to escape effects of the steroids from 4 PM to 6 AM, when serum levels are normally low, thus minimizing cushingoid effects.

37. A patient with liver cancer is being discharged home with a hepatic artery catheter in place. The nurse should be aware that this catheter will facilitate which of the following? A) Continuous monitoring for portal hypertension B) Administration of immunosuppressive drugs during the first weeks after transplantation C) Real-time monitoring of vascular changes in the hepatic system D) Delivery of a continuous chemotherapeutic dose

Ans: D Feedback: In most cases, the hepatic artery catheter has been inserted surgically and has a prefilled infusion pump that delivers a continuous chemotherapeutic dose until completed. The hepatic artery catheter does not monitor portal hypertension, deliver immunosuppressive drugs, or monitor vascular changes in the hepatic system.

23. A patient who attempted suicide being treated in the ED is accompanied by his mother, father, and brother. When planning the nursing care of this family, the nurse should perform which of the following action? A) Refer the family to psychiatry in order to provide them with support. B) Explore the causes of the patient's suicide attempt with the family. C) Encourage the family to participate in the bedside care of the patient. D) Ensure that the family receives appropriate crisis intervention services.

Ans: D Feedback: It is essential that family crisis intervention services are available for families of ED patients. It would be inappropriate and insensitive to explore causes of the patient's suicide attempt with the family. Family participation in bedside care is often impractical in the ED setting. Psychiatry is not the normal source of psychosocial support and crisis intervention.

11. Level C personal protective equipment has been deemed necessary in the response to an unknown substance. The nurse is aware that the equipment will include what? A) A self-contained breathing apparatus B) A vapor-tight, chemical-resistant suit C) A uniform only D) An air-purified respirator

Ans: D Feedback: Level C incorporates the use of an air-purified respirator, a chemical resistant coverall with splash hood, chemical-resistant gloves, and boots. Level A provides the highest level of respiratory, mucous membrane, skin, and eye protection, incorporating a vapor-tight, chemical-resistant suit and self-contained breathing apparatus (SCBA). Level B personal protective equipment provides the highest level of respiratory protection, with a lesser level of skin and eye protection, incorporating a chemical-resistant suit and SCBA. Level D is the same as a work uniform.

7. A geriatric nurse is performing an assessment of body systems on an 85-year-old patient. The nurse should be aware of what age-related change affecting the renal or urinary system? A) Increased ability to concentrate urine B) Increased bladder capacity C) Urinary incontinence D) Decreased glomerular filtration rate

Ans: D Feedback: Many age-related changes in the renal and urinary systems should be taken into consideration when taking a health history of the older adult. One change includes a decreased glomerular surface area resulting in a decreased glomerular filtration rate. Other changes include the decreased ability to concentrate urine and a decreased bladder capacity. It also should be understood that urinary incontinence is not a normal age-related change, but is common in older adults, especially in women because of the loss of pelvic muscle tone.

14. The nurse is caring for a patient with hyperparathyroidism. What level of activity would the nurse expect to promote? A) Complete bed rest B) Bed rest with bathroom privileges C) Out of bed (OOB) to the chair twice a day D) Ambulation and activity as tolerated

Ans: D Feedback: Mobility, with walking or use of a rocking chair for those with limited mobility, is encouraged as much as possible because bones subjected to normal stress give up less calcium. Best rest should be discouraged because it increases calcium excretion and the risk of renal calculi. Limiting the patient to getting out of bed only a few times a day also increases calcium excretion and the associated risks.

12. The nurse is preparing to provide care for a patient diagnosed with myasthenia gravis. The nurse should know that the signs and symptoms of the disease are the result of what? A) Genetic dysfunction B) Upper and lower motor neuron lesions C) Decreased conduction of impulses in an upper motor neuron lesion D) A lower motor neuron lesion

Ans: D Feedback: Myasthenia gravis is characterized by a weakness of muscles, especially in the face and throat, caused by a lower neuron lesion at the myoneural junction. It is not a genetic disorder. A combined upper and lower neuron lesion generally occurs as a result of spinal injuries. A lesion involving cranial nerves and their axons in the spinal cord would cause decreased conduction of impulses at an upper motor neuron.

17. A patient was exposed to a dose of more than 5,000 rads of radiation during a terrorist attack. The patient's skin will eventually show what manifestation? A) Erythema B) Ecchymosis C) Desquamation D) Necrosis

Ans: D Feedback: Necrosis of the skin will become evident within a few days to months at doses of more than 5,000 rads. With 600 to 1,000 rads, erythema will occur; it can disappear within hours and then reappear. At greater than 1,000 rads, desquamation (radiation dermatitis) of the skin will occur. Ecchymosis does not occur.

8. A nurse is participating in the emergency care of a patient who has just developed variceal bleeding. What intervention should the nurse anticipate? A) Infusion of intravenous heparin B) IV administration of albumin C) STAT administration of vitamin K by the intramuscular route D) IV administration of octreotide (Sandostatin)

Ans: D Feedback: Octreotide (Sandostatin)a synthetic analog of the hormone somatostatinis effective in decreasing bleeding from esophageal varices, and lacks the vasoconstrictive effects of vasopressin. Because of this safety and efficacy profile, octreotide is considered the preferred treatment regimen for immediate control of variceal bleeding. Vitamin K and albumin are not administered and heparin would exacerbate, not alleviate, bleeding.

5. An occupational health nurse is called to the floor of a factory where a worker has sustained a flash burn to the right arm. The nurse arrives and the flames have been extinguished. The next step is to cool the burn. How should the nurse cool the burn? A) Apply ice to the site of the burn for 5 to 10 minutes. B) Wrap the patients affected extremity in ice until help arrives. C) Apply an oil-based substance or butter to the burned area until help arrives. D) Wrap cool towels around the affected extremity intermittently.

Ans: D Feedback: Once the burn has been sustained, the application of cool water is the best first-aid measure. Soaking the burn area intermittently in cool water or applying cool towels gives immediate and striking relief from pain, and limits local tissue edema and damage. However, never apply ice directly to the burn, never wrap the person in ice, and never use cold soaks or dressings for longer than several minutes; such procedures may worsen the tissue damage and lead to hypothermia in people with large burns. Butter is contraindicated.

40. The nurse is providing care for an older adult patient whose current medication regimen includes levothyroxine (Synthroid). As a result, the nurse should be aware of the heightened risk of adverse effects when administering an IV dose of what medication? A) A fluoroquinalone antibiotic B) A loop diuretic C) A proton pump inhibitor (PPI) D) A benzodiazepine

Ans: D Feedback: Oral thyroid hormones interact with many other medications.Even in small IV doses, hypnotic and sedative agents may induce profound somnolence, lasting far longer than anticipated and leading to narcosis (stupor like condition). Furthermore, they are likely to cause respiratory depression, which can easily be fatal because of decreased respiratory reserve and alveolar hypoventilation. Antibiotics, PPIs and diuretics do not cause the same risk.

27. The nurse performing the health interview of a patient with a new onset of periorbital edema has completed a genogram, noting the health history of the patient's siblings, parents, and grandparents. This assessment addresses the patient's risk of what kidney disorder? A) Nephritic syndrome B) Acute glomerulonephritis C) Nephrotic syndrome D) Polycystic kidney disease (PKD)

Ans: D Feedback: PKD is a genetic disorder characterized by the growth of numerous cysts in the kidneys. Nephritic syndrome, acute glomerulonephritis, and nephrotic syndrome are not genetic disorders.

13. A nurse is caring for a patient who has sustained a deep partial-thickness burn injury. In prioritizing the nursing diagnoses for the plan of care, the nurse will give the highest priority to what nursing diagnosis? A) Activity Intolerance B) Anxiety C) Ineffective Coping D) Acute Pain

Ans: D Feedback: Pain is inevitable during recovery from any burn injury. Pain in the burn patient has been described as one of the most severe causes of acute pain. Management of the often-severe pain is one of the most difficult challenges facing the burn team. While the other nursing diagnoses listed are valid, the presence of pain may contribute to these diagnoses. Management of the patients pain is the priority, as it may have a direct correlation to the other listed nursing diagnoses.

33. A patient has been diagnosed with pancreatic cancer and has been admitted for care. Following initial treatment, the nurse should be aware that the patient is most likely to require which of the following? A) Inpatient rehabilitation B) Rehabilitation in the home setting C) Intensive physical therapy D) Hospice care

Ans: D Feedback: Pancreatic carcinoma has only a 5% survival rate at 5 years regardless of the stage of disease at diagnosis or treatment. As a result, there is a higher likelihood that the patient will require hospice care than physical therapy and rehabilitation.

33. A nurse on the neurologic unit is providing care for a patient who has spinal cord injury at the level of C4. When planning the patients care, what aspect of the patients neurologic and functional status should the nurse consider? A) The patient will be unable to use a wheelchair. B) The patient will be unable to swallow food. C) The patient will be continent of urine, but incontinent of bowel. D) The patient will require full assistance for all aspects of elimination.

Ans: D Feedback: Patients with a lesion at C4 are fully dependent for elimination. The patient is dependent for feeding, but is able to swallow. The patient will be capable of using an electric wheelchair.

35. A patient lives with a diagnosis of sickle cell anemia and receives frequent blood transfusions. The nurse should recognize the patient's consequent risk of what complication of treatment? A) Hypovolemia B) Vitamin B12 deficiency C) Thrombocytopenia D) Iron overload

Ans: D Feedback: Patients with chronic transfusion requirements can quickly acquire more iron than they can use, leading to iron overload. These individuals are not at risk for hypovolemia and there is no consequent risk for low platelet or vitamin B12 levels.

24. A 15-year-old is admitted to the renal unit with a diagnosis of postinfectious glomerular disease. The nurse should recognize that this form of kidney disease may have been precipitated by what event? A) Psychosocial stress B) Hypersensitivity to an immunization C) Menarche D) Streptococcal infection

Ans: D Feedback: Postinfectious causes of postinfectious glomerular disease are group A beta-hemolytic streptococcal infection of the throat that precedes the onset of glomerulonephritis by 2 to 3 weeks. Menarche, stress, and hypersensitivity are not typical causes.

23. The nurse is caring for a patient who is going to have an open renal biopsy. What would be an important nursing action in preparing this patient for the procedure? A) Discuss the patient's diagnosis with the family. B) Bathe the patient before the procedure with antiseptic skin wash. C) Administer antivirals before sending the patient for the procedure. D) Keep the patient NPO prior to the procedure.

Ans: D Feedback: Preparation for an open biopsy is similar to that for any major abdominal surgery. When preparing the patient for an open biopsy you would keep the patient NPO. You may discuss the diagnosis with the family, but that is not a preparation for the procedure. A pre-procedure wash is not normally ordered and antivirals are not administered in anticipation of a biopsy.

32. A nurse is caring for a patient with burns who is in the later stages of the acute phase of recovery. The plan of nursing care should include which of the following nursing actions? A) Maintenance of bed rest to aid healing B) Choosing appropriate splints and functional devices C) Administration of beta adrenergic blockers D) Prevention of venous thromboembolism

Ans: D Feedback: Prevention of deep vein thrombosis (DVT) is an important factor in care. Early mobilization of the patient is important. The nurse monitors the splints and functional devices, but these are selected by occupational and physical therapists. The hemodynamic changes accompanying burns do not normally require the use of beta blockers.

30. A nurse is performing a home visit to a patient who is recovering following a long course of inpatient treatment for burn injuries. When performing this home visit, the nurse should do which of the following? A) Assess the patient for signs of electrolyte imbalances. B) Administer fluids as ordered. C) Assess the risk for injury recurrence. D) Assess the patients psychosocial state.

Ans: D Feedback: Recovery from burns can be psychologically challenging; the nurses assessments must address this reality. Fluid and electrolyte imbalances are infrequent during the rehabilitation phase of recovery. Burns are not typically a health problem that tends to recur; the experience of being burned tends to foster vigilance.

6. When assessing patients who are victims of a chemical agent attack, the nurse is aware that assessment findings vary based on the type of chemical agent. The chemical sulfur mustard is an example of what type of chemical warfare agent? A) Nerve agent B) Blood agent C) Pulmonary agent D) Vesicant

Ans: D Feedback: Sulfur mustard is a vesicant chemical that causes blistering and results in burning, conjunctivitis, bronchitis, pneumonia, hematopoietic suppression, and death. Nerve agents include sarin, soman, tabun, VX, and organophosphates (pesticides). Hydrogen cyanide is a blood agent that has a direct effect on cellular metabolism, resulting in asphyxiation through alterations in hemoglobin. Chlorine is a pulmonary agent, which destroys the pulmonary membrane that separates the alveolus from the capillary bed.

34. A patient on corticosteroid therapy needs to be taught that a course of corticosteroids of 2 weeks duration can suppress the adrenal cortex for how long? A) Up to 4 weeks B) Up to 3 months C) Up to 9 months D) Up to 1 year

Ans: D Feedback: Suppression of the adrenal cortex may persist up to 1 year after a course of corticosteroids of only 2 weeks duration.

6. An emergency department nurse has just admitted a patient with a burn. What characteristic of the burn will primarily determine whether the patient experiences a systemic response to this injury? A) The length of time since the burn B) The location of burned skin surfaces C) The source of the burn D) The total body surface area (TBSA) affected by the burn

Ans: D Feedback: Systemic effects are a result of several variables. However, TBSA and wound severity are considered the major factors that affect the presence or absence of systemic effects.

16. A patient is brought to the ED by paramedics, who report that the patient has partial-thickness burns on the chest and legs. The patient has also suffered smoke inhalation. What is the priority in the care of a patient who has been burned and suffered smoke inhalation? A) Pain B) Fluid balance C) Anxiety and fear D) Airway management

Ans: D Feedback: Systemic threats from a burn are the greatest threat to life. The ABCs of all trauma care apply during the early postburn period. While all options should be addressed, pain, fluid balance, and anxiety and fear do not take precedence over airway management.

23. A patient's diagnosis of atrial fibrillation has prompted the primary care provider to prescribe warfarin (Coumadin), an anticoagulant. When assessing the therapeutic response to this medication, what is the nurse's most appropriate action? A) Assess for signs of myelosuppression. B) Review the patient's platelet level. C) Assess the patient's capillary refill time. D) Review the patient's international normalized ratio (INR).

Ans: D Feedback: The INR and aPTT serve as useful screening tools for evaluating a patient's clotting ability and to monitor the therapeutic effectiveness of anticoagulant medications. The patient's platelet level is not normally used as a short-term indicator of anticoagulation effectiveness. Assessing the patient for signs of myelosuppression and capillary refill time does not address the effectiveness of anticoagulants.

33. A nurse is caring for a patient who has been the victim of sexual assault. The nurse documents that the patient appears to be in a state of shock, verbalizing fear, guilt, and humiliation. What phase of rape trauma syndrome is this patient most likely experiencing? A) Reorganization phase B) Denial phase C) Heightened anxiety phase D) Acute disorganization phase

Ans: D Feedback: The acute disorganization phase may manifest as an expressed state in which shock, disbelief, fear, guilt, humiliation, anger, and other such emotions are encountered. These varied responses to the assault are not associated with a denial, heightened anxiety, or reorganization phase.

27. A patient's most recent laboratory findings indicate a glomerular filtration rate (GFR) of 58 mL/min. The nurse should recognize what implication of this diagnostic finding? A) The patient is likely to have a decreased level of blood urea nitrogen (BUN). B) The patient is at risk for hypokalemia. C) The patient is likely to have irregular voiding patterns. D) The patient is likely to have increased serum creatinine levels.

Ans: D Feedback: The adult GFR can vary from a normal of approximately 125 mL/min (1.67 to 2.0 mL/sec) to a high of 200 mL/min. A low GFR is associated with increased levels of BUN, creatinine, and potassium.

36. A patient with chronic kidney disease has been hospitalized and is receiving hemodialysis on a scheduled basis. The nurse should include which of the following actions in the plan of care? A) Ensure that the patient moves the extremity with the vascular access site as little as possible. B) Change the dressing over the vascular access site at least every 12 hours. C) Utilize the vascular access site for infusion of IV fluids. D) Assess for a thrill or bruit over the vascular access site each shift.

Ans: D Feedback: The bruit, or "thrill," over the venous access site must be evaluated at least every shift. Frequent dressing changes are unnecessary and the patient does not normally need to immobilize the site. The site must not be used for purposes other than dialysis.

20. A nurse is assessing an elderly patient with gallstones. The nurse is aware that the patient may not exhibit typical symptoms, and that particular symptoms that may be exhibited in the elderly patient may include what? A) Fever and pain B) Chills and jaundice C) Nausea and vomiting D) Signs and symptoms of septic shock

Ans: D Feedback: The elderly patient may not exhibit the typical symptoms of fever, pain, chills jaundice, and nausea and vomiting. Symptoms of biliary tract disease in the elderly may be accompanied or preceded by those of septic shock, which include oliguria, hypotension, change in mental status, tachycardia, and tachypnea.

21. A patient has presented with signs and symptoms that are characteristic of acute kidney injury, but preliminary assessment reveals no obvious risk factors for this health problem. The nurse should recognize the need to interview the patient about what topic? A) Typical diet B) Allergy status C) Psychosocial stressors D) Current medication use

Ans: D Feedback: The kidneys are susceptible to the adverse effects of medications because they are repeatedly exposed to substances in the blood. Nephrotoxic medications are a more likely cause of AKI than diet, allergies, or stress.

8. A patient admitted with nephrotic syndrome is being cared for on the medical unit. When writing this patient's care plan, based on the major clinical manifestation of nephrotic syndrome, what nursing diagnosis should the nurse include? A) Constipation related to immobility B) Risk for injury related to altered thought processes C) Hyperthermia related to the inflammatory process D) Excess fluid volume related to generalized edema

Ans: D Feedback: The major clinical manifestation of nephrotic syndrome is edema, so the appropriate nursing diagnosis is "Excess fluid volume related to generalized edema." Edema is usually soft, pitting, and commonly occurs around the eyes, in dependent areas, and in the abdomen.

18. A nurse is performing an admission assessment for an 81-year-old patient who generally enjoys good health. When considering normal, age-related changes to hepatic function, the nurse should anticipate what finding? A) Similar liver size and texture as in younger adults B) A nonpalpable liver C) A slightly enlarged liver with palpably hard edges D) A slightly decreased size of the liver

Ans: D Feedback: The most common age-related change in the liver is a decrease in size and weight. The liver is usually still palpable, however, and is not expected to have hardened edges.

7. An ED nurse has just received a call from EMS that they are transporting a 17-year-old man who has just sustained a spinal cord injury (SCI). The nurse recognizes that the most common cause of this type of injury is what? A) Sports-related injuries B) Acts of violence C) Injuries due to a fall D) Motor vehicle accidents

Ans: D Feedback: The most common causes of SCIs are motor vehicle crashes (46%), falls (22%), violence (16%), and sports (12%).

33. An interdisciplinary team has been commissioned to create policies and procedures aimed at preventing acute hemolytic transfusion reactions. What action has the greatest potential to reduce the risk of this transfusion reaction? A) Ensure that blood components are never infused at a rate greater than 125 mL/hr. B) Administer prophylactic antihistamines prior to all blood transfusions. C) Establish baseline vital signs for all patients receiving transfusions. D) Be vigilant in identifying the patient and the blood component.

Ans: D Feedback: The most common causes of acute hemolytic reaction are errors in blood component labeling and patient identification that result in the administration of an ABO-incompatible transfusion. Actions to address these causes are necessary in all health care settings. Prophylactic antihistamines are not normally administered, and would not prevent acute hemolytic reactions. Similarly, baseline vital signs and slow administration will not prevent this reaction.

5. A nurse is caring for a patient who undergoing preliminary testing for a hematologic disorder. What sign or symptom most likely suggests a potential hematologic disorder? A) Sudden change in level of consciousness (LOC) B) Recurrent infections C) Anaphylaxis D) Severe fatigue

Ans: D Feedback: The most common indicator of hematologic disease is extreme fatigue. This is more common than changes in LOC, infections, or analphylaxis.

15. A patient returns to the floor after a laparoscopic cholecystectomy. The nurse should assess the patient for signs and symptoms of what serious potential complication of this surgery? A) Diabetic coma B) Decubitus ulcer C) Wound evisceration D) Bile duct injury

Ans: D Feedback: The most serious complication after laparoscopic cholecystectomy is a bile duct injury. Patients do not face a risk of diabetic coma. A decubitus ulcer is unlikely because immobility is not expected. Evisceration is highly unlikely, due to the laparoscopic approach.

13. A home health nurse is caring for a patient discharged home after pancreatic surgery. The nurse documents the nursing diagnosis Risk for Imbalanced Nutrition: Less than Body Requirements on the care plan based on the potential complications that may occur after surgery. What are the most likely complications for the patient who has had pancreatic surgery? A) Proteinuria and hyperkalemia B) Hemorrhage and hypercalcemia C) Weight loss and hypoglycemia D) Malabsorption and hyperglycemia

Ans: D Feedback: The nurse arrives at this diagnosis based on the complications of malabsorption and hyperglycemia. These complications often lead to the need for dietary modifications. Pancreatic enzyme replacement, a low-fat diet, and vitamin supplementation often are also required to meet the patients nutritional needs and restrictions. Electrolyte imbalances often accompany pancreatic disorders and surgery, but the electrolyte levels are more often deficient than excessive. Hemorrhage is a complication related to surgery, but not specific to the nutritionally based nursing diagnosis. Weight loss is a common complication, but hypoglycemia is less likely.

17. A patient is being treated on the acute medical unit for acute pancreatitis. The nurse has identified a diagnosis of Ineffective Breathing Pattern Related to Pain. What intervention should the nurse perform in order to best address this diagnosis? A) Position the patient supine to facilitate diaphragm movement. B) Administer corticosteroids by nebulizer as ordered. C) Perform oral suctioning as needed to remove secretions. D) Maintain the patient in a semi-Fowlers position whenever possible.

Ans: D Feedback: The nurse maintains the patient in a semi-Fowlers position to decrease pressure on the diaphragm by a distended abdomen and to increase respiratory expansion. A supine position will result in increased pressure on the diaphragm and potentially decreased respiratory expansion. Steroids and oral suctioning are not indicated.

40. What nursing action should the nurse perform when caring for a patient undergoing diagnostic testing of the renal-urologic system? A) Withhold medications until 12 hours post-testing. B) Ensure that the patient knows the importance of temporary fluid restriction after testing. C) Inform the patient of his or her medical diagnosis after reviewing the results. D) Assess the patient's understanding of the test results after their completion.

Ans: D Feedback: The nurse should ensure that the patient understands the results that are presented by the physician. Informing the patient of a diagnosis is normally the primary care provider's responsibility. Withholding fluids or medications is not normally required after testing.

31. Two units of PRBCs have been ordered for a patient who has experienced a GI bleed. The patient is highly reluctant to receive a transfusion, stating, I'm terrified of getting AIDS from a blood transfusion. How can the nurse best address the patient's concerns? A) All the donated blood in the United States is treated with antiretroviral medications before it is used. B) That did happen in some high-profile cases in the twentieth century, but it is no longer a possibility. C) HIV was eradicated from the US blood supply in the early 2000s. D) The chances of contracting AIDS from a blood transfusion in the United States are exceedingly low.

Ans: D Feedback: The patient can be reassured about the very low possibility of contracting HIV from the transfusion. However, it is not an absolute impossibility. Antiretroviral medications are not introduced into donated blood. The blood supply is constantly dynamic, due to the brief life of donated blood.

38. While performing a patients ordered wound care for the treatment of a burn, the patient has made a series of sarcastic remarks to the nurse and criticized her technique. How should the nurse best interpret this patients behavior? A) The patient may be experiencing an adverse drug reaction that is affecting his cognition and behavior. B) The patient may be experiencing neurologic or psychiatric complications of his injuries. C) The patient may be experiencing inconsistencies in the care that he is being provided. D) The patient may be experiencing anger about his circumstances that he is deflecting toward the nurse.

Ans: D Feedback: The patient may experience feelings of anger. The anger may be directed outward toward those who escaped unharmed or toward those who are now providing care. While drug reactions, complications, and frustrating inconsistencies in care cannot be automatically ruled out, it is not uncommon for anger to be directed at caregivers.

1. A nurse is assessing a patient who has been diagnosed with cholecystitis, and is experiencing localized abdominal pain. When assessing the characteristics of the patients pain, the nurse should anticipate that it may radiate to what region? A) Left upper chest B) Inguinal region C) Neck or jaw D) Right shoulder

Ans: D Feedback: The patient may have biliary colic with excruciating upper right abdominal pain that radiates to the back or right shoulder. Pain from cholecystitis does not typically radiate to the left upper chest, inguinal area, neck, or jaw.

3. A patient has been witness to a disaster involving a large number of injuries. The patient appears upset, but states that he feels capable of dealing with his emotions. What is the nurse's most appropriate intervention? A) Educate the patient about the potential harm in denying his emotions. B) Refer the patient to social work or spiritual care. C) Encourage the patient to take a leave of absence from his job to facilitate emotional healing. D) Encourage the patient to return to normal social roles when appropriate.

Ans: D Feedback: The patient should be encouraged to return to normal social roles when appropriate if he is confident and genuine about his ability to cope. The nurse should use active listening to the patient's concerns and emotions to enable the patient to process the situation. The patient is not necessarily being unrealistic or dishonest. As a result, social work or spiritual care may not be needed. Time away from work may not be required.

12. An obtunded patient is admitted to the ED after ingesting bleach. The nurse should prepare to assist with what intervention? A) Prompt administration of an antidote B) Gastric lavage C) Administration of activated charcoal D) Helping the patient drink large amounts of water

Ans: D Feedback: The patient who has ingested a corrosive poison, such as bleach, is given water or milk to drink for dilution. Gastric lavage is not used to treat ingestion of corrosives and activated charcoal is ineffective. There is no antidote for a corrosive substance such as bleach.

16. A middle-aged woman has sought care from her primary care provider and undergone diagnostic testing that has resulted in a diagnosis of MS. What sign or symptom is most likely to have prompted the woman to seek care? A) Cognitive declines B) Personality changes C) Contractures D) Difficulty in coordination

Ans: D Feedback: The primary symptoms of MS most commonly reported are fatigue, depression, weakness, numbness, difficulty in coordination, loss of balance, spasticity, and pain. Cognitive changes and contractures usually occur later in the disease.

2. A workplace explosion has left a 40-year-old man burned over 65% of his body. His burns are second- and third-degree burns, but he is conscious. How would this person be triaged? A) Green B) Yellow C) Red D) Black

Ans: D Feedback: The purpose of triaging in a disaster is to do the greatest good for the greatest number of people. The patient would be triaged as black due to the unlikelihood of survival. Persons triaged as green, yellow, or red have a higher chance of recovery.

5. A patient who has been exposed to anthrax is being treated in the local hospital. The nurse should prioritize what health assessments? A) Integumentary assessment B) Assessment for signs of hemorrhage C) Neurologic assessment D) Assessment of respiratory status

Ans: D Feedback: The second stage of anthrax infection by inhalation includes severe respiratory distress, including stridor, cyanosis, hypoxia, diaphoresis, hypotension, and shock. The first stage includes flu-like symptoms. The second stage of infection by inhalation does not include headache, vomiting, or syncope.

10. A patient's wound has begun to heal and the blood clot which formed is no longer necessary. When a blood clot is no longer needed, the fibrinogen and fibrin will be digested by which of the following? A) Plasminogen B) Thrombin C) Prothrombin D) Plasmin

Ans: D Feedback: The substance plasminogen is required to lyse (break down) the fibrin. Plasminogen, which is present in all body fluids, circulates with fibrinogen and is therefore incorporated into the fibrin clot as it forms. When the clot is no longer needed (e.g., after an injured blood vessel has healed), the plasminogen is activated to form plasmin. Plasmin digests the fibrinogen and fibrin. Prothrombin is converted to thrombin, which in turn catalyzes the conversion of fibrinogen to fibrin so a clot can form.

29. The nurse has been notified that the ED is expecting terrorist attack victims and that level D personal protective equipment is appropriate. What does level D PPE include? A) A chemical-resistant coverall with splash hood, chemical-resistant gloves, and boots B) A self-contained breathing apparatus (SCBA) and a fully encapsulating, vapor-tight, chemical-resistant suit with chemical-resistant gloves and boots. C) The SCBA and a chemical-resistant suit, but the suit is not vapor tight D) The nurse's typical work uniform

Ans: D Feedback: The typical work uniform is appropriate for Level D protection

13. A patient with a head injury has been increasingly agitated and the nurse has consequently identified a risk for injury. What is the nurses best intervention for preventing injury? A) Restrain the patient as ordered. B) Administer opioids PRN as ordered. C) Arrange for friends and family members to sit with the patient. D) Pad the side rails of the patients bed.

Ans: D Feedback: To protect the patient from self-injury, the nurse uses padded side rails. The nurse should avoid restraints, because straining against them can increase ICP or cause other injury. Narcotics used to control restless patients should be avoided because these medications can depress respiration, constrict the pupils, and alter the patients responsiveness. Visitors should be limited if the patient is agitated.

28. A patient has experienced excessive losses of bicarbonate and has subsequently developed an acidbase imbalance. How will this lost bicarbonate be replaced? A) The kidneys will excrete increased quantities of acid. B) Bicarbonate will be released from the adrenal medulla. C) Alveoli in the lungs will synthesize new bicarbonate. D) Renal tubular cells will generate new bicarbonate.

Ans: D Feedback: To replace any lost bicarbonate, the renal tubular cells generate new bicarbonate through a variety of chemical reactions. This newly generated bicarbonate is then reabsorbed by the tubules and returned to the body. The lungs and adrenal glands do not synthesize bicarbonate. Excretion of acid compensates for a lack of bicarbonate, but it does not actively replace it.

30. The nurse is preparing to admit patients who have been the victim of a blast injury. The nurse should expect to treat a large number of patients who have experienced what type of injury? A) Chemical burns B) Spinal cord injury C) Meningeal tears D) Tympanic membrane rupture

Ans: D Feedback: Tympanic membrane (TM) rupture is the most frequent injury after subjection to a pressure wave resulting from a blast injury because the TM is the body's most sensitive organ to pressure. In most cases, other injuries such as meningeal tears, spinal cord injury, and chemical injuries are likely to be less common.

26. A patient has been brought to the ED after suffering genitourinary trauma in an assault. Initial assessment reveals that the patient's bladder is distended. What is the nurse's most appropriate action? A) Withhold fluids from the patient. B) Perform intermittent urinary catheterization. C) Insert a narrow-gauge indwelling urinary catheter. D) Await orders following the urologist's assessment.

Ans: D Feedback: Urethral catheter insertion when a possible urethral injury is present is contraindicated; a urology consultation and further evaluation of the urethra are required. The nurse would withhold fluids, but urologic assessment is the priority.

6. The nurse is caring for a patient who has a fluid volume deficit. When evaluating this patient's urinalysis results, what should the nurse anticipate? A) A fluctuating urine specific gravity B) A fixed urine specific gravity C) A decreased urine specific gravity D) An increased urine specific gravity

Ans: D Feedback: Urine specific gravity depends largely on hydration status. A decrease in fluid intake will lead to an increase in the urine specific gravity. With high fluid intake, specific gravity decreases. In patients with kidney disease, urine specific gravity does not vary with fluid intake, and the patient's urine is said to have a fixed specific gravity.

23. A nurse is amending a patients plan of care in light of the fact that the patient has recently developed ascites. What should the nurse include in this patients care plan? A) Mobilization with assistance at least 4 times daily B) Administration of beta-adrenergic blockers as ordered C) Vitamin B12 injections as ordered D) Administration of diuretics as ordered

Ans: D Feedback: Use of diuretics along with sodium restriction is successful in 90% of patients with ascites. Beta-blockers are not used to treat ascites and bed rest is often more beneficial than increased mobility. Vitamin B12injections are not necessary.

4. The nurse is planning the care of a patient with a nutritional deficit and a diagnosis of megaloblastic anemia. The nurse should recognize that this patient's health problem is due to what? A) Production of inadequate quantities of RBCs B) Premature release of immature RBCs C) Injury to the RBCs in circulation D) Abnormalities in the structure and function RBCs

Ans: D Feedback: Vitamin B12 and folic acid deficiencies are characterized by the production of abnormally large erythrocytes called megaloblasts. Because these cells are abnormal, many are sequestered (trapped) while still in the bone marrow, and their rate of release is decreased. Some of these cells actually die in the marrow before they can be released into the circulation. This results in megaloblastic anemia. This pathologic process does not involve inadequate production, premature release, or injury to existing RBCs.

36. A patient with a history of incontinence will undergo urodynamic testing in the physician's office. Because voiding in the presence of others can cause situational anxiety, the nurse should perform what action? A) Administer diuretics as ordered. B) Push fluids for several hours prior to the test. C) Discuss possible test results as the patient voids. D) Help the patient to relax before and during the test.

Ans: D Feedback: Voiding in the presence of others can frequently cause guarding, a natural reflex that inhibits voiding due to situational anxiety. Because the outcomes of these studies determine the plan of care, the nurse must help the patient relax by providing as much privacy and explanation about the procedure as possible. Diuretics and increased fluid intake would not address the patient's anxiety. It would be inappropriate and anxiety-provoking to discuss test results during the performance of the test. 37. A nurse is working with a patient who will undergo invasive urologic testing. The nurse has informed the patient that slight hematuria may occur after the testing is complete. The nurse should recommend what action to help resolve hematuria? A) Increased fluid intake following the test B) Use of an OTC diuretic after the test C) Gentle massage of the lower abdomen D) Activity limitation for the first 12 hours after the test [Ans: A Feedback: Drinking fluids can help to clear hematuria. Diuretics are not used for this purpose. Activity limitation and massage are unlikely to resolve this expected consequence of testing.

17. A patient arrives in the emergency department after being burned in a house fire. The patients burns cover the face and the left forearm. What extent of burns does the patient most likely have? A) 13% B) 25% C) 9% D) 18%

Ans: D Feedback: When estimating the percentage of body area or burn surface area that has been burned, the Rule of Nines is used: the face is 9%, and the forearm is 9% for a total of 18% in this patient.

16. A patient is brought to the ER in an unconscious state. The physician notes that the patient is in need of emergency surgery. No family members are present, and the patient does not have identification. What action by the nurse is most important regarding consent for treatment? A) Ask the social worker to come and sign the consent. B) Contact the police to obtain the patient's identity. C) Obtain a court order to treat the patient. D) Clearly document LOC and health status on the patient's chart.

Ans: D Feedback: When patients are unconscious and in critical condition, the condition and situation should be documented to administer treatment quickly and timely when no consent can be obtained by usual routes. A social worker is not asked to sign the consent. Finding the patient's identity is not a priority. Obtaining a court order would take too long.

1. The care team is considering the use of dialysis in a patient whose renal function is progressively declining. Renal replacement therapy is indicated in which of the following situations? A) When the patient's creatinine level drops below 1.2 mg/dL (110 mmol/L) B) When the patient's blood urea nitrogen (BUN) is above 15 mg/dL C) When approximately 40% of nephrons are not functioning D) When about 80% of the nephrons are no longer functioning

Ans: D Feedback: When the total number of functioning nephrons is less than 20%, renal replacement therapy needs to be considered. Dialysis is an example of a renal replacement therapy. Prior to the loss of about 80% of the nephron functioning ability, the patient may have mild symptoms of compromised renal function, but symptom management is often obtained through dietary modifications and drug therapy. The listed creatinine and BUN levels are within reference ranges.

A client present to the emergency department in sickle cell crisis. What intervention by the nurse takes priority? A. Apply an oximetry probe B. Start an IV line C. Give pain medication D. Administer oxygen

Ans: D. Administer oxygen

Which of the following is a type of leukemia? A. Acute myeloid B. Chronic Myeloid C. Chronic lymphocytic D. All of the above

Ans: D. All of the above

A nurse is caring for four clients with leukemia. After hand-off report, which client should the nurse see first? A. Client who has been premedicated for nausea prior to chemotherapy B. Client with a respiratory rate change from 18 to 22 breaths/min C. Client with an unchanged lesion to the lower right lateral malleolus D. Client who had two bloody diarrhea stools this morning

Ans: D. Client who had two bloody diarrhea stools this morning

What condition is often misdiagnosed in the older adult with acute glomerulonephritis? A. Aortic aneurysm B. Cerebrovascular accident C.Transient ischemic attack D. Congestive heart failure

Ans: D. Congestive heart failure

The nurse is aware that a patient with hyperthyroidism will exhibit which serum test result? A. Serum T4 of 11 mcg/dL and a Serum of T3 of 99 B. Decreased radioactive iodine uptake C. Free T4 index of 0.9 ng.dL and Serum T3 of 202 ng/dL D. Serum T4 of 21 mcg/dL and a Serum of T3 of 235 ng/dL

Ans: D. Serum T4 of 21 mcg/dL and Serum of T3 of 235 ng/dL

The nurse is administering a blood transfusion to a patient. Which actions by the nurse are most appropriate? (Select all that apply) A. Taking a full set of vital signs prior to starting the blood transfusion B. Ensuring the patient has an informed consent and a 24 gauge IV access C. Hanging the blood product using normal saline and a filtered tubing set. D. Telling the client someone will remain at the bedside for the first 5 minutes

Ans:A, C


Set pelajaran terkait

Cumulative Test, 6.4.2021, Eng 9 HON

View Set

Pharm - Archer Review (4/4) - Psychiatric Medications, Neurologic, Reproductive and Maternity

View Set

Edhesive 1.6 - 1.9 [Subscribe to youtube.com/oofer]

View Set

CE Shop Section 2: Real Property Ownership

View Set

CHEM 31 - Chapter 1 (For Exam #1)

View Set

Security+ Network Security 20% (part 1)

View Set